Untitled

You might also like

Download as pdf or txt
Download as pdf or txt
You are on page 1of 222

IOQJS (2020-21)

INDIAN OLYMPIAD QUALIFIER IN JUNIOR SCIENCE (IOQJS)

PART : A1
OUT OF FOUR OPTIONS, ONLY ONE IS CORRECT. BUBBLE THE CORRECT OPTION.
1. Gravitational collapse is the contraction of an astronomical object under its own gravity. This draws
the matter inwards towards the centre of gravity. A neutron star is an example of the collapsed core
of a giant star. A certain neutron star of radius 10 km is of mass 1.5 M  . The acceleration due to
gravity on the surface of the neutron star is nearly
(a) 2.0 × 108 m/s2 (b) 2.0 × 1012 m/s2
(c) 2.6 × 1016 m/s2 (d) 2.6 × 1020 m/s2
Answer (b)
2. The tympanic membrane (ear drum) is a very delicate component of the human ear. Typically, its
diameter is 1 cm. The maximum force the ear can withstand is 2.5 N. In case a diver has to enter sea
water of density 1.05 × 103 kg/m3 without any protective gear, the maximum safe depth for the diver
to go into water is about
(a) 12 m (b) 9 m
(c) 3 m (d) 1.5 m
Answer (c)
3. Two illuminated point objects O1 and O2 are placed at a distance 24 cm from each other along the
principal axis of a thin convex lens of focal length 9 cm such that images of both the objects are
formed at the same position. Then the respective distances of the lens from O1 and O2 (in cm) are
(a) 12 and 12 (b) 18 and 6
(c) 14 and 10 (d) 16 and 8
Answer (b)
4. A nuclear reactor is working at 30% efficiency (i.e. conversion of nuclear energy to electrical energy).

In this reactor 235


92 U nucleus undergoes fission and releases 200 MeV energy per atom. If 1000 kW
of electrical power is obtained in this reactor, then the number of atoms disintegrated (undergone
fission) per second in the reactor is
(a) 1.04 × 1017 (b) 6.5 × 1012
(c) 3.125 × 1012 (d) 3.25 × 1032
Answer (a)
5. Two blocks A and B are in contact with each other and are placed on a frictionless horizontal surface.
A force of 90 N is applied horizontally on block A (situation I) and the same force is applied
horizontally on block B (situation II). Mass of A is 20 kg and B is 10 kg. Then the correct statement is

A B B A
90 N 90 N
I II
(a) Since both the blocks are in contact, magnitude of force by block A on B will be 90 N (situation
I) and magnitude of force by block B on A will also be 90 N (situation II)
(b) Magnitude of force by block A on B is 30 N (situation I) and magnitude of force by block B on A is
60 N (situation II)
(c) Magnitude of force by block A on B is 60 N (situation I) and magnitude of force by block B on A is
30 N (situation II)
(d) The 90 N force will produce acceleration of different magnitudes in A and B
Answer (b)

2
IOQJS (2020-21)

6. In the adjoining circuit, R = 5 . It is desired that the voltage across Rx should be 6 V, then the value
of Rx should be

R R R

18 V
Rx
(a) 4  (b) 12 
(c) 16  (d) 20 
Answer (a)
7. In one process for waterproofing, a fabric is exposed to (CH3)2SiCl2 vapors. The vapors react with the
hydroxyl groups on the surface of the fabric or with traces of water to form the waterproofing film
[(CH3)2SiO]n, by the reaction; n(CH3)2SiCl2 + 2nOH–  2nCl– + nH2O + [(CH3)2SiO]n where n stands for
a larger integer. The waterproofing film is deposited on the fabric layer upon layer. Each layer is 6 Å
thick [the thickness of the (CH3)2SiO group]. How much (CH3)2SiCl2 is needed to waterproof one side
of a piece of fabric, 1m by 2m, with a film 300 layers thick? The density of the film is 1.0 g/cm3.
(a) 0.63 g (b) 0.36 g
(c) 6.3 g (d) 3.6 g
Answer (a)
8. Given that at a certain temperature, in 1.5 L vessel, 5.0 mole of A, 7.0 mole of B and 0.1 mole of C
are present. Then the value of equilibrium constant for the reaction: A + B 2C + heat is about
(a) 7.22 × 10–4 (b) 2.31 × 10–4
(c) 7.22 × 10–5 (d) 6.11 × 10–4
Answer (*)
9. An alcohol (A) on dehydration with conc. H2SO4 at a high temperature yields compound (B). On
ozonolysis every molecule of compound (B) yields two molecules of acetaldehyde. Which of the
following is the starting alcohol (A)?
(a) 1- butanol (b) 2- butanol
(c) propanal (d) 2-propanol
Answer (b)
10. In an experiment with 100 mL 0.1 M solution of Copper Chloride, by mistake 5 gms of a mixture
containing equal weights of Tin, Silver, Lead and Calcium, was added. Finally after some time the
solution gets completely decolorized. This is mainly due to :
(a) Silver reacts with Copper Chloride
(b) Calcium reacts with Copper Chloride
(c) All the metals react with Copper Chloride
(d) Only Lead reacts with Copper Chloride forming white precipitate of lead chloride
Answer (b)

3
IOQJS (2020-21)

11. Triclosan (C12H7CI3O2) is an antibacterial and antifungal agent. It is a polychloro phenoxy phenol. It
is widely used as a preservative and antimicrobial agent in personal care products such as soaps,
skin creams, and deodorants etc. A label on a 200 mL hand sanitizer bottle claims that it contain
Triclosan 0.2% w/v. What will be the number of molecules of Triclosan present in the bottle? (NA is
Avogadro’s Number)
(a) 1.4 × 1025 NA (b) 1.4 × 1024 NA
(c) 1.4 × 1023 NA (d) 1.4 × 1022 NA
Answer (*)
12. Suppose that A and B forms compound B2A3 and B2A. If 0.05 mole of B2A3 weighs 12 g and 0.1 mole
of B2A weighs 10 g, what are the atomic weight of A and B respectively?
(a) 70 and 25 (b) 50 and 20
(c) 40 and 30 (d) 30 and 40
Answer (*)
13. If in a wheat mutant, the length of chromosome 1B was found to be 6.7 m instead of 5.0 m,
approximately how many additional base pairs are incorporated in the mutant chromosome?
(a) 0.5 × 104 bp
(b) 5 × 104 bp
(c) 1.7 × 104 bp
(d) 5.78 × 104 bp
Answer (a)
14. Considering following characteristics, identify the correct inheritance pattern from the given options.
 Most affected individuals are male.
 Affected sons result from female parents who are either affected or who are known to be carriers
because they have affected brothers, fathers, or maternal uncles.
 Affected daughters are born to affected fathers and either affected or carrier mothers.
 The sons of affected mothers should be affected.
 Approximately half the sons of carrier mothers should be affected.
(a) Autosomal Recessive Inheritance (b) Autosomal Dominant Inheritance
(c) Sex-Linked Recessive Inheritance (d) Sex-Linked Dominant Inheritance
Answer (c)
15. In a marine ecosystem with rich diversity of fauna, which of the following images would be a correct
representation of pyramid of biomass?

(a) (b)

(c) (d)

Answer (d)

4
IOQJS (2020-21)
16. The transpiration pull is maximum under which of the following conditions?
(a) Closed stomata, low light intensity, humid air
(b) Open stomata, dry air, moist soil
(c) Open stomata, dry air, dry soil
(d) Open stomata, high humidity in air, moist soil
Answer (b)
17. Curcuma longa, Azadirachta indica, Basmati Rice, Indian Ginseng are all related to which of the
following concepts?
(a) Bioterrorism (b) Biomagnification
(c) Biopiracy (d) Biodegradation
Answer (c)
18. Read following criteria carefully.
• Slow evolutionary change relative to similar entities
• Gross similarity to an ancestral fossil
• Very low taxonomic richness today compared to the past
• Phylogenetic inference of specific characters as plesiomorphic
• Phylogenetic inference of genealogical divergence between other groups that diverged in the
distant past
• Known in the fossil record before being discovered alive
These criteria can be used to categorize a group of organisms the most probably into
(a) Connecting links (b) Living fossils
(c) Endangered species (d) Extinct species
Answer (b)
19. If x2 + ax + b = 0 and x2 + bx + a = 0 have one common root, then
(a) a + b = 0 (b) a + b = 1
(c) a + b = –1 (d) a2 + b2 = 1
Answer (c)
20. Six circles each of radius 3 cm are inscribed in an equilateral triangle ABC such that they touch each
other and also touch the sides of the triangle as shown in the adjacent figure. Then height of triangle
ABC is
A

B C

(a) 6(2 3  3) (b) 3(2 3  6)


(c) 3(2 3  3) (d) 6(2  3)
Answer (c)
21. Find the remainder when x51 is divided by x2 – 3x + 2
(a) x (b) (251 – 2)x + 2 – 251
(c) (251 – 1)x + 2 – 251 (d) 0
Answer (c)

5
IOQJS (2020-21)

3
22. If  1 , where x is a real number, then
x2

(a) 2 < x < 5 (b) x < 2 or 5 < x


(c) x < –2 or x > 5 (d) None of these
Answer (b)
23. If 10025 – 25 is written in decimal notations, then the sum of its digits is
(a) 444 (b) 442
(c) 424 (d) 422
Answer (a)
24. ABC is a triangle, the bisector of angle A meets BC in D. The relation between AD, AB and AC is
(a) AD  AB.AC (b) AD > AB.AC

(c) AD  AB.AC (d) AD  AB.AC


Answer (d)

PART : A2
MORE THAN ONE CORRECT OPTIONS. BUBBLE ALL CORRECT OPTIONS ONLY.
25. An infinitely long conductor when carrying current I, produces a magnetic field B around it. If such
a conductor is placed along the X-axis, then the magnitude of B at a distance r is given by the relation
0 2I 
B , (where 0  107 NA 2 is a constant). The following figure shows such an infinitely long
4 r 4
conductor placed along X-axis carrying current I and B at S is 2 × 10–4 T, directed into the plane of
the paper at S. Given r = 1 cm. Then, the correct statements are

Y S
Y2

Y1 90° X
X1 X2

(a) I = 10 A
(b) The number of electrons transported across the cross section of the conductor during time 1 s
is 6.25 × 1019
(c) The direction of current I is from X2 to X1
(d) The electrons will flow in the direction X2 to X1
Answer (a, b, c)
26. The ratio of the charge of an ion or subatomic particle to its mass (q/m) is called specific charge.
Then the correct options are
(a) SI unit of specific charge can be written as A·s /kg.
(b) If all the isotopes of hydrogen are ionized then tritium will have least specific charge among them.
(c) Specific charge of an -particle will be greater than that of an electron.
(d) Specific charge ratio of an electron is 1.75 × 1011 C/kg.
Answer (a, b, d)

6
IOQJS (2020-21)

27. Acetylene torches and burners used by glassblowers produce intense ultraviolet light. Glassblowers
wear special glasses that contain which of the following elements to absorb the UV?
(a) Neodymium (b) Praseodymium
(c) Cerium (d) Didymium
Answer (a, b)
28. Equal lengths of magnesium ribbons are taken in four test tubes A, B, C and D. In test tube A, 1M
acetic acid is added; in test tube B, 1M HCI is added; in test tube C, 1M HNO3 is added; and in test
tube D, 1M NaOH is added. The observed results will be:
(a) The fizzing occurs more vigorously in A (b) The fizzing occurs more vigorously in B
(c) The fizzing occurs more vigorously in C (d) The fizzing occurs more vigorously in D
Answer (b, c)
29. Choose the correct statements from following options.
(a) A robust adaptive immune response is initiated using weakened forms of the bacterium known as
live attenuated vaccines.
(b) Administration of a killed or chemically inactivated virus can trigger a weaker adaptive immune
response, but can be strengthened with booster doses.
(c) A conjugate or multivalent component always reduces immunogenicity of the vaccine.
(d) Inclusion of alum, cytokines, and/or lipids always reduces the immune response to a vaccine.
Answer (a, b)
30. The minimum energy required to exist that is the energy required to perform chemical reactions
even when a person is at rest is called the basal metabolic rate (BMR), which accounts for about 50
to 70 percent of the daily energy expenditure in most sedentary individuals. It is influenced by many
factors. Some statements are made about these factors. Choose the correct statements from the
following options.
(a) Thyroid hormone decreases metabolic rate
(b) Growth hormone increases metabolic rate
(c) Fever decreases metabolic rate
(d) Malnutrition decreases metabolic rate
Answer (b, d)

If 0  x  and 81sin x + 81cos x = 30 , then x =


2 2
31.

 
(a) (b)
6 3
5 2
(c) (d)
6 3
[Useful information : C = 180°, sin (180 – ) = sin , sin  0 when 0    180°]
Answer (a, b, c, d)
32. Given (a – b)2 + (a – c)2 = (b – c)2, then which of the following statements are true?
(a) Equation is valid when b = c and a  c
(b) Equation is valid when a = b
(c) Equation is valid when a = c
(d) Given equation is not valid when a, b and c are distinct
Answer (b, c, d)

‰ ‰ ‰

7
INDIAN ASSOCIATION OF PHYSICS TEACHERS
NATIONAL STANDARD EXAMINATION IN JUNIOR SCIENCE (NSEJS) 2019 20
Q1: Let be the roots of . If for then the
value of is

(a) 2 (b) 3 (c) 4 (d) 5

Q2: The number of triples such that any one of these numbers is added to the product of the other
two, the result is 2, is

(a) 1 (b) 2 (c) 4 (d) infinitely many

Q3: In rectangle ABCD, AB=5 and BC=3. Points F and G are on the line segment CD so that DF = 1
and GC=2. Lines AF and BG intersect at E. What is the area of AEB?

(a) 10 sq. units (b) 15/2 sq. units (c) 25/2 sq. units (d) 20 sq. units

Q4:
In the given figure, two concentric circles are shown with centre O.
PQRS is a square inscribed in the outer circle. It also circumscribes
the inner circle, touching it at points B, C, D and A. What is the ratio
of the perimeter of the outer circle to that of quadrilateral ABCD?

(a) (b) (c) (d)

Q5: How many positive integers N give a remainder 8 when 2008 is divided by N.

(a) 12 (b) 13 (c) 14 (d) 15

Q6: What is the product of all the roots of the equation ?

(a) - 64 (b) - 24 (c) 576 (d) 24.

Q7: LCM of two numbers is 5775. Which of the following cannot be their HCF?
(a) 175 (b) 231 (c) 385 (d) 455
Q8: If are distinct real numbers such that evaluate

(a) (b) (c) (d)

Q9: If the equation has more than two roots, then


the value of is

(a) 2 (b) 3 (c) 1 (d) none of these

Q10: Mr. X with his eight children of different ages is on a family trip. His oldest child, who is 9 years
old saw a license plate with a 4-

(a) 4 (b) 5 (c) 6 (d) 7

Q11: How many numbers lie between 11 and 1111 which divided by 9 leave a remainder 6 and when
divided by 21 leave a remainder 12?

(a) 18 (b) 28 (c) 8 (d) None of these

Q12: Two unbiased dice are rolled. What is the probability of getting a sum which is neither 7 nor 11?

(a) 7/9 (b) 7/18 (c) 2/9 (d) 11/18

Q13: The solution of the equation is

(a) 73 (b) 76 (c) 70 (d) 74

Q14: If then value of is

(a) (b) (c) (d)


Q15: An observer standing at the top of a tower, finds that the angle of elevation of a red bulb on the top
of a light house of height H is . Further, he finds that the angle of depression of reflection of the
bulb in the ocean is . Therefore, the height of the tower is

(a) (b) (c) (d) H

Q16: The sum of the roots of is zero. The product of roots is

(a) 0 (b) (c) (d)

Q17: In the convex quadrilateral ABCD, the diagonals AC and BD meet at O and the measure of angle
AOB is If the areas of triangle AOB, BOC, COD and AOD are 1, 2, 8 and 4 square units
respectively, what is the product of the lengths of the diagonals AC and DB in sq. units?

(a) 60 (b) 56 (c) 54 (d) 64

Q18: If then which of the following is NOT a possible value of


cos ?

(a) 3 (b) -3 (c) -1 (d) -2

Q19: Find the remainder when is divided by

(a) (b) (c) (d) 0

Q20: In an equilateral triangle, three coins of radii 1 unit each are kept so that they touch each other and
also sides of the triangle. The area of triangle ABC (in sq. units) is

(a) (b) (c) 12 + (d) 3 +

Q21: Gymn

(a) Male gamete (b) Ovule (c) Ovary (d) Seeds


Q22: In case of mice coat colour, two genes are responsible for colour

hair shaft (agouti), whereas recessive allele produces no yellow band. There is another allele of A,
known as AY, which is embryonic lethal in homozygous condition only. In an experiment, two
yellow mice were crossed to obtain a progeny of 6 pups. What would be the most probable number
of agouti mice among them?

(a) 0 (b) 2 (c) 4 (d) None of the above

Q23. A stain was developed by a group of scientists to stain a particular cell organelle. The stain was
tested on various tissues derived from an autopsy sample from a mammal. The organelles were
counted. The results showed maximum number of the organelles in cells of brain, lesser in cells of
heart, least in mature sperms and absent in erythrocytes. Identify the organelles from following
options.

(a) Nissl bodies (b) Mitochondria


(c) Golgi bodies (d) Endoplasmic reticulum

Q24: Pinus sylvestris grows at low temperatures in Russia. The plant survives under such freezing
conditions due to the presence of:

(a) Saturated lipids in plasma membrane (b) Glycoproteins in plasma membrane


(b) Glycolipids in plasma membrane (d) Polyunsaturated lipids in plasma membrane

Q25: In an experimental setup, certain pathogen caused a disease in primates with nasal congestion, sore
throat and fever being the common symptoms. The scientists injected an extract from blue-green
mold as the first line of action. However, the symptoms did not subside. The possible causative
agents of the disease were listed out as follows.
i. a virus ii. a fungus iii. a conjugation deficient bacterium iv. a tapeworm
Choose the correct option from the following that indicate the pathogen.

(a) i, ii (b) i, iii (c) ii, iv (d) iii only

Q26: An organism has 27 pairs of homologous chromosomes. In each daughter cell after completion of
mitosis and in each gamete after completion of meiosis II, _________ and _________
chromosomes would be present respectively.

(a) 27 and 27 (b) 54 and 27 (c) 108 and 54 (d) 54 and108


Q27: A group of students was studying development of an organism under controlled laboratory
conditions. Following observations were made by them.
i. The larvae had a rod-like supporting structure that separated the nervous system and the gut.
ii. A prominent central cavity was present in the transverse section of the part of the nervous
system of the larvae; while the adults had cerebral ganglia as the main component of the
nervous system.
iii. The eyes were prominently seen in larvae.
iv. The tails were absent in the adults, which the larvae had.
v. A lot of phagocytic activity was observed before conversion of larvae into adults.
vi. The adults had a cuticular exoskeleton.
The organism under study must be belonging to:

(a) Amphibia (b) Pisces (c) Protochordata (d) Arthropoda

Q28: A process is represented in the adjacent figure. The arrows


indicate the flow of a biochemical reaction. The arrowhead
points to the product, while the base of the arrow indicates
the template biomolecule. What do P, Q, R, and S represent?

(a) P : Replication, Q : Translation, R : Transcription, S : Reverse Transcription


(b) P : Transcription, Q : Replication, R : Reverse Transcription, S : Translation
(c) P : Reverse Transcription, Q : Replication, R : Translation, S : Transcription
(d) P : Reverse Transcription, Q : Replication, R : Transcription, S : Translation

Q29: The whooping cranes were on the verge of extinction with only 21 individuals in wild in 1941. After
conservation measures, the cranes are now included in the endangered category by IUCN. The
highlight of the conservation efforts is the reintroduction of the whooping cranes in wild. This was
possible due to raising of the young cranes in absence of their parents by biologists dressed in crane
costumes. Aircraft Guided bird migration technique was used for teaching the captive-bred cranes
to follow the scientists to learn the migratory route. What type of animal behaviour might be
responsible for these captive-bred cranes to follow the crane costume dressed scientists?

(a) Cognitive learning (b) Habituation (c) Operant conditioning (d) Genetic Imprinting
Q30: In the baking industry, when the dough is prepared, various ingredients are mixed together with the
flour. At one instance, the dough was fermented, but failed to rise sufficiently during the baking
process. Choose the correct cause(s) from following possibilities.
i. The salt was mixed before the fermentation process was completed
ii. The sugar was added in excess
iii. Yeast granules were not activated prior to mixing with the flour.

(a) i, iii (b) iii only (c) i, ii, iii (d) i, ii

Q31: Given below are four statements.


I. Prokaryotic cells are unicellular while eukaryotes are multicellular.
II. Histones are present in eukaryotes and absent in prokaryotes.
III. The nucleoid contains the genetic material in prokaryotes and eukaryotes.
IV. Prokaryotic flagellum is composed of flagellin while eukaryotic flagellum is composed of
tubulin.
Identify which amongst these are false.

(a) I and II (b) III and IV (c) II and III (d) I and III

Q32: The students of a college were working on regeneration using Planaria (Platyhelminthes) and
Asterias (Echinodermata). Planaria was cut in three pieces, namely, a piece with head, with tail
and the middle piece. Asterias (bearing five arms) was cut in such a way that after separation, six
pieces were obtained, namely, an arm with a portion of the central disc, four pieces cut from tips of
each of the remaining arms and the remaining body. The animals were allowed to regenerate
completely. How many Planaria and Asterias respectively will be obtained after the completion of
regeneration in both?

(a) 1, 1 (b) 3, 2 (c) 3, 6 (d) 1, 2

Q33: Fecundity in animal world is the maximum possible ability of an individual to produce offsprings
during its entire lifetime. Following factors were checked for their effect on fecundity of different
animal models.
i. Availability of food during breeding season
ii. Mode of fertilization
iii. Population density
Which of these factor(s) can regulate fecundity?

(a) i, ii (b) ii, iii (c) i, ii, iii (d) None of the above
Q34:

(a) Ethylene (b) Abscisic acid (c) Auxin (d) Gibberellic acid

Q35: In case of peppered moths, pale and dark moths are observed. Pale variety is known to be the wild
type variety. During industrial revolution, industrial melanism led to prevalence of dark variety
around the cities and pale variety continued to be in majority in areas away from the industries.
After enforcement of regulations for controlling pollution, reappearance of pale moths in majority
was observed around cities again. Driving force(s) for these adaptive changes is/are:
i. Increased pollution around industries
ii. A stable transposition of a gene in moths
iii. Limitations of the vision of birds to differentiate dark moths on darkened barks and pale moths
in presence of lichens
iv. Ability of lichens to grow on barks in less polluted areas only.

(a) i, iv (b) i, iii, iv (c) i, ii (d) i, ii, iii and iv

Q36: Any damage or injury to a particular area causes nociceptors to release some chemicals, which carry
the signal to the higher centres in the nervous system for the processing and a subsequent action.
However, there is a difference in the way in which the stimulus is received which is related to the
acuity of the detection. Fingertips are more sensitive as compared to the forearm. Following
reasons for the observed phenomenon were suggested.

i. The receptive fields in the fingertip are smaller


ii. The number of nociceptors per receptive field in the forearm is lesser
iii. The amount of prostaglandins released by the nociceptors per receptive field is more in
fingertips
The most probable reason(s) for this may be:

(a) i (b) i, iii (c) ii, iii (d) i, ii, iii

Q37: On a study tour, plants with leathery leaves with thick cuticle, vivipary, salt glands, apogeotropic
roots, and stomata limited to abaxial surface were observed. The plants might be:

(a) Bromeliads (b) Cycads


(c) Mangroves (d) None of the above
Q38:

Rate of photosynthesis in hydrophytes depends


on various parameters. The adjacent graph
shows the effect of one parameter (while
keeping all the others constant) on the rate of
photosynthesis. Rate of photosynthesis is
plotted on Y axis. Identify the parameter
which is plotted along X axis:

(a) light intensity (b) wavelength (c) temperature (d) CO2 concentration

Q39: long bacterial cell was magnified and drawn to a dimension of 6 cm. How many times has it
been magnified?

(a) 1.5 x 103 (b) 15 x 104 (c) 1.5 x 104 (d) 1.5

Q40: Four different human body fluid samples were subjected to quantification of hydrogen ion
concentration. mEq/L is the unit of measurement for hydrogen ion concentration. The results of the
experiment were as follows:
Sample A: 1.6 X 102 units Sample B: 4.5 X 10 5 units
3
Sample C: 1 X 10 units Sample D: 3 X 10 2 units
Identify the samples in sequence from A to D.

(a) Gastric HCl, Venous blood, Intracellular Fluid, Urine


(b) Venous blood, Intracellular Fluid, Gastric HCl, Urine
(c) Urine, Gastric HCl, Venous blood, Intracellular Fluid
(d) Intracellular Fluid, Urine, Gastric HCl, Venous blood

Q41: Four gram of mixture of calcium carbonate and sand is treated with excess of HCl and 0.880 g of
carbon-di-oxide is produced. What is the percentage of calcium carbonate in original mixture?
(a) 40 % (b) 50% (c) 55% (d) 45%

Q42: How many sigma bonds are present between any two carbon atoms in fullerenes?

(a) 1 (b) 2 (c) 3 (d) 4


Q43:
Gammaxene insecticide powder is
prepared by the reaction given in the
adjacent box. If 78 g of benzene when
reacted with 106.5 g of chlorine, how
much Gammaxene would be formed?

(a) 140 g (b) 154.5 g (c) 145.5 g (d) 160 g

Q44: An element Y is a white translucent solid at room temperature and exhibits various allotropic forms.
Some compounds of element Y find application in agricultural industry. Y forms two solid oxides
which dissolve in water to form comparatively weak acids. The element Y is:

(a) Sulphur (b) Nitrogen (c) Phosphorous (d) Carbon

Q45: A student was studying reactions of metals with dilute NaOH at room temperature. The student took
dilute NaOH in four different test tubes and added copper powder to test tube A, zinc dust to test
tube B, aluminium powder to test tube C and iron powder to test tube D and observed
effervescence in__.

(a) Test tubes A & B (b) Test tubes B & C (c) Test tubes C & D (d) Test tubes A & D

Q46. Which of the following polymeric material will be ideal for remoulding?

(a) Polythene and Melamine (b) Polyvinyl chloride and Polythene


(c) Melamine and Bakelite (d) Bakelite and Polyvinyl chloride

Q47: A magician performed following act: He dipped Rs. 50 note in a 50% solution of alcohol in water
and held it on the burning flame, but the note did not burn. The reason behind this is -

(a) The alcohol kept on dousing the fire (b) Air required for burning was not available
(c) The Rs 50 note failed to reach ignition temperature (d) The Rs. 50 note is fire proof

Q48. Which of the following is iso-structural with CO2?

(a) NO2 (b) N2O4 (c) NO (d) N2O


Q49: Substance X is white crystalline solid which melts after 10 seconds on burner flame. It is soluble in
water and insoluble in CCl4. It is a poor conductor of electricity in molten state as well as in the
form of aqueous solution, hence we conclude that substance X is
(a) an ionic compound (b) a non polar covalent compound
(c) a polar covalent compound (d) a pure element

Q50: In a beaker 50 ml of a normal HCl solution was taken and NH3 gas was passed through it for some
time. The contents of the beaker were then titrated, which required 60 ml of semi normal NaOH
solution. How much ammonia was passed through the beaker?

(a) 0.85 g (b) 0.34 g (c) 0.51 g (d) 0.4 g

Q51: Which is the correct order of metals with reference to their melting point in increasing order?

(a) Hg, Ga, Li, Ca (b) Ca, Li, Ga, Hg (c) Hg, Li, Ga, Ca (d) Hg, Ga, Ca, Li

Q52: Sodium tungstate has formula Na2WO4, lead phosphate has formula Pb3(PO4)2, formula for lead
tungstate should be:
(a) PbWO4 (b) Pb2(WO4)3 (c) Pb3(WO4)2 (d) Pb3(WO4)4

Q53: What is the ratio of reducing agent to oxidizing agent, if the following reaction is correctly
balanced? NH3 + O2 2O

(a) 4:5 (b) 5:4 (c) 5:3 (d) 3:5

Q54: Arrange following solutions in increasing hydronium ion concentration. The solutions are:
(P) 0.1 M HCl (Q) 0.1 M H2SO4 (R) 0.001 M NH4OH (S) 0.001 M Ca(OH)2.
The correct order will be

(a) P > Q > R > S (b) Q > P > S > R (c) S > R > Q > P (d) S > R > P > Q

Q55: In one litre of pure water, 44.4 g of calcium chloride is dissolved. The number of ions in one mL of
the resultant solution is:

(a) 7.23 ×1023 (b) 7.23 ×1020 (c) 4.82 ×1023 (d) 4.82×1020
Q56: A zinc rod was dipped in 100 cm3 of 1M copper chloride solution. After certain time the molarity of
Cu2+ ions in the solution was found to be 0.8 M. If the weight of zinc rod is 20 g, then the molarity
of chloride ions is_____.

(a) 2 M (b) 1.5 M (c) 1 M (d) 0.5 M

Q57: When four dilute solutions of (I) vinegar, (II) common salt, (III) caustic soda and (IV) baking soda
are tested with universal indicator which will be the correct observation

(a) I- Green, II - Violet, III - Blue, IV - Red (b) I - Green, II - Blue, III -Violet, IV- Red
(c) I - Red, II - Green, III - Violet, IV - Blue (d) I-Red, II- Violet, III - Green, IV Blue

Q58: Which of the following species is / are isoelectronic with Neon?


(i) N3 (ii) Mg2+ (iii) K+ (iv) Ca2+

(a) only (iv) (b) only (ii) (c) both (i) & (ii) (d) both (i) & (iii)

Q59: Which of the following gases will have equal volume at STP, if the weight of gases is
14.0 g?
(i) N2O (ii) NO2 (iii) N2 (iv) CO

(a) (i) & (ii) (b) (ii) & (iii) (c) (i) & (iii) (d) (iii) & (iv)

Q60: Which of the following are not ionic?


(i) AlCl3 (ii) CaCl2 (iii) MgCl2 (iv) LiCl

(a) (i) and (iv) (b) (i) and (ii) (c) (ii) and (iii) (d) (iii) and (iv)

Q61: Apples dropping from apple trees were observed by many people before Newton. But why they fall,
was explained by Isaac Newton postulating the law of universal gravitation. Which of the following
statements best describes the situation?

(a) The force of gravity acts only on the apple


(b) The apple is attracted towards the surface of the earth
(c) Both earth and apple experience the same force of attraction towards each other
and (c) is absurd
Q62:
A rectangular metal plate, shown in the adjacent

uniformly distributed over it. Then how much is the


charge over the shaded area? No part of metal plate
is cut. (Circles and the diagonal are shown for clarity
only.

(a) 45 µC (b) 450 µC (c) 15 µC (d) 150 µC

Q63:

In the adjacent circuit, the voltages across AD, BD


and CD are 2 V, 6 V and 8 V respectively. If
resistance RA = 1
RB and RC are _____ and ____ respectively.

(a)

Q64: A new linear scale of temperature measurement is to scale on which


the freezing and boiling points of water are 20 Z and 220 Z respectively. What will be the
temperatu scale corresponding to a temperature of 20º C on the Celsius scale?

(a) 10 Z (b) 20 Z (c) 40 Z (D) 60 Z

Q65: Consider the motion of a small spherical steel body of mass m, falling freely through a long column
of a fluid that opposes its motion with a force proportional to its speed. Initially the body moves
down fast, but after some time attains a constant velocity known as terminal velocity. If weight mg,
opposing force (Fv) and buoyant force (Fb) act on the body, then the correct equation relating these
forces, after the terminal velocity is reached, is:

(a) mg + Fv = Fb (b) mg = Fv - Fb (c) mg = Fv + Fb (d) none


Q66: A piece of wire P and three identical cells are connected in series. An amount of heat is generated in
a certain time interval in the wire due to passage of current. Now the circuit is modified by
replacing P with another wire Q and N identical cells, all connected in series. Q is four times longer
in length than P. The wire P and Q are of same material and have the same diameter. If the heat
generated in second situation is also same as before in the same time interval, then find N.

(a) 4 (b) 6 (c) 16 (d) 36

Q67:

Some waveforms among


I, II, III and IV superpose
(add graphically) to
produce the waveforms
P, Q, R and S. Among
the following, match the
pairs that give the correct
combinations:

Resultant Superposition of

P (K) III and IV


Q (L) II and IV
R (M) I, II and III
S (N) I and IV
(O) II and III

(a)
(c)

Q68: At any instant of time, the total energy (E) of a simple pendulum is equal to the sum of its kinetic
energy and potential energy , where, m is the mass, v is the velocity, x is the
displacement of the bob and k is a constant for the pendulum. The amplitude of oscillation of the
pendulum is 10 cm and its total energy is 4 mJ. Find k.

(a) 1.8 Nm-1 (b) 0.8 Nm-1 (c) 0.5 Nm-1 (d) data insufficient
Q69:
A rigid body of mass m is suspended from point O
using an inextensible string of length L. When it is
displaced through an angle , what is the change in
the potential energy of the mass? (Refer adjacent
figure.)

(a) mg L (1- cos ) (b) mg L (cos - 1) (c) mg L cos (d) mg L (1- sin )

Q 70:

Refer to the adjacent figure. A variable


force F is applied to a body of mass 6 kg
at rest. The body moves along x - axis as
shown. The speed of the body at x = 5 m
and x = 6 m is ___ and ___ respectively.

(a) 0 m/s, 0 m/s (b) 0 m/s, 2 m/s (c) 2 m/s, 2 m/s (d) 2 m/s, 4 m/s

Q71: When a charged particle with charge q and mass m enters uniform magnetic field B with velocity v
at right angles to B, the force on the moving particle is given by qvB. This force acts as the
centripetal force making the charged particle go in a uniform circular motion with radius r = .
Now if a hydrogen ion and a deuterium ion enter the magnetic field with velocities in the ratio 2:1
respectively, then the ratio of their radii will be__________.
(a) 1:2 (b) 2:1 (c) 1:4 (d) 1:1

Q72: A piece of ice is floating in water at 4º C in a beaker. When the ice melts completely, the water level
in the beaker will

(a) rise (b) fall (c) remains unchanged (d) unpredictable


Q73: In a screw-nut assembly (shown below) the nut is held fixed in its position and the screw is allowed
to rotate inside it. A convex lens (L) of focal length 6.0 cm is fixed on the nut. An object pin (P) is
attached to the screw head. The image of the object is observed on a screen Y. When the screw
head is rotated through one rotation, the linear distance moved by the screw tip is 1.0 mm. The
observations are made only when the image is obtained in the same orientation on the screen. At a
certain position of P, the image formed is three times magnified as that of the pin height. Through
how many turns should the screw head be rotated so that the image is two times magnified?

(a) 8 (b) 10 (c) 12 (d) 14

Q74: A school is located between two cliffs. When the metal bell is struck by school attendant, first
echo is heard by him after 2.4 s and second echo follows after 2.0 s for him at the same position
near the bell. If the velocity of sound in air is 340 ms-1 at the temperature of the surroundings, then
the distance between the cliffs is approximately____.

(a) 0.488 km (b) 0.751 km (c) 1.16 km (d) 1.41 km

Q75: The triangular face of a crown glass prism ABC is isosceles. Length AB = length AC and the
rectangular face with edge AC is silvered. A ray of light is incident normally on rectangular face
with edge AB. It undergoes reflections at AC and AB internally and it emerges normally through
the rectangular base with edge BC. Then angle BAC of the prism is____.

(a) 24º (b) 30º (c) 36º (d) 42º

Q76: The radius of curvature of a convex mirror is 'x'. The distance of an object from focus of this mirror
y

(a) y2 / 4x (b) x2 / y (c) x2 / 4y (d) 4y2/ x


Q77: A physics teacher and his family are travelling in a car on a highway during a severe lightning
storm. Choose the correct option:

(a) Safest place will be inside the car as the charges due to lightning tend to remain on the metal
sheet / skin of the vehicle if struck by lightning.
(b)
accumulate on the surface and will generate a strong electric field inside the car.
(c)
a path to the charges for earthing.
(d) It is safer to exit the car and stand on open ground

Q78: A conductor in the form of a circular loop is carrying current I. The direction of the current is as
shown. Then which figure represents the correct direction of magnetic field lines on the surfaces of
the planes XY and XZ. (Consider those surfaces of the XY and XZ planes which are seen in the
figure.)

(b)
(a)

(d)
(c)
Q79: A particle experiences constant acceleration for 20 s after starting from rest. If it travels a distance
S1 in the first 10 s and distance S2 in the next 10 s, the relation between S1 and S2 is:

(a) S2=3S1 (b) S1=3S2 (c) S2= 2S1 (d) S1=10S2

Q80: A sound wave is produced by a vibrating metallic string stretched between its ends.
Four statements are given below. Some of them are correct.
(P) Sound wave is produced inside the string.
(Q) Sound wave in the string is transverse.
(R) Wavelength of the sound wave in surrounding air is equal to the wavelength of the transverse
wave on the string.
(S) Loudness of sound is proportional to the square of the amplitude of the vibrating string.
Choose the correct option.

(a) P (b) R and S (c) P and Q (d) S


1. Trigonometric ratios are defined for angles greater than 90º also.

2. The following identities may be useful:

3. Periodic Table of elements


Answer key for NSEJS held on 17 Nov. 2019
QUESTION QUESTION QUESTION QUESTION
Correct response Correct response Correct response Correct response
NUMBER ↓ NUMBER ↓ NUMBER ↓ NUMBER↓
SUBJECT → MATHS PHY CHEM BIO SUBJECT → BIO MATHS PHY CHEM SUBJECT → CHEM BIO MATHS PHY SUBJECT → PHY CHEM BIO MATHS
QP CODE → 51 52 53 54 QP CODE → 51 52 53 54 QP CODE → 51 52 53 54 QP CODE → 51 52 53 54
1 d c b c 21 c d c c 41 b b d c 61 c b b d
2 b a c b 22 b a b a 42 a b b a 62 a c b c
3 c b a b 23 b d a b 43 c d c b 63 b b c c
4 c d b d 24 d d b b 44 c b c a 64 d c b b
5 d b c b 25 b b b c 45 b c d b 65 c a d a
6 a a b d 26 b c a c 46 b d a b 66 b b d d
7 d b c d 27 c c d DELETED 47 c d d c 67 b c c d
8 d c c c 28 d d c a 48 d c d a 68 b c d d
9 a c b c 29 d a c b 49 c d a c 69 a b d b
10 b b a d 30 c b d c 50 b b c b 70 c a b a
11 a d a b 31 d a b c 51 a c b a 71 d d c a
12 a b a c 32 b a b b 52 a b a d 72 a a b c
13 a c d b or d 33 c a c a 53 a d a a 73 b a d a
14 a a DELETED a 34 d a a a 54 DELETED b or d a c 74 c DELETED c a
15 a a a d 35 b or d c a a 55 b c a b 75 c a c a
16 c c c b 36 d a a d 56 a c a c 76 c c b or d a
17 a c b b 37 c a c b 57 c d a d 77 a b a a
18 d a c c 38 b d c c 58 c b d a 78 a c c d
19 c a d d 39 c c a d 59 d c c c 79 a d d c
20 b d a c 40 a b d a 60 a a b d 80 d a b b

Corrected answer
NSEJS-2018 (IJSO STAGE-I)
Date of Examination : 18th November, 2018
PAPER CODE - JS511

1. A tiny ball of mass m is initially at rest at height H above a cake of uniform thickness h. At some
moment the particle falls freely, touches the cake surface and then penetrates in it at such a constant
rate that its speed becomes zero on just reaching the ground (bottom of the cake). Speed of the ball at
the instant it touches the cake surface and its retardation inside the cake are respectively

æH ö æH ö
(a) 2gh and g ç - 1 ÷ (b) 2g(H - h) and g ç - 1 ÷
èh ø èh ø

æh ö æh ö
(c) 2gh and g ç - 1 ÷ (d) 2g(H - h) and g ç - 1 ÷
èH ø èH ø

2. Two sound waves in air have wavelengths differing by 2 m at a certain temperature T. Their notes
have musical interval 1.4. Period of the lower pitch note is 20 ms. Then, speed of sound in air at this
temperature (T) is
(a) 350 m/s (b) 342 m/s (c) 333 m/s (d) 330 m/s

3. Two plane mirrors M1 & M2 have their reflecting faces inclined at q. Mirror M1 receives a ray AB,
reflects in at B and sends it as BC. It is now reflected by mirror M2 along CD, as shown in the figure.
Total angular deviation d suffered by the incident ray AB is

M1
D
A

B
d1

q d2 M2
O
C

(a) d = 90° + 2q (b) d = 180° + 2q (c) d = 270° – 2q (d) d = 360° – 2q

4. In the adjacent figure, line AB is parallel to screen S. A linear obstacle PQ between the two is also parallel
to both. AB, PQ and Screen S are coplanar. A point source is carried from A to B, along the line AB. What
will happen to the size of the shadow of PQ (cast due to the point source) on the screen S?

1
A
P

Q
B

(a) It will first increase and then decrease


(b) It will first decrease and the increase.
(c) It will be of the same size for any position of the point source on the line AB.
(d) Umbra will increase and penumbra will decrease till central position.

5. Two particles P 1 and P 2 move towards origin O, along X and Y-axes at constant speed u 1 and u2
respectively as shown in the figure. At t = 0, the particles P1 and P2 are at distances a and b respectively
from O. Then the instantaneous distance s between the two particles is given by the relation.
O
P1
u1

u2

P2
(a) s = [a + b + (u + u )t – 2t(au1 + bu2)]
2 2
1
2
2
2 2 1/2
(b) s = [a2 + b2 + (u12 + u22)t2 – 2t(bu1 + au2)]1/2
(c) s = [a2 + b2 + (u + u )t + 2t(au1 + bu2)]1/2
1
2
2
2 2
(d) s = [a2 – b2 + (u12 + u22)t2 – 2t(au1 + bu2)]1/2

6. An electric generator consume some oil fuel and generates output of 25 kW. Calorific value (amount
of heat released per unit mass) of the oil fuel is 17200 kcal/kg and efficiency (output to input ratio) of
the generator is 0.25. Then, mass of the fuel consumed per hour and electric energy generated per ton
of fuel burnt are respectively
(a) 0.5 kg, 20000 kWh (b) 0.5 kg, 5000 kWh
(c) 5 kg, 5000 kWh (d) 5 kg, 20000 kWh

7. Image is obtained on a screen by keeping an object at 25 cm and at 40 cm in front of a concave mirror.


Image in the former case is four times bigger than in the latter. Focal length of the mirror must be
____.
(a) 12 cm (b) 20 cm (c) 24 cm (d) 36 cm

8. A glass cube of refractive index 1.5 and edge 1 cm has a tiny black spot at its center. A circular dark
sheet is to be kept symmetrically on the top surface so that the central spot is not visible form the top.

1 1 1
Minimum radius of the circular sheet should be (Given : = 0.707, = 0.577, = 0.447)
2 3 3
(a) 0.994 cm (b) 0.447 cm (c) 0.553 cm (d) 0.577 cm

2
9. A metal rod of length L at Temperature I, when heated to temperature T', expands to new length L'.
These quantities are related as L' = L(1 + a[T' – T] where a is a constant for that material and called as
coefficient of linear expansion. Correct SI unit of a is
(a) m-K–1 (b) m-K (c) K–1 (d) a is a pure number

10. A paramedical staff nurse improvises a second's pendulum (time period 2s) by fixing one end of a
string of length L to a ceiling and the other end to heavy object of negligible size. Withing 60 oscillations
of this pendulum, she finds that the pulse of a wounded soldier beats 110 times. A symptom of
bradycardia is pulse < 60 per minute and that of tachycardia is > 100 per minute. Then the length of
the string is nearly _______ and soldier has symptoms of ______.
(a) 1 m, bradycardia (b) 4 m, bradycardia (c) 1 m, tachycardia (d) 4 m, tachycardia

11. Each resistance in the adjacent circuit is R W. In order to have an integral value for equivalent resistance
between A and B, the minimum value of R must be :

A B

(a) 4 W (b) 8 W (c) 16 W (d) 29 W


th
æ 3ö
12. A block of wood floats on water with ç ÷ of its volume above water. It is now made to float on a salt
è8ø
solution of relative density 1.12. The fraction of its volume that remains above the salt solution now,
is nearly _______.
(a) 0.33 (b) 0.44 (c) 0.67 (d) 0.56

13. Suppose our scientific community had chosen force, speed and time as the fundamental mechanical
quantities instead of length, mass and time respectively and they chose the respective units of magnitudes
1
10 N, 100 m/s and s. Then the unit of mass in their system is equivalent to _______ in our system.
100
(a) 10 kg (b) 10–3 kg (c) 10 kg (d) 10–1 kg

14. Two equally charged identical pith balls are suspended by identical massless strings as shown in the
adjacent figure. If this set up in on Mercury (g = 3.7 m/s2), Earth (g = 9.8 m/s2) and Jupiter (g = 24.5 m/s2),
then angle 2q will be _______.

3
q q

1.0 m 1.0 m

+ +
Q Q

(a) maximum on Mercury


(b) maximum on Earth, as it has atmosphere
(c) maximum on Jupiter
(d) the same on any planet as Coulomb force is independent of gravity

15. Three objects of the same material coloured white, blue and black can withstand temperature up to
2000°C. All these are heated to 1500°C and viewed in dark. Which option is correct?
(a) White object will appear brightest
(b) Blue object will appear brightest
(c) Black object will appear brightest
(d) Being at the same temperature, all look equally bright

16. A car running with a velocity of 30 m/s reaches midway between two vertical parallel walls separated
by 360 m, when the driver sounds the horn for a moment. Speed of sound in air is 330 m/s. After
blowing horn, the first three echoes will be heard by the driver respectively at ________.
(a) 1.2 s, 2.4 s, 3.0 s (b) 1.0 s, 2.4 s, 3.0 s (c) 1.0 s, 2.0 s, 3.0 s (d) 1.2 s, 2.4 s, 3.6 s

17. Choose correct option from the following statements from electrostatics :
(I) If two copper spheres of same radii, one hollow and the other solid are charged to the same
electrical potential, the solid sphere will have more charge.
(II) A charged body can attract another uncharged body.
(III) Electrical lines of force originating from like charges will exert a lateral force on each other, while
those originating from opposite charges can intersect each other.
(a) Only (I) is correct. (b) Only (II) is correct.
(c) Only (I) and (II) are correct. (d) All (I), (II) and (III) are correct.

18. Refer the adjacent circuit. The voltmeter reads 117 V and ammeter reads 0.13 A. If the resistance of
voltmeter and ammeter are 9 kW and 0.015 W respectively, the value of R is _____.

A
R
(a) 500 W (b) 1 kW (c) 1.5 kW (d) 2 kW

4
19. A bar magnet is allowed to fall freely from the same height towards a current carrying loop along its
axis, as shown in the four situations I to IV. Arrows show direction of conventional current. Choose
the situations in which the potential energy of the magnet coil interaction is maximum _____.

I N II S III N IV S

S N S N

(a) I, III (b) I, IV (c) II, IV (d) II, III

20. A beaker is completely filled with water at 4°C. Consider the following statements:
(I) Water will overflow if the beaker is cooled for some time.
(II) Water will overflow if the beaker is heated for some time.
Select correct option regarding (I) and (II).
(a) Only (I) is correct (b) Only (II) is correct
(c) Both (I) and (II) are correct (d) Neither (I) nor (II) is correct

21. P3– has a larger radius than atom of P because


(a) There is greater coulombic attraction between the nucleus and electrons in the p3– ion.
(b) The core electrons in p3– exert a weaker shielding force than those of a neutral atom.
(c) The nuclear charge is weaker in p3– than it is in P.
(d) The electrons in p3– have a greater coulombic repulsion than those in P atom.

22. A substance is dissolved in water, forming a 0.5 molar solution. If 4.0 L of solution contains 240 g of
the substance, what is the molecular mass of the substance?
(a) 60 g/mole (b) l20 g/mole (c) 240g/mole (d) 480 g/mole

23. A car battery was kept for charging and after getting fully charged density of the battery acid (H2SO4)
was measured and found to be 1.28 g cm–3. If initial molarity of battery acid was 4.2 M then mass
percentage will be around.
(a) 28% (b) 30% (c) 32% (d) 34%

24. Element "X" with atomic mass 10 was allowed to react completely with element "Y" of atomic mass
20 to form a compound. When this compound was analysed it was found that it contains 60% of X
and 40% of Y by weight. The simplest formula of this compound will be
(a) X3 Y (b) x2Y3 (c) Y3 x (d) x6Y4

25. 4.095 X 1024 nitrogen atoms are filled in an enclosed gas cylinder of capacity two litre. The number of
moles of nitrogen gas in the cylinder is ____.
(a) 14.7 (b) 6.8 (c) 3.4 (d) 2.9

5
26. When surface tension experiment with capillary tube is performed, water rises up to 0.1 m. If the
experiment is carried out in space, water will rise in capillary tube _____.
(a) up to height of 0.1 m (b) up to height of 0.2 m
(c) up to height of 0.98 m (d) along its full length

27. Deepa was studying properties of gases. She took a flask and filled it with sulphur dioxide gas, and
weighed it at temperature T and pressure P. The weight of the flask containing the gas was found to be
W1 . She then flushed the flask, cleaned and filled it with methane at the same temperature and pressure.
The weight of the flask containing oxygen was found to be W2. She repeated the process with oxygen
under the same conditions and found the weight to be W3 : The ratio of the weights W1 : W2 : W3 is
(a) 2 : 1 : 4 (b) 4 : 2 : 1 (c) 4 : l : 2 (d) 1 : 2 : 4

28. Four gas jars filled with sulphur dioxide gas were inverted into troughs of water by four students P, Q,
R, S. The following observations and inference were reported by them.
P : Water did not enter the gas jar and sulphur dioxide is soluble in water.
Q : Water rushed into the gas jar and sulphur dioxide is soluble in water.
R : Water did not enter in the gas jar and sulphur dioxide is insoluble in water.
S : A small amount of water entered the gas jar slowly and sulphur dioxide is sparingly soluble in
water.
Then the correct set of observations and inference is reported by,
(a) P (b) Q (c) R (d) s

29. A solution of pure aluminium sulphate containing 0.170 g of aluminium ions is treated with excess of
barium hydroxide solution. Total weight of the precipitate will be:
(a) 0.5 g (b) 2.7 g (c) 1.7 g (d) 0.54 g

30. A region of one square meter area was given to each Suhas, Bobby, Sandy and Kimi in a garden. The
daffodil plants grow best in the soil having a pH range of 6.0 to 6.5. If the soil has a pH 4.5, to grow
daffodils, Suhas added common salt, Bobby added sodium phosphate, Sandy added aluminium
sulphate and Kimi added ammonium chloride in their allotted area. Who was successful in growing
daffodil?
(a) Suhas (b) Bobby (c) Sandy (d) Kimi

31. Electrons in the last shell of X, Y, W and Z are 2, 6, 4 and 1 respectively. Which of the following
statement is correct?
(a) melting point of compound formed by X and Y is more than that of by W and Z.
(b) compound formed by X and Y is more volatile than that of by W and Z.
(c) melting point of compound formed by X and Z is more than that of by W and Y.
(d) Incomplete information so inference cannot be drawn.

32. W g of pure coal was combusted in pure dry oxygen. The carbon dioxide gas obtained was absorbed
in 0.1 M KOH solution. The complete absorption of CO2 required 5 cm3 of 0.1 M KOH. The amount
of coal combusted is
(a) 3 mg (b) 6 mg (c) 11 mg (d) 12 mg

6
33. Sulphur di-oxide gas and ammonia gas were mixed in different proportions. The pair of gases containing
same number of molecules at NTP is __________.
(a) 1120 cm3 of SO2 + 0.85 g of ammonia (b) 0.25 g mole of SO2 + 2240 cm3 of ammonia
(c) 1680 cm3 of SO2 + 1.7 g of ammonia (d) 0.25 g mole of SO2 + 0.85 g of ammonia

34. A strip of iron with mass 15.5 g is placed in a solution containing 21.0 g copper sulphate. After some
time the reaction stops. Iron strip was found to have mass 8.5 g. The mass of copper formed was
found to be 8.60 g. Find the mass of ferrous sulphate formed in this reaction.
(a) 19.40 g (b) 18.40 g (c) 17.40 g (d) 16.40 g

35. Sonu has N/2 HCl solution and Monu has N/10 HCl solution. They are asked to prepare 2 litres of
N/5 HCl solution. What volume of two solutions be mixed?
(a) (0.5 + 1.5) litre (b) (1.0 + 1.0) litre (c) (0.3 + 1.7) litre (d) (0.2 + 1.8) titre

36. A solution (P) was prepared by dissolving 6.3 g of oxalic acid in 100 ml water. 25 ml of this solution
was taken and was further diluted to 250 ml to prepare solution (Q). What weight of NaOH in ppm will
be required to neutralize 10 ml of solution (Q)?
(a) 10 ppm (b) 20 ppm (c) 40 ppm (d) 80 ppm

37. Which of the following can improve the quality of petrol?


(a) n heptane (b) benzene (c) n hexadecane (d) iso-octane

38. 2KBrO3 + 12H+ + l0e– ® Br2 + 6H2O + 2K+


From above reaction the equivalent weight of KBrO3 can be calculated as (M is molecular weight of
KBrO3 )
(a) M/5 (b) M/10 (c) M/12 (d) M/2

39. Shaila took about l0 cm3 of a diluted Potassium hydrogen carbonate solution in a test tube. To this
solution she added few drops of universal indicator. The colour of the solution turned:
(a) orange (b) geen (c) blue (d) yellow

40. Which of the following is incorrect?


(a) Chalcocite – Copper (b) Magnetite – Iron
(c) Calamine – Aluminium (d) Galena – Lead

41. Let AB be a diameter of circle C1 of radius 30 cm and with center O. Two circes C2 and C3 of raddi 15
cm and 10 cm touch C1 internally at A and B respectively. A fourth circle C4 touches C1,C2 and C3 .
What is the largest possible radius of C4?
(a) 12 cm (b) 15 cm (c) 20 cm (d) 30 cm

42. A 5 × 5 × 5 cube is built using unit cubes. How many different cuboids (that differ in at least one unit
cube) can be formed using the same number of unit cubes?
(a) 1000 (b) 1728 (c) 2730 (d) 3375

7
43. What is the largest value of the positive integer k such that k divides n2(n2 – 1) (n2 – n – 2) for every
natural number n?
(a) 6 (b) 12 (c) 24 (d) 48

44. A person kept rolling a regular (six faced) die until one of the numbers appeared third time on the top.
This happened in 12th throw and the sum of all the numbers in 12 throws was 46. Which number
appeared least number of times?
(a) 6 (b) 4 (c) 2 (d) l

45. In a square ABCD, a point P is inside the square such that ABP is an equilateral triangle. The segment
AP cuts the diagonal BD in E. Suppose AE = 2. The area of ABCD

(a) 4 + 2 3 (b) 5 + 2 3 (c) 4 + 4 3 (d) 5 + 4 3

46. Let n be a positive integer not divisible by 6. Suppose n has 6 positive divisors. The number of
positive divisors of 9n is
(a) 54 (b) 36 (c) 18 (d) 12

a+x - a-x 2ab


47. The value of when x = is
a+x + a-x 2
b2 + 1
(a) a (b) b (c) x (d) 0

48. Two regular polygons of different number of sides are taken. In one of them, its sides are coloured red
and diagonals are coloured green; in the other, sides are coloured green and diagonals are coloured
red. Suppose there are 103 red lines and 80 green lines. The total number of sides the two polygons
together have is:
(a) 23 (b) 28 (c) 33 (d) 38

49. A box contains some red and some yellow balls. If one red ball is removed, one seventh of the
remaining balls would be red; if one yellow ball is removed, one-sixth of the remaining balls would be
red. If n denotes the total number of balls in the box, then the sum of the digits of n is
(a) 6 (b) 7 (c) 8 (d) 9

50. Let ABCD be a rectangle. Let X and Y be points respectively on AB and CD such that AX : XB = 1 : 2 =
CY : YD. Join AY and CX ; let BY intersect CX in K ; let DX intersect AY in L. If m/n denotes the ratio
of the area of XKYL to that of ABCD, then m + n equals
(a) 9 (b) 11 (c) 13 (d) 15

51. Let ABC be an equilateral triangle. The bisector of ÐBAC meets the circumcircle of ABC in D. Suppose
DB + DC = 4. The diameter of the circumcircle of ABC is

(a) 4 (b) 3 3 (c) 2 3 (d) 2

52. Let Tk denpte the k-th term of an arithmetic progression. Suppose there are positive integers m ¹ n
such that Tm = 1/n and Tn = 1/m. Then Tmn equals
1 1 1
(a) (b) + (c) 1 (d) 0
mn m n
8
53. In a triangle ABC, let AD be the median from A; let E be a point on AD such that AE : ED = 1 : 2 ; and
let BE extended meets AC in F. The ratio of AF/FC is
(a) 1/6 (b) 1/5 (c) l/4 (d) 1/3

54. If sinq and cosq are roots of the equation px2 + qx + r = 0, then:
(a) p2 – q2 + 2pr = 0 (b) (p – r)2 + q2 – r2
(c) p2 + q2 – 2pr = 0 (d) (p – r)2 = q2 + r2

55. For a regular k-sided polygon, let a(k) denotes its interior angle. Suppose n > 4 is such that a(n – 2),
a(n), a(n + 3) forms an arithmetic progression. The sum of digits of n is
(a) 2 (b) 3 (c) 4 (d) 5

56. The sum of 5 numbers in geometric progression is 24. The sum of their reciprocals is 6. The product
of the terms of the geometric progression is
(a) 36 (b) 32 (c) 24 (d) 18

57. Digits a and be are such that the product 4a1´ 25b is divisible by (in base 10). The number of ordered
pairs (a, b) is
(a) 15 (b) 8 (c) 6 (d) 4

58. The integer closest to 111...1 - 222...2 , where there are 2018 ones and 1009 twos, is

101009 - 1 101009 - 1 102018 - 1 102018 - 1


(a) (b) (c) (d)
3 9 3 9

59. In a triangle ABC, a point D on AB is such that AD : AB = 1 : 4 and DE is parallel to BC with E on AC.
Let M and N be the mid points of DE and BC respectively. What is the ratio of the area of the quadrilateral
BNMD to that of triangle ABC?
(a) 1/4 (b)9/32 (c) 7/32 (d) 15/32

é 10 2 ù é102 ù é102 ù é 10 2 ù
60. The number of distinct integers in the collection ê ú, ê ú,ê ú ,..... ê ú , where [x] denotes the
ë 1 û ë 2 û ë 3 û ë 20 û
largest not exceeding x, is
(a) 20 (b) 18 (c) 17 (d) 15

61. True coelom is not present in animals of:


(a) Platyhelminthes (b) Annelida
(c) Echinodermata (d) Arthropoda

62. The intracellular organelle that is responsible for formation of acrosomal vesicle is:
(a) Endoplasmic reticulum (b) Golgi apparatus
(c) Mitochondrion (d) None of the above

9
63. The genetically modified (GM) brinjal in India has been developed for:
(a) Enhancing shelf life (b) Insect-resistance
(c) Drought-resistance (d) Enhancing mineral content

64. A scientist observed few cells under a microscope with following characters:
i. Cells divided by binary fission or fragmentation, or budding
ii. Cells moved with the help of flagella
iii. Ether lipids were observed in cell membranes
iv. Peptidoglycans were noted in the cell walls
Which of the following category do the cells belong to?
(a) Archaea (b) Plant cells
(c) Unicellular eukaryotes (d) Cyanobacteria

65. Character(s) of acquired immunity is (are):


(a) Differentiation between self and non-self (c) Retains memory
(b) Specificity of antigen (d) All the above

66. Instead of using chemical fertilizers in a paddy field, a farmer thought of employing nitrogen fixation
technique. Amongst the following which would be beneficial for his cause?
(a) Glycine max - Rhizobium (b) Cycas-Nostoc
(c) Casuarina - Frankia (d) Azolla-Anabaena

67. An action potential in the nerve fibre is produced when positive and negative on outside and inside of
the axon membrane are reversed because:
(a) all potassium ions leave the axon
(b) more potassium ions enter the axon as compared to sodium ions leaving it
(c) more sodium ions enter the axon as compared to potassium ions leaving it
(d) all sodium ions enter the axon

68. A geneticist was studying the pathway of synthesis of an amino acid 'X' in an organism. The presence
(either synthesized de novo or externally added) of 'X' is a must for the survival of that organism. She
isolated several mutants that require 'X' to grow. She tested whether each mutant would grow when
different additives, P, Q, R, S and T were used.'+' indicates growth and '-' indicates the inability to
grow in the mutants tested. Find out the correct sequence of additives in the biosynthetic pathway of
'X'.

Organisms Additives
P Q R S T
Wild-type + + + + +
Mutant 1 – – – – +
Mutant 2 – + + + +
Mutant 3 – – + – +
Mutant 4 – + + +
(a) P ® Q ® R ® S ® T (b) P ® R ® S ® Q ® T
(c) T ® P ® Q ® S ® R (d) P ® S ® Q ® R ® T

10
69. In a case of mammalian coat color, the principal gene identified is 'C which codes for a tyrosinase
enzyme. In case of rabbits four different phenotypes are observed Full Color > Chinchilla > Himalayan > Albino
(in order of the expression of gene 'C and its alleles). In a progeny obtained after crossing two rabbits,
the percentages of Chinchilla, Himalayan and Albino rabbits were 50, 25 and 25 respectively. What
must have been the genotypes of the parent rabbits?
(a) CchCchX Cchc (b) CchCh X Cchc (c)Cchc X Ch c (d) ChCh X CchCch

70. It was observed in a group of tadpoles of a mutant frog reared in a laboratory that their development
was arrested at a particular stage. The exact tissue that was affected by the mutation is unknown. The
development was then resumed and accelerated by injecting the tadpoles with the extracts prepared
from various tissues of the wild type frogs. The observations of the experiment are given below.
Experiment No. Tissue Extract Observations
1 Anterior lobe of pituitary Development resumed
2 Posterior lobe of pituitary Development did not resume
3 Thyroid gland Development resumed
4 Anterior lobe of pituitary + Thyroid gland Development resumed
5 Anterior + posterior lobe of pituitary Development resumed
6 Posterior lobe of pituitary + Thyroid gland Development did not resume
From the above observations, find out the tissue that is affected by the mutation.
(a) Anterior lobe of pituitary (b) Posterior lobe of pituitary
(c) Thyroid gland (d) Both pituitary and thyroid gland

71. Identify the odd ones from each group (A and B) based on same criterion.

Group A Group B
Salmon Alpine salamander
Bullfrog Spiny anteater
Platypus Common toad
Bull shark Crocodile
(a) Platypus, Alpine Salamander (b) Bull shark, Alpine salamander
(c) Bullfrog, Crocodile (d) Platypus, Common toad

72. A patient was administered a chemical agent called Guanfacine hydrochloride after the patient showed
the symptoms like shortness of breath and headache. Guanfacine hydrochloride is a known stimulant
of central a2-adrenergic receptors of the medulla regulating the sympathetic nervous system. The
patient in this case must be suffering from______.
(a) Hypertension (b) Hyperstimulation
(c) Hyperpolarization (d) None of the above

73. A bacterial dsDNA molecule, 2988 bp in length, was found to have the following composition:
The respective values of X and Y are:
T C A G
Strand I 348 X 1400
Strand II 650 Y
(a) 1400 and 590 (b) 590 and 1400 (c) 590 and 590 (d) None of the above

11
74. What would be the length of a polypeptide translated from mRNA which is encoded by 2988 bp of a
bacterial gene?
(a) 989 (b)992 (c)995 (d)998

75. A student recorded the data for five types of cells as given below:

Character P P Q R S T
Cell wall + + – – +
Centrioles – – – + _
Chloroplast – + – – _
Mitochondrion – + + + +
Nucleus – + - + +
Plasma membrane + + - + +
RNA/DNA + + + + +
Vacuoles + + - + +
The five cell types P, Q, R, S and T are:
(a) P - Bacterium, Q - Plant, R- Virus, S - Animal, T - Fungus
(b) P - Bacterium, Q - Plant, R- Virus, S - Fungus, T - Animal
(c) P - Fungus, Q - Plant, R- Bacterium, S - Animal, T - Virus
(d) P - Plant, Q - Bacterium, R- Virus, S - Animal, T - Fungus

76. An environment conservation group performed a survey of some diverse locations in the country and
represented it as under:

Which amongst these sites should be included as a biodiversity hotspot?


(a) Site A (b)Site B (c)Site C (d)Site D

77. A bacterium has a generation time of 50 minutes. A culture containing 108 cells per mL is incubated
for 300 minutes. What will be the number of cells after 300 minutes?
(1) 64 × 103 cells (b) 6.4 × 108 cells (c) 64 × 109 cells (d) 6.4 × 109 cells

12
78. The blood grouping system is an example of 'multiple allelism. In order to find out the gene products
of various gene variants, different enzymes (codes used for the purpose of experimentation are X and
Y) from four blood samples were assayed. The enzymes were quantified and the information obtained
from these experiments is given in percentages in the following table. indicates presence of an enzyme
and indicates the absence of that enzyme from the blood sample. The standard codes for dominant
and recessive alleles are considered. Identify the blood groups of subjects and choose the correct
option of their genotypes from given options. (In table: + means present, – means absent)
Subjects Ramesh Ali Sophia Balwinder

Enzymes P/A % P/A % P/A % P/A %

X + 50 + 50 + 100 – –

Y – – + 50 – – + 100

(a) IAi, ii, IBi, IAIB (b) IAi, IAIB, IAIA, IBIB
(c) IBi, IAIB, ii, IBi (d) IBi, ii, IAIB, IAi

79. In an experiment, a scientist discovered a darkly stained chromatin body on the periphery of nucleus
of epithelial cells obtained from an eight year old boy. This is indicative of a particular syndrome. Find
out the best possible chromosome combination of their parents from the options given below; which
have the highest probability of producing the child under investigation. 'A' indicates autosome. 'X'
and 'Y' represent the sex chromosomes.
(a) 22AA+XY, 22AA+XXX (c) 22AA+XY, 22AA+XX
(b) 22AA+XXY, 22AA+XXX (d) 22AA+XXY, 22AA+XX

80. A millionaire Mr. Jim, died recently. Two women, Mary and Lou, claiming to have a child by Jim
approached the police demanding a share in his wealth. Fortunately Jim's semen sample was
cryopreserved. The scientists used DNA fingerprinting technique to study the three highly variable
chromosome regions. The results obtained are shown in the adjoining figure:
Marry's Lou's
Jim Mary child Lou child

After studying the DNA profile, which of the alleged heirs are children of Jim?
(a) Mary's child (b) both are children of Jim
(c) Lou's child (d) none are children of Jim

13
Q Q Q Q
Correct response Correct response Correct response Correct response
no no no no
PHY BIO MATHS CHEM CHEM PHY BIO MATHS MATHS CHEM PHY BIO BIO MATHS CHEM PHY
JS511 JS512 JS513 JS514 JS511 JS512 JS513 JS514 JS511 JS512 JS513 JS514 JS511 JS512 JS513 JS514
1 X a or d X b 21 d c a or d b 41 c a c a or d 61 a c b d
2 a a b b 22 b d a c 42 d d d a 62 b d a c
3 d b c a 23 c a b c or d 43 d d c d 63 b c or d c a
4 c d c or d c 24 a b d X 44 c or d b a d 64 a or d a c c
5 a d d X 25 c c d d 45 a c a d 65 d d X a
6 c c c or d c 26 d a d a 46 c or d c b c 66 d c or d b b
7 b d a d 27 X X c d 47 X X c a 67 c X d a
8 b c d a 28 b c a c or d 48 b b a b 68 d b a X
9 c a a a 29 b b c a 49 b b X c 69 c a a c
10 a c b a 30 b a a b 50 b b b a 70 a b a b
11 b b b a 31 a b c b 51 a a b c 71 b c a b
12 b a a d 32 a b b a 52 c a b b 72 a b d b
13 b c d b 33 a b a b 53 c a b a 73 c a b b
14 a b a a 34 a c c a 54 a a c a 74 c d b c
15 c d b b 35 a a b d 55 b a a c 75 a b a X
16 X a c a 36 c X d c 56 b a X a 76 b b a a
17 b a b b or c 37 d b a b 57 X b or c c b 77 d X b or c c
18 b b c b 38 a b b c 58 a b b b 78 b c b b
19 b b d a 39 c c a d 59 d a b b 79 a a a b
20 c a X c 40 b or c b b X 60 d c b d 80 a d c b
IMPORTANT INSTRUCTIONS
X-- means the question is deleted from all sets.
b or c -- means any one of the options will get credit. If a or d is marked then it’s a wrong choice.
a or d -- means any one of the options will get credit. If b or c is marked then it’s a wrong choice.
c or d -- means any one of the options will get credit. If a or b is marked then it’s a wrong choice.
Corrct answer -- award 3 marks
No answer -- award 0 (zero) marks
Wrong answer -- negative 1 mark (minus one mark).
NSEJS-2017 (IJSO STAGE-I)
Date of Examination : 19th November, 2017
PAPER CODE - JS531

SOLUTIONS
1. Rajiv, Nikhil, Shubha and Nilima wanted to establish a relationship between loss in weight of a solid
with weight of water displaced by immersing it in tap water and sea water. After performing their
experiment, they noted their observations for the same solid as follows :
Rajiv : Loss of weight of solid is more in tap water.
Nikhil : Loss of weight of solid is more in sea water.
Shubha : Loss of weight of solid is equal in the tap water and the sea water.
Nilima : Loss of weight of solid may be more in tap water or sea water, depending upon how deeply
it is immersed, identify the correct observation.
(a) Nikhil (b) Nilima
(c) Shubha (d) Rajiv
2. The ratio of atoms present in 4 g of magnesium and 4 g of sulphur is (Mg = 24 ; S = 32)
(a) 1 : 1 (b) 2 : 1
(c) 3 : 2 (d) 3 : 4
3. If Z = 10 the valency of the element is..............
(a) zero (b) one
(c) two (d) three
4. The average atomic mass of an element X is 80 u. The present of isotopes 79
X35 and 82
X35 in the
sample is
(a) 90.99 and 9.01 (b) 80.8 and 19.2
(c) 66.67 and 33.34 (d) 50 and 50
5. An aqueous solution used to preserve biological specimen is
(a) Methane (b) Methanol
(c) Methanal (d) Methanoic acid
6. The molecular formulae of some organic compounds are given below, which of these compounds
contains a Ketone group ?
(a) C3 H6 O 2 (b) C3 H6 O
(c) C3 H4 O (d) C3 H8 O
7. 'Duralumin' is an alloy of aluminium with
(a) iron, manganese and magnesium (b) copper, manganese and magnesium
(c) copper, chromium and magnesium (d) iron, nickel and magnesium
8. Tooth decay starts when the pH around tooth is around
(a) 7.5 (b) 7
(c) 6.5 (d) 5.5

1
9. What will happen if a copper piece is dipped in aqueous solution of silver nitrate for quite some time?
(i) Solution will remain colourless
(ii) Solution will turn blue
(iii) Silver will deposit on the copper piece
(iv) Bubbles of brown gas will be formed around copper piece
(a) i and iv (b) ii and iv
(c) ii and iii (d) iii and iv
10. Neeta mixed 10 mL of 0.1 M HCl solution with 15 mL of 0.067 M NaOH solution. She checked the
pH of the resulting solution using pH paper. The colour obtained was
Red Orange Yellow Green Pale blue Dark blue Violet
Strong acid ¬¾¾ Weak acid Neutral Weak alkali ¬¾¾ Strong alkali
(a) Green (b) Yellow
(c) Pale blue (d) Violet
11. (I) Zn + CuSO4(aq) ® Reaction occurs
(II) Zn + Al2(SO4)3 (aq) ® Reaction does not occur
(III) Zn + AgNO3 (aq) ® Reaction does not occur
(IV) Zn + PbNO3(aq) ® Reaction occurs
Which of the above statements is not correct ?
(a) I (b) II (c) III (d) IV
12. An open vessel contains air at 27°C. The vessel is heated till two-fifth of the air in it has been expelled.
Assuming the volume of the vessel remains constant, find the temperature to which the vessel has to
be heated ?
(a) 750 K (b) 700 K (c) 550 K (d) 500 K
13. A teacher wanted to give acid base titration to her students. For that she prepared (i) HCl solution by
dissolving 73 g of hydrochloric acid in one litre of water and (ii) sodium hydroxide solution by
dissolving 0.46 g of sodium metal in one litre of water. Find the volume of the hydrochloric acid
solution required for complete neutralisation of sodium hydroxide solution. (Cl = 35.5 ; Na = 23.0 ; O
= 16.0)
(a) 20 mL (b) 10 mL (c) 46 mL (d) 5 mL
14. What would be the atomic number of the next halogen element, if discovered in future ?
(a) 103 (b) 115 (c) 117 (d) 121
15. A white solid known to be a compound of sodium, given rise to water vapour and a colourless gas on
heating. The residual white powder is dissolved in water and when the solution is added to alum
solution, a white gelatinous precipitate is obtained. The original solid was :
(a) Sodium carbonate (b) Sodium bicarbonate
(c) Sodium hydroxide (d) Sodium nitrate
16. Harsha was trying to neutralize phosphoric acid using various bases. Those available were caustic
soda, lime water and hydrated alumina. If Harsha took 1 equivalent of phosphoric acid each time,
what will be the ratio for moles of each of the above bases required for complete neutralization ?
(a) 1 : 1 : 1 (b) 1 : 0.5 : 0.33
(c) 1 : 2 : 3 (d) 1 : 0.33 : 0.5

2
17. A flask containing SO2 gas was weighed at a particular temperature and pressure. The flask was then
flushed and filled with oxygen gas at the same temperature and pressure. The weight of SO2 in the
flask will be about :
(a) same as that of oxygen (b) one-fourth that of oxygen
(c) four times that of oxygen (d) twice that of oxygen
18. Arun needs 1.71 g of cane sugar (C12H22O11) to sweeten his tea. What would be the number of carbon
atoms consumed through sugar in the tea ?
(a) 3.66 × 1022 (b) 7.2 × 1021 (c) 5 × 1021 (d) 6.6 × 1022
19. Choose the correct sets which represent the oxides as Acidic : basic : neutral : amphoteric respectively
(i) CO2 : MgO : N2O : H2O (ii) SO2 : NO : CO : Al2O3
(iii) P2O5 : ZnO : NO : Al2O3 (iv) SO3 : CaO : N2O : PbO
(a) i and ii (b) ii and iii
(c) iii and iv (d) i and iv
20. During a meteorite shower a few meteorites fell into a water body having pH around 7. The pH of the
water body was measured after meteorite shower and found to be
(a) > 7
(b) < 7
(c) = 7
(d) no change in pH of water due to the meteorite shower
21. The position of two blocks at successive 0.20-second time intervals are represented by the numbered
squares in the figure below. The blocks are moving towards right.

Block a 1 2 3 4 5 6 7

Block b
1 2 3 4 5

The acceleration of the blocks are related as follows :


(a) acceleration of ‘a’ is greater than acceleration of ‘b’.
(b) acceleration of ‘a’ equals acceleration of ‘b’. Both accelerations are greater than zero.
(c) acceleration of ‘b’ is greater than acceleration of ‘a’
(d) acceleration of ‘a’ equals acceleration of ‘b’. Both acceleration are zero
22. In rural areas, an indigenous way of keeping kitchen materials cool is to put them in a box and wrap
the box with weiblanket; the blanket is kept wet as tap is allowed to drip in to its comer. Choose the
correct statement:
(a) This method works because the water from the tap is cold. If one uses room temperatufe water, it
will not work.
(b) Method will work only if the box is a bad Conductor of heat. If one uses tin box, it will not work.
(c) Method doesn't work
(d) method works because the latent heat necessary for evaporation of water in the blanket is taken
from the box so the box and its content remain cool'

3
23. In the adjacent circuit what is the current flowing from N to K?
30W 60W
A B J K
30W 10W
G H 60W L
D C N M
30W 60W

120 V
(a) 3A (b) 2A (c) 1A (d) 0.5 A
24. If x, v and t represent displacement (m), velocity (m/s) and time 9s) respectively for a certain particle
then which pair of the following figures can be best correlated to each other.
x x x x

t t t t
Fig.I Fig.II Fig.III Fig.IV

(a) I & II (b) I & III (c) I & IV (d) None


25. The take off speed of Airbus A340 is 288 km/hr. From the taxi track it comes to the main runway and
waits for a while for the final clearance from Air Traffic control. The aircraft then achieves this speed
within 50 seconds. Neglecting the effect of the wind direction and friction, what should be the minimum
length of main runway decided by civil engineeis for this aircraft for a take-off?
(a) 1800 m (b) 2000 m (c) 2200 m (d) 2400 m
26. An empty office chair is at rest on a floor. Consider the following forces :
I- A downward force of gravity,
II-An upward force exerted by the floor,
III-A net downward force exerted by the air.
Then, which of the force(s) is (are) acting on the office chair?
(a) I only
(b) I and II
(c) I, II and III
(d) None of the forces. (Since the chair is at rest there are no forces acting upon it.)
27. The ability of eye to focus both near and distant objects, by adjusting its focal length, is called
(a) Myopia (b) Presbyopia
(c) accommodation of eye (d) Tyndall effect
28. In bringing a a-particle, the electrostatic potential energy of the system _________.
(a) increases (b) decreases
(c) remains unchanged (d) becomes zero

4
29. A magnet is placed between two coils AB and CD as shown. It is being moved in the direction as
shown by the arrow, then which of the following statement is correct.
(a) looking from end A, current in coil AB will be anticlockwise and looking from end D, the direction
of current in coil CD will be anticlockwise. A B C D
(b) looking from end A, current in coil AB will N S
be clockwise and looking frojm end D, the
direction of current in coil CD will be
G G
clockwise
(c) looking from end A, curent in coil AB will be clockwise and looking from end D, then direction of
current in coil CD will be anticlockwise
(d) looking from end A, current in coil AB will be anticlockwise and looking from end D, the direction
of current in coil CD will be clockwise
30. A boy throws a steel ball straight up. Consider the motion of the ball only after it has left the boy's
hand but before it touches the ground and assume that forces exerted by the air are negligible. For
these conditions, the force(s) acting on the ball is (are) :
(a) a downward force of gravity along with a steadily decreasing upward force.
(b) a steadily decreasing upward force from the moment it leaves the boy's hand until it reaches its
height point; on the way down there is a steadily increasing downward force of gravity as the
object gets closer to the earth.
(c) constant downward force of gravity along with un upward force that steadily decreases until the
ball reaches its highest point; on the way down there is only a constant downward force of gravity.
(d) constant downward force of gravity only.
31. A large truck collides heat-on with a small compact car. During the collision :
(a) the truck exerts a greater force on the car than the car exertes on the truck.
(b) the car exerts a greater force on the truck than the truck exerts on the car.
(c) the truck exerts a force on the car but the car does not exert a force on the truck.
(d) the truck exerts the same force on the car as the car exerts on the truck.
32. A common hydrometer has a uniform scale and its stem is graduated downwards from 0 to 20. While
floating in water, it read 0 and while floating in a liquid of density 1.40 g/cm3, it reads 20. Then the
density of the liquid in which it will reacd 10 is
(a) 0.7 g/cm3 (b) 0.85 g/cm3
(c) 1.17 g/cm3 (d) 2.8 g/cm3
33. For the same angle of incidence, the angle of refraction in three different media A, B, C are 15°, 25°
and 35° respectively. Then which statement is correct : (µA is refractive index of A)
(a) µA is maximum and velocity of light is maximum in medium A.
(b) µA is minimum and velocity of light is maximum in medium A.
(c) µA is maximum and velocity of light is minimum in medium A.
(d) µA is minimum and velocity of light is minimum in medium A.

5
34. A liquid, whose density doesn't change during the motion, is flowing steadily through a pipe of
varying cross sectional area as shown in the adjacent figure. If a1, a2 are the cross sectional areas, v1 ,
v2 are the values of velocities (or speeds) at L and H respectively, then the correct relation between a1,
a2 and v1, v2 is :
H

(a) a1v2 = a2v1 (b) a1v1 = a2v2 (c) a12v2 = a23v1 (d) a1 v12 = a2v22
35. As shown is adjacent figure two plane mirros. M1 and M2 are inclined to each other at the angle 70°
(angle M1OM2). Incident ray AB makes an algne of incidence q on M1. This ray after relfection at B
on M1 and further at C on M2 travesl along the direction CD, such that path CD is parallel to M1. Then
angle q is _______.
M2

C D
E A

M1
O B
(a) 45° (b) 50° (c) 55º (d) 60°
36. A copper disc of radius a0 has a hole of radius b0 at the centre, at 0°C. The disce is now heated and
mainteained at 200°C. The new radii of disc and hole are a1 and b1 respectively. for the heated disc it
can be concluded that.

a0

b0

(a) a0 < a1, b0 < b1and density of disc decreases (b) a0 < a1, b0 > b1and density of disc decreases
(c) a0 < a1, b0 < b1and density of disc increases (d) a0 < a1, b0 > b1and density of disc increases
37. A concave mirror of radius of curvature 1m is placed at the bottom of a water tahnk. The mirror forms
an image of the sun when it is directly overhead. If the depth of water in the tank is 80 cm, then the
distance of the image formed is _______(refractive index of water is 1.33)
(a) on surface of water (b) 110 cm above mirror
(c) 50 cm above mirror (d) image cannot be formed
38. The equivalent resistance of two resistances in series is 'S'. These resistance are now joined in parallel.
The parallel equivalent resistance is 'P'. If S = n P. Then the minimumpossible value of n is
(a) 2 (b) 3 (c) 4 (d) 5

6
39. An electron a a-particle enter a regionof uniform magnetic field (of induction B) with equal velocities.
The direction of B is perpendicualr and into the plane of the paper. The qualitatively identify the
direction of paths of electron and the a-particle.
II
I
III

(a) I for a- particle, II for electron (b) I for electron, II for a- particle
(c) I for a- particle, III for electron (d) I for electron, III for a- particle
40. Two wave pulses I and II and the same wavelenth. They are travelling in the directions as shown by
the single headed arrows. The resultant sketch of the two wave pulses at some instant of time when P
coincides with R is _________.

I
a

2a

II

a
(a) (b) a

a
a
(c) (d)

41. Ravi mixed two substances A and B in a vessel and left it as it is. After few hours he detected an
alcoholic smell emanating from the vessel. Identify what A and B are:
(a) Salt solution and Lactobacillus (b) Fruit juice and Saccharomyces
(c) Fruit juice and Lactobacillus (d) Salt solution and Saccharomyces
42. Which amongst the following shows the characters of both plants and animals:
(i) Anabaena (ii) Paramecium (iii) Euglena (iv) Amoeba
(a) i and iv (b) iii (c) ii (d) i and iii
43. Which amongst the following are not plastids:
(a) Leucoplasts (b) Chromoplasts (c) Amyloplasts (d) Tonoplasts
44. During a study the number of cells was recorded to increase as follows:
64 ® 128 ® 256 ® 512 ® 1024. This represents:
(a) Budding (b) Meiosis (c) Binary fission (d) Fragmentation

7
45. A plant kept in a box with only a hole for entry of light shows bending, the process called phototropism.
It occurs due to:
(a) Synthesis and diffusion of cytokinin in the leaves
(b) Breakdown of auxin in the shoot
(c) Synthesis and diffusion of abscisic acid
(d) Synthesis and diffusion of auxin in the shoot
46. What would be the minimum required length of codon to encode 400 amino acids, if there existed
three purines and pyrimidines each?
(a) 3 (b) 4 (c) 5 (d) 6
47. A 'life- style' disorder among these is:
(a) Hypertension (b) Presbyopia (c) Herpes (d) Scurvy
48. Metamerism is a characteristic of
(a) Hirudinaria (b) Taenia (c) Asterias (d) Pila
49. Health is all about 'eating-fasting' balance. When you fast for extended periods, your cells clean out
and recycle the intracellular garbage. The organelles responsible for this are:
(a) Microtubules (b) Microfilaments
(c) Golgi Apparatus (d) Lysosomes
50. A plant may not exchange CO2 or O2 with air at:
(a) twilight (b) mid-night
(c) late hours in the morning (d) noon
51. If a small part of the esophagus of a person is excised, the consequence would be the person will have
to eat _________.
(a) larger portion of food with large time interval
(b) small portions of food at small time interval
(c) small portions of food at large time interval
(d) majorly subsist on liquid diet
52. When heated, the hydrogen bonds between the complementary strands of DNA break and the 2
strands separate in a process called melting. Which of the following pieces of DNA will require maximum
temperature for melting?
(a) 3' AAGGTATACAAT 5' (b) 3' GAGCUAUCCGAG5'
5' TTCCATATGTTA 3' 5'CUCGAUAGGCUC 3'
(c) 3' ACGTCCGCTGCG 5' (d) 3' ATTAGCTAGCAA 5'
5' TGCAGGCGACGC 3' 5' TAATCGATCGTT 3'
53. In a self-pollinated plant, what would be minimum number of meiotic divisions required for setting
400 seeds?
(a) 100 (b) 200 (c) 400 (d) 500
54. If a flower is producing a large number of minute and smooth pollen, the agency for cross pollination
is most likely to be:
(a) Air (b) Water (c) Insects (d) Bats
8
55. To meet the increasing demand for food, there have been several 'revolutions'. Which of the following
revolutions is likely to have contributed most to global warming?
(a) Green (b) White (c) Blue (d) Silver
56. A mammal adapted to desert conditions is likely to have large:
(a) Nostrils (b) Pinnae (c) Muzzle (d) Nails
57. Which of the following feature indicates omnivorous feeding of human species?
(a) Presence of canines as well as premolars and molars
(b) Presence of appendix
(c) Presence of 11th and 12th pair of ribs
(d) Presence of opposable thumb
58. In a dihybrid cross, what is the proportion of organisms with dihybrid geonotype?
(a) 2/16 (b) 6/16 (c) 4/16 (d) 9/16
59. If the cell is using less oxygen molecules than the molecules of carbon dioxide evolved in respiration,
the substrate for respiration has to be:
(a) simple sugars (b) organic acids
(c) fatty acids (d) cholesterol
60. Panting is a means of thermoregulation in dogs. This is due to:
(a) high specific heat of water (b) high vapour pressure of water
(c) high latent heat of vapourization (d) high specific gravity of water
61. How many four digit numbers are there such that when they are divided by 101, they have 99 as
remainder ?
(a) 90 (b) 98 (c) 100 (d) 101

62. If x = ( )
21 – 20 and y = ( )
18 – 17 , then

(a) x = y (b) x < y (c) x > y (d) x + y = 0


63. What is the sum of all odd numbers between 500 and 600 ?
(a) 26000 (b) 27000 (c) 27500 (d) 29500

1 1 1 1 1 1
64. 1 + 1 + 1 + 1 + 1 + ....................+ 1 = ___________.
2 6 12 20 30 380
(a) 20.25 (b) 20.05 (c) 19.95 (d) 19.85
65. A train is running at a speed of 54 km/hr. It is not stopping at a certain station. It crosses the person
showing green flag in 20 seconds and crosses the platform in 36 seconds. What is the length of the
train ?
(a) 240 m (b) 300 m (c) 320 m (d) 360 m
66. In triangle ABC, segment AD, segment BE and segment CF are altitudes. If AB × AC = 172.8 cm2 and
BE × CF = 108.3 cm2 then AD × BC = ________.
(a) 136.8 cm2 (b) 132.4 cm2 (c) 129.2 cm2 (d) 128.6 cm2
67. Diagonals of a quadrilateral bisect each other. Therefore the quadrilateral must be a ______.
(a) parallelogram (b) rhombus (c) rectangle (d) square

9
68. If (a + b + c + d) = 4, then

1 1 1 1
+ + + = ____________.
(1 - a)(1 - b)(1 - c) (1 - b)(1 - c)(1 - d) (1 - c)(1 - d)(1 - a) (1 - d)(1 - a)(1 - b)

(a) 0 (b) 0.25 (c) 1 (d) 4


69. The sum of two numbers is 13 and the sum of their cubes is 1066. Find the product of those two
numbers.
(a) 26 (b) 27 (c) 28 (d) 29
70. By which smallest number we should divide 198396198 to get a perfect square ?
(a) 14 (b) 18 (c) 22 (d) 28
71. What will be the remainder if the number (7)2017 is divided by 25?
(a) 1 (b) 7 (c) 18 (d) 24
72. If ABCD is a cyclic quadrilatteral AB = 204, BC = 104, CD = 195, DA = 85 and
BD = 221, then AC = ________.
(a) 210 (b) 220 (c) 225 (d) 240
73. If x + xy + xz = 135, y + yz + xy = 351 and z + xz + yz = 243, then x + y2 + z2 = _________.
2 2 2 2

(a) 225 (b) 250 (c) 275 (d) 300


74. What is the radius of the circumcircle of a triangle whose sides are 30 cm, 36 cm and 30 cm
(a) 15 cm (b) 16 cm (c) 17 cm (d) 18 cm
75. On seventy first 'Independence day' there are Tuesday. After how many years there will be Tuesday on
'Independence day' ?
(a) 4 yrs. (b) 5 yrs (c) 6 yrs. (d) 7 yrs.
76. If p + q + r = 2, p2 + q2 + r2 = 30 and pqr = 10, the value of (1 – p) (1 – q) (1 – r) will be ________.
(a) –18 (b) –24 (c) –27 (d) –35
77. The mean of the following frequency distribution is ___________.

Class interval 0 - 10 10 -20 20 -30 30 - 40 40 - 50

Frequency 4 6 8 10 12

(a) 25 (b) 28 (c) 30 (d) 32


x - bx m - 1
2
78. If the roots of the equation = are equal and of opposite signs, then the value of 'm' is
ax - c m + 1
_______.

a-b a+b ab a+b


(a) (b) (c) (d)
a+b a-b a+b ab

æ 1ö æ 3 1 ö æ 2 1 ö æ 1ö
79. If ç x + ÷ = 5, then ç x + 3 ÷ – 5 ç x + 2 ÷ + ç x + ÷ = _________.
è x ø è x ø è x ø è xø

(a) 0 (b) 5 (c) –5 (d) 10


80. If x2 – 3x + 2 is a factor of x4 – px2 + q, then p, q are
(a) 2,3 (b) 4, 5 (c) 5,4 (d) 0, 0

10
Answer keys for NSEJS- 2017. Please send your suggestions to the answer keys
by filling the Google forms using the link given below. The last date for
suggestions will be December 4th 2017. The corrections to final answer keys if
any will be displayed on December 10th 2017. Google forms link:
https://goo.gl/forms/MdPEJbPiCUaxdQRw1
QP CODE JS531 QP CODE JS532 QP CODE JS533 QP CODE JS534
Q. No. Answer Q. No. Answer Q. No. Answer Q. No. Answer
1 a 1 a 1 b 1 a
2 deleted 2 c 2 c 2 b
3 a 3 b 3 c 3 d
4 c 4 d 4 d 4 a
5 c 5 d 5 d 5 c
6 b 6 b 6 d 6 d
7 b 7 c 7 b 7 a
8 d 8 c 8 b 8 b
9 b 9 c 9 d 9 b
10 a 10 a 10 a 10 d
11 c 11 c 11 b 11 a
12 d 12 c 12 a 12 a
13 b 13 a 13 c 13 b
14 c 14 a 14 d 14 a
15 b 15 c 15 d 15 d
16 b 16 b 16 b 16 d
17 d 17 d 17 b 17 c
18 a 18 c 18 c 18 b
19 d 19 d 19 b 19 c
20 a 20 d 20 c 20 d
21 d 21 deleted 21 a 21 d
22 d 22 c 22 c 22 c
23 c 23 b 23 c 23 c
24 b 24 a 24 c 24 d
25 b 25 d 25 b 25 b
26 b 26 d 26 b 26 a
27 c 27 b 27 c 27 a
28 a 28 b 28 b 28 c
29 a 29 a 29 b 29 c
30 d 30 a 30 d 30 b
31 d 31 d 31 c 31 b
32 c 32 a 32 d 32 a
33 c 33 d 33 c 33 b
34 b 34 b 34 c 34 a
35 b 35 b 35 c 35 b
36 a 36 c 36 a 36 d
37 c 37 b 37 d 37 a
38 c 38 b 38 b 38 c
39 c 39 d 39 c 39 d
40 a 40 d 40 a 40 b
41 b 41 d 41 b 41 d
42 b 42 b 42 c 42 c
43 d 43 b 43 c 43 b
44 c 44 d 44 d 44 d
45 d 45 c 45 b 45 c
46 b 46 d 46 b 46 c
47 a 47 d 47 d 47 a
48 a 48 b 48 deleted 48 a
49 d 49 b 49 c 49 b
50 a 50 c 50 d 50 d
51 b 51 d 51 c 51 a
52 c 52 b 52 c 52 c
53 d 53 b 53 c 53 c
54 a 54 b 54 a 54 c
55 b 55 d 55 a 55 b
56 b 56 c 56 c 56 d
57 a 57 b 57 b 57 a
58 c 58 a 58 d 58 b
59 b 59 c 59 b 59 b
60 c 60 a 60 a 60 a
61 a 61 d 61 d 61 a
62 b 62 c 62 b 62 c
63 c 63 b 63 c 63 b
64 c 64 c 64 d 64 c
65 b 65 a 65 b 65 a
66 a 66 c 66 a 66 a
67 a 67 a 67 d 67 d
68 a 68 b 68 d 68 d
69 d 69 b 69 c 69 d
70 c 70 c 70 a 70 deleted
71 b 71 b 71 d 71 d
72 b 72 d 72 d 72 d
73 c 73 d 73 c 73 d
74 d 74 a 74 b 74 b
75 c 75 d 75 c 75 c
76 b 76 b 76 b 76 b
77 c 77 d 77 a 77 b
78 a 78 d 78 b 78 c
79 a 79 a 79 b 79 a
80 c 80 d 80 b 80 a
Thanks for the suggestions to the answer keys. The expert committee
has gone through all the suggestions and recommended the following
corrections to answer keys for NSEJS- 2017. No more queries and
suggestions will be accepted. The list of students above MAS will be
announced on December 22nd, 2017.

SET JS531 SET JS532 SET JS533 SET JS534


Q. No. Answer Q. No. Answer Q. No. Answer Q. No. Answer
1 a 1 a 1 b 1 a
2 deleted 2 c 2 c 2 b
3 a 3 b 3 c 3 d
4 c 4 d 4 d 4 a
5 c 5 d 5 d 5 c
6 b 6 b 6 d 6 d
7 b 7 c 7 b 7 a
8 d 8 c 8 b 8 b
9 c 9 c 9 d 9 b
10 a 10 a 10 a 10 d
11 c 11 c 11 b 11 a
12 d 12 c 12 a 12 a
13 b 13 a 13 c 13 b
14 c 14 a 14 d 14 a
15 b 15 c 15 d 15 d
16 b 16 b 16 b 16 d
17 d 17 deleted 17 b 17 c
18 a 18 c 18 c 18 b
19 d 19 d 19 b 19 c
20 a 20 d 20 c 20 d
21 d 21 deleted 21 a 21 d
22 d 22 c 22 c 22 c
23 c 23 b 23 c 23 c
24 b 24 a 24 c 24 d
25 b 25 d 25 b 25 b
26 b 26 d 26 b 26 a
27 c 27 b 27 c 27 a
28 a 28 b 28 b 28 c
29 a 29 a 29 b 29 c
30 d 30 a 30 d 30 b
31 d 31 d 31 c 31 b
32 c 32 a 32 deleted 32 a
33 c 33 d 33 c 33 b
34 b 34 b 34 c 34 a
35 b 35 c 35 c 35 b
36 a 36 c 36 a 36 d
37 c 37 b 37 d 37 a
38 c 38 b 38 b 38 c
39 c 39 d 39 c 39 d
40 a 40 d 40 a 40 b
41 b 41 d 41 b 41 d
42 b 42 b 42 c 42 c
43 d 43 b 43 c 43 b
44 c 44 d 44 d 44 deleted
45 d 45 c 45 b 45 c
46 b 46 d 46 b 46 c
47 a 47 d 47 d 47 a
48 a 48 b 48 deleted 48 a
49 d 49 b 49 c 49 b
50 a 50 c 50 d 50 d
51 b 51 d 51 c 51 a
52 c 52 b 52 c 52 c
53 d 53 b 53 c 53 c
54 a 54 b 54 a 54 c
55 b 55 d 55 a 55 b
56 b 56 c 56 c 56 d
57 a 57 b 57 c 57 a
58 c 58 a 58 d 58 b
59 b 59 c 59 b 59 b
60 c 60 a 60 a 60 a
61 a 61 d 61 d 61 a
62 b 62 c 62 b 62 c
63 c 63 b 63 c 63 b
64 c 64 c 64 d 64 c
65 b 65 a 65 b 65 a
66 a 66 c 66 c 66 a
67 a 67 a 67 d 67 d
68 a 68 b 68 d 68 d
69 d 69 b 69 c 69 d
70 c 70 c 70 a 70 deleted
71 b 71 b 71 d 71 d
72 b 72 d 72 d 72 d
73 c 73 d 73 c 73 d
74 deleted 74 a 74 b 74 b
75 c 75 d 75 c 75 c
76 b 76 b 76 b 76 c
77 c 77 d 77 a 77 b
78 a 78 d 78 b 78 c
79 a 79 a 79 b 79 a
80 c 80 d 80 b 80 a
JS530

INDIAN ASSOCIATION OF PHYSICS TEACHERS

NATIONAL STANDARD EXAMINATION IN JUNIOR SCIENCE 2016 -17


Date of Examination: 20TH November, 2016

Time: 1000 to 1200 Hrs

Q. Paper Code: JS530

Write the question paper code mentioned above on YOUR answer sheet (in
the space provided), otherwise your answer sheet will NOT be assessed. Note
that the same Q. P. Code appears on each page of the question paper.
Instructions to Candidates –

1. Use of mobile phones, smartphones, ipads during examination is STRICTLY PROHIBITED.


2. In addition to this question paper, you are given answer sheet along with Candidate’s copy.
3. On the answer sheet, make all the entries carefully in the space provided ONLY in BLOCK
CAPITALS as well as by properly darkening the appropriate bubbles.
Incomplete/ incorrect/carelessly filled information may disqualify your candidature.
4. On the answer sheet, use only BLUE or BLACK BALL POINT PEN for making entries and filling
the bubbles.
5. The email ID and date of birth entered in the answer sheet will be your login credentials for
accessing performance report. Please take care while entering.
6. Question paper has 80 multiple choice questions. Each question has four alternatives, out of
which only one is correct. Choose the correct alternative and fill the appropriate bubble, as
shown.

7. A correct answer carries 3 marks whereas 1 mark will be deducted for each wrong answer.
8. Any rough work should be done only in the space provided.
9. Use of non-programmable calculator is allowed.
10. No candidate should leave the examination hall before the completion of the examination.
11. After submitting your answer paper, take away the Candidate’s copy for your reference.

Please DO NOT make any mark other than filling the appropriate bubbles properly in the space
provided on the answer sheet.

Answer sheets are evaluated using machine, hence CHANGE OF ENTRY IS NOT ALLOWED.

Scratching or overwriting may result in a wrong score.

DO NOT WRITE ON THE BACK SIDE OF THE ANSWER SHEET.

1
JS530

Instructions to Candidates (continued) –

Read the following instructions after submitting the answer sheet.

12. Comments regarding this question paper, if any, may be sent by email only to
iapt.nse@gmail.com till 29th November, 2016.
13. The answers/solutions to this question paper will be available on our website –
www.iapt.org.in by 2nd December, 2016.
14. CERTIFICATES and AWARDS –
Following certificates are awarded by the IAPT to students successful in NSEs
(i)Certificates to “Centre Top 10%” students

(ii)Merit Certificates to “Statewise Top 1%” students

(iii)Merit Certificates and a book prize to “National Top 1%” students

15. Result sheets can be downloaded from our website in the month of February. The “Centre
Top 10%” certificates will be dispatched to the Prof-in-charge of the centre by February,
2017.
16. List of students (with centre number and roll number only) having score above MAS will be
displayed on our website (www.iapt.org.in) by 22nd December, 2016. See the Eligibility
Clause in the Student’s brochure on our website.
17. Students eligible for the INO Examination on the basis of selection criteria mentioned in
Student’s brochure will be informed accordingly.

2
JS530

Indian Association of Physics Teachers


NATIONAL STANDARD EXAMINATION IN JUNIOR SCIENCE 2016-2017
Total time: 120 minutes Marks: 240

Only one out of four options is correct


1) Two bodies A and B are charged with equal magnitude of charge but A with positive charge and B with
negative. If MA and MB are masses before charging and and are the masses after charging, then (m0 is
some constant mass)

a) and b) and
c) d) and

2)
The number of natural numbers for which is a natural number is

a) 30 b) Zero c) 6 d) 5

3) Elements A, B and C have atomic numbers X, X+1 and X+2, respectively. ‘C’ is an alkali metal. ‘A’ reacts with
another element ‘Y’ to form the compound ‘AY’. ‘A’ and ‘Y’ belong to the same group. ‘AY’ possesses an

a) ionic bond b) covalent bond c) metallic bonding d) coordinate bond

4) Air has three major components: nitrogen, oxygen, and argon. Given that one mole of air at sea level is made
up of 78% nitrogen, 21% oxygen, and 1% argon, by volume. What is the density of air? Assume that gases
behave in an ideal manner. (Atomic mass of argon is 40 amu)

a) 14.62 g/L
b) 1.3 g/L
c) 29 g/L
d) 0.65 g/L

5) A conductor of length L has a varying cross section with area 2A at P P


and A at Q as shown in the adjacent figure. If it carries a steady Q
current I, then

a) Net charge per unit volume near P is more than net charge
L
per unit volume near Q.
b) Net charge per unit volume near Q is less than net charge per unit volume.
c) Current per unit area near P is more than current per unit area near Q.
d) Current per unit area near P is less than current per unit area near Q.

3
JS530

6)
The number of natural numbers for which is a prime number is

a) Three b) Zero c) Nine d) Two

7) Rhodoferax fermentans is a species of photosynthetic bacteria. From your knowledge about bacteria in general,
identify the components that CANNOT be present in this organism.

a) Chloroplasts
b) ATP
c) Ribosomes
d) Cell wall

8) If atmospheric humidity decreases, transpiration rate

a) Decreases because the concentration gradient between the mesophyll and the atmosphere decreases.
b) Stays the same because active transport does not depend on humidity
c) increases because of the higher concentration gradient between the air spaces of the mesophyll and the
atmosphere
d) Decreases because the concentration of water vapour decreases

9) Vessels A and B are made of conducting material. Both contain water. Vessel A floats in B. Vessel B is now
heated at a uniform rate, then

a) Water in A boils first.


b) Water in A boils some time after water in B starts boiling.
c) Water in both A and B start boiling simultaneously.
d) Water in A does not boil.

10) The number of squares formed by 5 vertical and 4 horizontal lines (all are equispaced) is

a) 60 b) 20 c) 40 d) 46

11) If 0.50 mole of a monovalent metal (M+1) halide is mixed with 0.2 mole of a divalent metal (L+2) phosphate, the
maximum number of moles of M3PO4 that can be formed is

a) 0.25 b) 0.30 c) 0.16 d) 0.20

12) Every major city in India has a pollution control board to monitor air and water pollution. The following data is
from three different localities in Bangalore city from the year 2015.

Locality Annual average of SO2 in the air


(volume/volume)

P 16.3 mL/m3

Y 16.3 ppb (m3/m3)

Z 16.3 ppm (m3/m3)

ppb stands for parts per billion and ppm stands for parts per million. These are different units to express

4
JS530

concentration. They are very similar to percentage (which is actually parts per hundred).

Based on the above data, which place will you choose to live in?

a) All localities are equally polluted, so I have no preference.


b) P is the more polluted than Y and Z, hence I will live in either Y or Z.
c) Locality Y is least polluted, hence I will live in Y.
d) Z and Y are more polluted than P, hence I will live in P.

13) A body thrown vertically up reaches a maximum height and returns back. Its acceleration is

a) Downward during both ascent and descent.


b) Downward at all positions except at the highest point, where it is zero.
c) Upward during both ascent and descent.
d) Downward during ascent and upward during descent.

14) The number of integers for which is

a) 2 b) Infinite c) 0 d) 4

15) Which of the following is NOT produced by the endoplasmic reticulum?

a) Lipids
b) Proteins
c) Monosaccharides
d) Hormones

16) Vaccines prevent infections by pathogens by:

a) Presenting the body's immune system with antigens in a controlled manner, so that it is prepared to
counter the pathogen producing it when it attempts to infect the body
b) Affecting the reproductive cycle of the invading pathogen
c) Binding to antigens on the surface of the pathogen and inactivating it
d) Affecting the metabolic pathways of the pathogen

17) Velocity of a particle moving along a straight line varies with time as shown
in the adjacent figure. Net forces acting on the particle are , , ,
and in the intervals OA, AB, BC, CD and DE respectively. Indentify the
correct statement

a) increases with time.


b) is initially positive and then becomes negative
c) and are in opposite directions
d) is negative.

18) If set X consists of three elements then the number of elements in the power set of power set of X is

a) b) c) d)

5
JS530

19) The heat of neutralisation of CH3COOH, HCOOH, HCN and H2S are . 2, 56.07, 2. and 3.34 kJ per
equivalent respectively. The increasing order of strength of these acids is.

a) HCOOH ˂ CH3COOH ˂ H2S ˂ HCN

b) H2S ˂ HCN ˂ HCOOH ˂ CH3COOH

c) HCN ˂ H2S ˂ CH3COOH ˂ HCOOH

d) CH3COOH ˂ HCOOH ˂ HCN ˂ H2S

20) To prevent the formation of oxides, peroxides, and superoxides, alkali metals are sometimes stored in an
unreactive atmosphere. Which of the following gases should not be used for lithium:

a) Ne
b) Ar
c) N2
d) Kr

21) A wooden block (W) is suspended by using a cord from a heavy steel ball (B). The entire system is dropped from
a height. Neglecting air resistance, the tension in the cord is

a) Zero b) The differences in the masses of B and W

c) The differences in the weights of B and W d) The weight of B

22) In a sided regular polygon, the radius of the circum-circle is equal in length to the shortest diagonal. The
number of values of for which this can happen is

a) 0 b) 1 c) 10 d) 2

23) Which of the following does NOT contain living cells?

a) Bone tissue
b) Xylem sieve tubes
c) Phloem
d) Epidermis

24) If DNA was made of 6 nucleotides instead of 4, what are the total number of triplet codons possible?

a) 24
b) 18
c) 64
d) 216

6
JS530

25) A circus performer of weight is standing on a wire as


shown in the adjacent figure. The tension in the wire is

a) Approximately
b) Approximately
c) Much more than
d) Much less than

26) Number of integers such that the number is a divisor of the number is

a) 0 b) 1 c) 4 d) 2

27) When 1 gram of a mixture of aluminium and zinc was treated with HCl, a gas was liberated. At the end of the
reaction, the volume of the liberated gas was found to be 524 cm3, under STP conditions. The individual
weights of aluminium and zinc in the mixture, respectively, are:

a) 0.2g and 0.8g b) 0.8g and 0.2g c) 0.5g and 0.5g d) 0.322g and 0.678g

28) Choose that element which is most different from the other three:

a) Carbon
b) Nitrogen
c) Silicon
d) Phosphorous

29) In the following diagrams O is point object and I is its image formed by a concave mirror. Indentify the diagram
in which position of image I is nearly correct.

7
JS530

30) If for we have then the minimum value of is

a) 2 b) 1 c) 4 d)

31) Which of these is a mollusc?

a) Lobster
b) Scorpion
c) Crab
d) Octopus

32) What is the mechanism used by the kidneys to remove waste products from the body?

a) Nephrons convert nitrogenous waste to uric acid and pass it out as urine
b) Nephrons actively transport uric acid and other nitrogenous waste  into the proximal and distal
convoluted tubules, from where it is collected to form urine.
c) The blood is filtered to retain cells and large plasma proteins within the blood. The remaining filtrate
passes through the proximal and distal convoluted tubules, where needed substances are reabsorbed.by
active transport
d) Nephrons filter out the nitrogenous waste which is passed through the proximal and distal convoluted
tubules and collected by the collecting duct as urine.

33) Following diagram shows refraction of parallel beam of light through a spherical surface. Indentify the correct
ray diagram

a) b)

R D
c) a e d)

r
34) Tenth term in the sequence
n is

a) 336 b) 63 c) 216 d) 68
e s
r e
r
8
JS530

35) An electron pair donor is a Lewis base and an electron pair acceptor is a Lewis acid. Which among the following
statements, is correct?

a) NH3 is a Lewis acid, because nitrogen has only 6 electrons around it.

b) BF3 is a Lewis base, because fluorine has 8 electrons around it.

c) NF3 is a Lewis base, because nitrogen has a lone electron.

d) BCl3 is a Lewis acid because it has only 6 electrons around it.

36) Greenhouse gases absorb (and trap) outgoing infared radiation (heat) from Earth and contribute to global
warming. A molecule that acts as a greenhouse gas, generally has a permanent dipole moment and sometimes
for other reasons. Going only by the condition of permanent dipole moment, in the list of gases given below,
how many can be potential greenhouse gases?

Water, Sulphur dioxide, Boron trifluoride, carbon monoxide, carbon dioxide, nitrogen, oxygen, methane,
hydrogen sulphide, ammonia.

a) Five
b) Six
c) Seven
d) Four

37) In the diagram M1 and M2 are two plane mirrors at


right angles to each other. O is a luminous point 2 cm M
1
object. Consider two images formed due to first
reflection at M1 and M2. The area of the triangle
O
formed by the object and two images is
2 cm
a) 4 cm2 b) 2 cm2
c) 8 cm2 d) 16 cm2

M2
38) The probability of a point within an equilateral triangle with side 1-unit lying outside its in-circle (inscribed
circle) is

a) b) c) d)

39) Penicillin cannot be used to treat influenza because:

a) It only helps to bring the temperature down, and does not reduce the infection
b) The penicillin is broken down by the organism
c) Viruses do not have cell walls
d) Reproduction of protozoans is not affected by penicillin

9
JS530

40) Thin cuboidal strips are made by slicing a potato. They are all made to be exactly 8 cm long and 2 mm wide.
Each strip is placed in sugar solutions of different concentration. After soaking it for 5 hours, their lengths are
measured again. The following graph shows the results of the experiment. What concentration of sugar
solution is isotonic with the conntents of the cells of the potato.

8.5

8.4

8.3

8.2
Length (cm)

8.1

7.9

7.8

7.7

7.6 Concentration (mol/liter)


0 0.1 0.2 0.3 0.4 0.5 0.6 0.7 0.8

a) 0.2 b) 0.4 c) 0.6 d) 0.1

41) A fisherman of height h is standing on the bank of a lake. A fish in the water perceives his height as h. Then

a) h > h b) h < h
c)h = h d) h > h or h < h depending on position of fish

42) A triangle has perimeter 316 and its sides are of integer length. The maximum possible area for such a triangle
is achieved for

a) Single triangle b) Two triangles

c) Three triangles d) More than three triangle

43) Hennig Brand, one of the many alchemists was in pursuit of the philosopher’s stone. Brand's method is
believed to have consisted of evaporating large quantities of urine to leave a black residue that was then left
for a few months. The residue was then heated with sand, driving off a variety of gases and oils. The final
substance to be driven off, was condensed as a white solid, which he called as “cold fire” as it was luminous
and glowed in the dark and also caught fire on slight warming and producing a large quantity of light. It has also
been called as the “Bearer of light”. Which element is “cold fire”?

a) Lithium b) Tungsten c) Phosphorous d) Cesium

10
JS530

44) When solid KOH is mixed with solid NH4Cl, a gas is produced. Which gas is it?

a) Chlorine b) hydrogen c) hydrogen chloride d) ammonia

45) Object A is completely immersed in water. True weight of object A is WA.


Weight of water with beaker is WB. Let B be the buoyant force. W1 and W2
are scale readings of spring balance and weighing scale respectively

a) W1 = WA b) W1 = WA + B

c) W2 = WB d) W2 = WB +B

46) Number of numbers less than 40 having exactly four divisors is

a) 15 b) 12 c) 11 d) 14

47) Antibodies play an important role in defending the body against infections by which of the following
mechanisms:

a) They engulf the bacteria and make them harmless


b) They bind to the surface of pathogens, so that they can be easily identified and removed by other
cells of the immune system
c) They enter the pathogen and prevent cell division
d) They are highly reactive and chemically react with the DNA of the pathogen

48) The figure shows a food web, where A, B, C, D etc. are different species. And the direction of
the arrows symbolizes the direction of flow of nutrients
An increase in the population of which species is likely to decrease the population of species A

a) Species D b) Species F c) Species G d) Species E

49) A point object O is kept at origin. When a concave mirror M1 placed at x = 6 cm, image is formed at infinity.
When M1 is replaced by another concave mirror M2 at same position, image is formed at x = 30 cm, then ratio
of the focal length of M1 to that of M2 is

a) b) c) 5 d)

50) The number is

a) A perfect square and a perfect cube


b) Neither a perfect square nor a perfect cube
c) A perfect cube but not a perfect square
d) A perfect square but not a perfect cube

11
JS530

51) Melting point of a substance is 10°C. What does this mean?

a) The substance is a liquid at 10°C.

b) The substance is a solid at 10°C.

c) There is an equilibrium between solid phase and liquid phase at 10°C.

d) The substance is 50% solid and 50% liquid at 10°C.

52) The following substances have approximately same molecular mass. Which is likely to have the highest boiling
point?

a) n-butane
b) isobutane
c) n-butanol
d) isobutanol

53) U-tube contains some amount of mercury. Immiscible Liquid X is poured in left arm
immiscible liquid Y is poured in the right arm. Length of liquid X is 8 cm, length X Y of liquid
Y is 10 cm and upper levels of X and Y are equal. If density of Y is 3.36 gcm-3 and of Hg is
13.6 gcm3 then density of X is

a) 0.8 gcm3 b) 1.2 gcm3 c) 1.4 gcm3 d) 1.6 gcm3

54) Let the number of rectangles formed by 6 horizontal and 4 vertical lines be and those formed by 5 vertical
and 5 horizontal lines be then we have

a) b) c) d)

55) In a human cell undergoing Meiosis, what are the total number of cellular DNA molecules present during
Prophase-I

a) 23 b) 46 c) 69 d) 92

56) During gaseous exchange in the alveoli, what happens to nitrogen?

a) There is no net nitrogen exchange, as nitrogen is filtered out by the alveoli.


b) The nitrogen is absorbed by the alveolus to form amino acids.
c) The nitrogen is filtered out by the alveolus, as the nitrogen molecule is too large to cross the gaps in
the capillaries
d) There is no net nitrogen exchange, as the blood is saturated with nitrogen

12
JS530

57) The effective resistance between A and D in the circuit shown in the
5 10  5
adjacent figure is A B

10  10 
a) 5  b)10 
C D
c) 15  d) 20  5 10  5

58) If is a rhombus and then

a) The points are concyclic


b) The quadrilateral has exactly half the area of the square with same sides as
c) The quadrilateral has area
d) The diagonals of the quadrilateral are equal and bisect each other at right angle

59) Identify the overall change in the following set of reactions:

1. Carbon dioxide → carbonic acid (H2CO3)

2. Ethanol (alcohol) → Ethanal (aldehyde)

3. Ethanal (aldehyde) → Ethanol (alcohol)

4. Sulphuric acid→ Sulphur trioxide (SO3)

Choose the correct option which best describes these conversions

a) oxidation, oxidation, reduction, reduction

b) hydration, oxidation, reduction, dehydration

c) reduction, dehydration, hydration, oxidation

d) reduction, reduction, oxidation, oxidation

60) An element with atomic number 44, is below which element in the periodic table?

a) Calcium
b) Iron
c) Argon
d) Magnesium

61) Three bulbs B1 , B2 and B3 having rated powers 100 W, 60 W and 60 W at 250 V are B1 B2
connected in a circuit as shown in the adjacent figure. If W1,W2 and W3 are the
output powers of the bulbs B1, B2 and B3 respectively, then B3

a) W1 > W2 = W3 b) W1 > W2 > W3


c) W1 < W2 = W3 d) W1 < W2 < W3
250 V

13
JS530

62) If then

a) b)
c) d) None of the above inequalities will hold

63) Which of the following is true about ATP

a) It is a derivative of one of the nitrogenous bases that form DNA


b) It splits into ADP and phosphate, and the energy produced is used by muscle cells to contract
c) It is produced in both aerobic and anaerobic conditions.
d) All of the above

64) Which of the following statements is true regarding communication in neurons

a) Free electrons are moved along the plasma membrane of the axon and control the expression of
neurotransmitters
b) A chemical signal travels along the axon and is converted into an electric impulse at the synapse
c) An electric impulse travels along the length of the axon. The electric impulse is converted to a
chemical signal at the synapse,
d) An electrical signal is converted to a chemical signal by the Myelin sheath before it reaches the
synnapse

65) Rod AB of radius 2r is joined with rod BC of radius r. They are of A B C


same material and are of same length. The combination carries a
I I
current I.
Choose the correct statement

a) VAB = 4VBC b) Current per unit area in AB and BC are equal


c) Resistance of AB is greater than that of BC d) VBC = 4VAB

66) The statement “ is correctly represented by

a) b) c) d)

67) A chemist mixes two ideal liquids A and B to form a homogeneous mixture. The densities of the liquids are 2.0
g/mL for A and 3 g/mL for B. When she drops a small object into the mixture, she finds that the object becomes
suspended in the liquid; that is, it neither sinks to the bottom nor does it float on the surface. If the mixture is
made of 40% A and 60% B, by volume, what is the density of the object?

a) 2.60 g/mL
b) 2.50 g/mL
c) 2.40 g/mL
d) 1.50 g/mL

14
JS530

68) How many different compounds can have the formula, C3H4?

a) One
b) Two
c) Three
d) Four

69) In the figure shown, the current carrying loop is fixed, where as current carrying
straight conductor is free to move. Then straight wire will
(ignore gravity) I2
I1

a) remain stationary b) move towards the loop


c) move away from the loop d) rotate about the axis
perpendicular to plane of paper

70) Two friends A and B watched a car from the top of their buildings. Angle of depression for A was more than
angle of depression for B, then

a) A’s apartment is taller than B’s apartment


b) B’s apartment is taller than A’s apartment
c) A’s apartment and B’s apartment have same height
d) We cannot compare the heights of the two apartments

71) How many times would a red blood cell pass through the heart during one complete cycle?

a) Once b) Twice c) 4 times d) 72 times

72) A gene has two alleles P(dominant) and p(recessive). The homozygous recessive combination leads to death in
the embryo stage. If two individuals with genotype Pp are mated, out of the offspring that survive to
adulthood, what is the probability of the genotype to be Pp?

a) 0.75 b) 0.33 c) 0.5 d) 0.67

73) A convex mirror of focal length f produces an image of size equal to times the size of the object. Then the
object distance is
f
a) nf b) c) (n + 1)f d) (n  1)f
n

74) Total surface area of a sphere S with radius is

a) b)
c) d)

15
JS530

75) There are many elements in the periodic table that are named after the country, where they were first made or
obtained. For example, the Latin name for copper was coined by the Romans because their chief source of
copper was from the Island of Cyprus. However, there is one country in the world which was named after an
element (the Latin name). A long time ago, it was believed that this country had mountains full of a valuable
element, however all expeditions to find these mountains failed. But the name stuck on. The element in
question is used for many applications today, and many of its compounds are used as catalysts. The ions of this
metal have very good anti-microbial property and finds application in water purification. The element is

a) Sodium
b) Gold
c) Silver
d) Francium

76) All of these species have the same number of valence electrons as nitrate ion, except

a) Carbonate ion
b) Bicarbonate ion
c) NF3
d) SO3

77) The angle between the hour arm and the minute arm of a clock at 2:10 a.m. is

a) Zero b) 4o c) 5o d) 6o

78) A craft teacher reshapes the wax from a cylinder of candle with section diameter 6 cm and the height 6 cm into
a sphere. The radius of this sphere will be

a) b)
c) d)

79) Plants absorb nitrates from the soil, which are most essential to produce:

a) Proteins
b) Carbohydrates
c) Fats
d) Cell wall

80) The dry mass (mass excluding water) of a seed in the process of germination:

a) increases over time until the first leaves appear


b) decreases over time until the first leaves appear
c) stays constant until the frst leaves appear
d) first increases and then decreaes until the first leaves appear.

16
JS530

Rough Sheet

17
JS530

Rough Sheet

18
NSEJS 2016 - ANSWER KEYS

CLICK HERE TO SEND YOUR SUGGESTION FOR THE ANSWER KEYS. LAST
DATE - 4/12/16
QUESTION PAPER - QUESTION PAPER - QUESTION PAPER - QUESTION PAPER -
JS530 JS531 JS532 JS533
QUESTI QUESTI QUESTI QUESTI
ON NO. ANSWER key ON NO. ANSWER key ON NO. ANSWER key ON NO. ANSWER key
1 b 1 d 1 a 1 d
2 d 2 c 2 b 2 b
3 b 3 a 3 b 3 d
4 b 4 c 4 d 4 c
5 d 5 b 5 c 5 d
6 a 6 d 6 c 6 c
7 a 7 b 7 a 7 a
8 c 8 d 8 c 8 c
9 d 9 d 9 a 9 b
10 b 10 a 10 b 10 d
11 c 11 c 11 d 11 b
12 c 12 c 12 c 12 d
13 a 13 c 13 b 13 d
14 c 14 c 14 d 14 a
15 c 15 a 15 d 15 c
QUESTION PAPER - QUESTION PAPER - QUESTION PAPER - QUESTION PAPER -
JS530 JS531 JS532 JS533
QUESTI QUESTI QUESTI QUESTI
ON NO. ANSWER key ON NO. ANSWER key ON NO. ANSWER key ON NO. ANSWER key
16 a 16 b 16 a 16 c
17 c 17 a 17 c 17 c
18 d 18 c 18 b 18 c
19 c 19 c 19 c 19 a
20 c 20 c 20 b 20 b
21 a 21 a 21 d 21 b
22 b 22 d 22 b 22 d
23 b 23 d 23 d 23 b
24 d 24 d 24 c 24 b
25 c 25 d 25 d 25 d
26 c 26 c 26 c 26 a
27 a 27 b 27 a 27 a
28 c 28 b 28 c 28 c
29 a 29 d 29 b 29 d
30 b 30 b 30 d 30 b
31 d 31 d 31 b 31 c
32 c 32 c 32 d 32 c
33 b 33 b 33 d 33 a
34 d 34 d 34 a 34 c
QUESTION PAPER - QUESTION PAPER - QUESTION PAPER - QUESTION PAPER -
JS530 JS531 JS532 JS533
QUESTI QUESTI QUESTI QUESTI
ON NO. ANSWER key ON NO. ANSWER key ON NO. ANSWER key ON NO. ANSWER key
35 d 35 d 35 c 35 c
36 a 36 a 36 c 36 a
37 c 37 c 37 c 37 c
38 b 38 b 38 c 38 d
39 c 39 c 39 a 39 c
40 b 40 b 40 b 40 c
41 a 41 a 41 b 41 a
42 a 42 a 42 d 42 b
43 c 43 c 43 b 43 b
44 d 44 d 44 b 44 d
45 b 45 b 45 d 45 c
46 d 46 d 46 a 46 c
47 b 47 b 47 a 47 a
48 a 48 a 48 c 48 c
49 a 49 c 49 d 49 a
50 c 50 d 50 b 50 b
51 c 51 c 51 c 51 d
52 c 52 c 52 c 52 c
53 a 53 a 53 a 53 b
QUESTION PAPER - QUESTION PAPER - QUESTION PAPER - QUESTION PAPER -
JS530 JS531 JS532 JS533
QUESTI QUESTI QUESTI QUESTI
ON NO. ANSWER key ON NO. ANSWER key ON NO. ANSWER key ON NO. ANSWER key
54 d 54 b 54 c 54 d
55 d 55 b 55 c 55 d
56 d 56 d 56 a 56 a
57 d 57 c 57 c 57 c
58 c 58 c 58 d 58 b
59 b 59 a 59 c 59 c
60 b 60 c 60 c 60 b
61 d 61 a 61 a 61 a
62 b 62 b 62 a 62 a
63 d 63 d 63 c 63 c
64 c 64 c 64 d 64 d
65 d 65 b 65 b 65 b
66 c 66 d 66 d 66 d
67 a 67 b 67 b 67 b
68 c 68 b 68 a 68 a
69 b 69 d 69 a 69 a
70 d 70 a 70 c 70 c
71 b 71 a 71 c 71 c
72 d 72 c 72 c 72 c
QUESTION PAPER - QUESTION PAPER - QUESTION PAPER - QUESTION PAPER -
JS530 JS531 JS532 JS533
QUESTI QUESTI QUESTI QUESTI
ON NO. ANSWER key ON NO. ANSWER key ON NO. ANSWER key ON NO. ANSWER key
73 d 73 d 73 a 73 a
74 a 74 b 74 d 74 d
75 c 75 c 75 d 75 d
76 c 76 c 76 d 76 d
77 c 77 a 77 d 77 d
78 c 78 c 78 c 78 c
79 a 79 c 79 b 79 b
80 b 80 a 80 b 80 b
JS 521

INDIAN ASSOCIATION OF PHYSICS TEACHERS


NATIONAL STANDARD EXAMINATION IN JUNIOR SCIENCE 2015 -16
Date of Examination: 22nd November, 2015
Time: 1500 to 1700 Hrs
Q. Paper Code: JS 521
Write the question paper code mentioned above on YOUR answer sheet (in
the space provided), otherwise your answer sheet will NOT be assessed. Note
that the same Q. P. Code appears on each page of the question paper.
Instructions to Candidates –
1. Use of mobile phones, smartphones, ipads during examination is STRICTLY
PROHIBITED.
2. In addition to this question paper, you are given answer sheet along with
Candidate’s copy.
3. On the answer sheet, make all the entries carefully in the space provided ONLY in
BLOCK CAPITALS as well as by properly darkening the appropriate bubbles.
Incomplete/ incorrect/carelessly filled information may disqualify your
candidature.
4. On the answer sheet, use only BLUE or BLACK BALL POINT PEN for making entries
and filling the bubbles.
5. Question paper has 80 multiple choice questions. Each question has four
alternatives, out of which only one is correct. Choose the correct alternative and fill
the appropriate bubble, as shown.

6. A correct answer carries 3 marks whereas 1 mark will be deducted for each wrong
answer.
7. Any rough work should be done only in the space provided.
8. Use of non-programmable calculator is allowed.
9. No candidate should leave the examination hall before the completion of the
examination.
10. After submitting your answer paper, take away the Candidate’s copy for your
reference.

Please DO NOT make any mark other than filling the appropriate bubbles properly in
the space provided on the answer sheet.
Answer sheets are evaluated using machine, hence CHANGE OF ENTRY IS NOT
ALLOWED.
Scratching or overwriting may result in a wrong score.
DO NOT WRITE ON THE BACK SIDE OF THE ANSWER SHEET.

1
JS 521

Instructions to Candidates (continued) –


Read the following instructions after submitting the answer sheet.
11. Comments regarding this question paper, if any, may be sent by email only to
iaptpune@gmail.com till 24th November, 2015.
12. The answers/solutions to this question paper will be available on our website –
www.iapt.org.in by 2nd December, 2015.
13. CERTIFICATES and AWARDS –
Following certificates are awarded by the IAPT to students successful in NSEs
(i)Certificates to “Centre Top 10%” students
(ii)Merit Certificates to “Statewise Top 1%” students
(iii)Merit Certificates and a book prize to “National Top 1%” students
14. Result sheets and the “Centre Top 10%” certificates will be dispatched to the Prof-
in-charge of the centre by January, 2016.
15. List of students (with centre number and roll number only) having score above MAS
will be displayed on our website (www.iapt.org.in) by 22nd December, 2015. See the
Eligibility Clause in the Student’s brochure on our website.
16. Students eligible for the INO Examination on the basis of selection criteria
mentioned in Student’s brochure will be informed accordingly.

2
JS 521

Indian Association of Physics Teachers


NATIONAL STANDARD EXAMINATION IN JUNIOR SCIENCE 2015-2016

Total time: 120 minutes Marks: 240

Only one out of four options is correct

1) Which of the following graphs is correct for a particle moving in a circle of radius r at a speed
of v (where ‘a’ is magnitude of acceleration) ?
a) | a | b) | a | c) | a | d) | a |

v v v v

2) Electronic configuration of Na+ is (2,8) and that of sodium element is (2,8,1). Choose the
correct statements.
i. Na+(g) is more stable than Na(g).
ii. Na+(g) is less stable than Na(g).
iii. Na+(aq) is more stable than Na(aq).
iv. Na+(aq) is less stable than Na(aq).
a) ii, iii b) i, iii c) ii, iv d) i, iv

3) What will be the remainder if the number 72015 is divided by 25?


a) 1 b) 7 c) 18 d) 24

4) In humans, the digestion of carbohydrates happens/takes place in the following parts of the
digestive system:
a) Mouth, stomach and small intestine b) Mouth and small intestine
c) Small intestine alone d) Stomach and small intestine

5) Light travels from medium X to medium Y as shown in the adjacent figure.


a) both the speed and frequency decrease X
b) speed increases and frequency decreases
c) the speed decreases and wavelength decrease
d) the speed decreases and wavelength increases Y

6) Hess' Law states that ‘the heat evolved or absorbed in a chemical reaction is the same
whether the process takes place in one or in several steps’

3
JS 521

Heat evolved in the combustion of one mole C(s) in excess oxygen is x kJ/mol, and that for the
combustion of H2(g) is y kJ/mol and for that of C6H6 (l) is z kJ/mol. Therefore, the heat change
(kJ/mol) involved in the formation one mole of C6H6 (l) from the given equation is
a) x+y+z b) 6x+3y-z c) 6x+3y+z d) x+y-z/6

7) If A(p, q + r), B(q, r + p) and C(r, p + q) are points then area of triangle ABC
1
a) p2 + q2 + r2 b) (p + q + r)2 c) (pq  qr  rp) d) zero
2

8) Photosynthesis in plants is carried out in


a) leaves b) leaves and stems
c) leaves, stems and aerial roots d) stems and roots

9) A particle moves along the x-axis according to the equation x = 6t2 where x is displacement
in meters and t is time in seconds. Therefore
a) the acceleration of the particle is 6 ms-2
b) the particle follows a parabolic path
c) each second the velocity of the particle changes by 9.8 ms-1
d) the velocity of the particle is 6 ms-1 at t = 0.5 s

10) What occurs when H2O(l) evaporates


(i) Covalent bonds are broken.
(ii) Only dipole-dipole forces are overcome.
(iii) Heat is absorbed by water from the surroundings.
(iv) It becomes oxygen and hydrogen gas.
a) Only (i) and (iii) occurs
b) Only (ii) and (iii) occurs
c) (i), (iii) and (iv) occurs
d) (i), (ii), (iii) and (iv) take place

3 3 3 3 3
11)     ...  ?
4 28 70 130 9700
a) 0.97 b) 0.99 c) 1 d) 1.03

12) The following technique that can be used for deciphering the arrangement of nucleotides in
genes.
a) karyotyping b) nucleic acid sequencing
c) DNA finger printing d) transcription

13) The “reaction” force does not cancel the “action” force because
a) the action force is greater than the reaction force
b) the reaction force exists only after the action force is removed
c) the reaction force is greater than the action force
d) they act on different bodies

4
JS 521

14) Which of the following Lewis dot structures best describes the structure of peroxide ion of
sodium peroxide?

15) What is the sum of all three digit even numbers divisible by seventeen?
a) 18846 b) 18684 c) 14688 d) 16848

16) When a red blood cell was placed in an animal cell (RBC) in 3 different solutions, the
following morphological observations were made under a microscope.
Solution 1 Solution 2 Solution 3
Normal morphology Swollen and hemolysed Shrunken or cremated
The above three solutions can be classified in the order of
a) isotonic, hypotonic and hypertonic b) hypotonic, isotonic and hypertonic
c) hypotonic, hypertonic and isotonic d) isotonic, hypertonic and hypotonic
X
17) A stone is thrown horizontally and follows the XYZ path as
shown in the adjacent figure. The direction of the acceleration Y
of the stone at point Y is
Z
a)  b)  c)  d) 

18) Ionization Energy is defined as ‘the energy required for removing the most loosely bound
electron from an isolated gaseous atom or ion’.

Molar Ionization Energy for Element ‘A’ 1st 2nd 3rd 4th 5th 6th
(kJ/mol) 1086.5 2352.6 4620.5 6222.7 37831 47277
Identify element A.
a) Nitrogen b) Oxygen c) Carbon d) Fluorine

5
JS 521

19) The adjacent sides of a parallelogram are 30 cm and 20 cm. The length of one of the
diagonal is 40 cm. What is the length of the other diagonal?
a) 60 cm b) 10 10 cm c) 20 5 cm d) 8 30 cm

20) Genetic material (DNA) in plants occurs in which of the following cell organelles?
a) Nucleus b) Nucleus and chloroplast
c) Nucleus, chloroplast and mitochondria d) Chloroplast and mitochondria
N
21) The diagram shows total internal reflection.
Which of the following statement is not true?
a) Angle AON is the incident angle A B
b) AON = BON Medium 1
c) AON is the critical angle
d) the speed of light in medium 2 is greater than that in
medium 1 O
Medium 2

22) A chemist’s report on a batch of pharmaceutical products, Aspirin (C9H8O4) (250mg tablets)
and Paracetamol (C8H9NO2) (500mg tablets) indicated a ‘+0.5%’ weight error in each tablet.
Due to this error, the consumer gets extra ‘x’ molecules of aspirin per tablet and extra ‘y’
molecules of paracetamol per tablet. Choose the ‘best’ relation between x and y.
a) x=y b) x>y c) y>x d) x=2y

23) (41)16  (14)16 is a multiple of


a) 1485 b) 1584 c) 1845 d) 1854

24) The equation given below represents the process of photosynthesis.


(i) chlorophyll (ii)
6CO2    C6 H12 O6   
sunlight
Identify (i) and (ii).
a) b) c) d)
(i) (ii) (i) (ii) (i) (ii) (i) (ii)

6H2O 6O2 6O2 6H2O 4H2O 4O2 4O2 4H2O

25) The centre of gravity of a body coincides with the center of mass
a) always
b) never
c) if the acceleration due to gravity is uniform over the body
d) if the body has a uniform distribution of mass

26) Sodium reacts with excess oxygen to form sodium oxide. A student wants to prepare 1.24g
of sodium oxide. While doing the calculations, he uses atomic number of sodium instead of
atomic mass. What is the approximate percentage error in the mass of sodium oxide
obtained due to this mistake?
a) 11% b) 23% c) 48% d) 60%

6
JS 521

27) What will be the remainder if the number (1000000)3 is divided by 143?
a) 9 b) 6 c) 1 d) 0

28) Which of the following sequence depicts the flow of blood in human circulatory system?
a)
Lungs Left atrium Left ventricle

Right ventricle Right atrium Aorta to body tissue

b)
Lungs Right ventricle Left atrium

Aorta to body tissue Right atrium Right ventricle

c)
Aorta to body tissue Left ventricle Left atrium

Right atrium Right ventricle Lungs

d)
Aorta to body tissue Left atrium Left ventricle

Lungs Right ventricle Right atrium

29) A body is in equilibrium under the combined action of several forces then
a) all the forces must be applied at the same point
b) all the forces form pairs of equal and opposite forces
c) the sum of the torques about any point must always be equal to zero
d) the lines of action of all the forces must pass through the centre of gravity of the
body.

7
JS 521

30) From the following pool of molecules

How many isomers are represented above


a) 2 b) 3 c) 4 d) 5

31) How many four digit numbers divisible by twenty nine have the sum of their digits 29?
a) 4 b) 5 c) 13 d) none of these

32) Choose the right combination of heart types and animals.


a) b)

2 chamber Sardine fish 2 chamber Sardine fish

3 chamber Amphibians 3 chamber Gharial reptiles

4 chamber Reptiles 4 chamber Owl / birds

c) d)

2 chamber Gharial reptiles 2 chamber Birds

3 chamber Birds / owl 3 chamber Gharial reptiles

4 chamber Human 4 chamber Fish sardine

33) A photo frame can be hung on the


wall with string in three different
ways as shown in the adjacent
figure below. The tension in the
string is
a) least in I b) greatest in II
c) greatest in III d) least in III

I II III

8
JS 521

34) Ionic radii of following species are


a) Si4+ > P5+ > S6+ > Cl7+ b) S6+ > P5+ > Si4+ > Al3+
c) N3- >O2- >F- >Na1+ d) Mg2+ > Na1+- >F- > O2-

35) How many triangles are there in this figure?

a) 50 b) 70 c) 84 d) 91

36) Raju Sharma, a 10th standard student participated in 100 meter sprint. During running he
developed painful muscle contraction and fell down. The physical education teacher rushed
to him and gave a hot water massage. Raju Sharma slowly recovered from the cramp. The
teacher explained the physiology behind the cramp and the sudsequent relief.
Identify the right explanation.
a) Because of the quick movement, the muscles looses its elasticity and are stressed. The
inflammation developed during this process causes cramp. After hot water massage the
inflammation subsides and the pain gets relieved.
b) During vigorous physical activity, aerobic respiration in the muscles increases which
leads to the accumulation of more CO2 in the muscles. This causes cramps. Later CO2 was
relieved upon hot water massage resulting in pain relief.
c) During vigorous physical activity, lactic acid accumulates in the muscles due to
anaerobic respiration. This causes the cramps. Hot water massage improves the
circulation of blood and O2 in the muscles. As a result lactic acid is converted into CO2 and
water. Thus the pain gets relieved.
d) During quick movements, the nerves will not co-operate with the muscles. There is a
stimulus which is taken to spinal cord and the effector function was done by motor
neurons which cause cramp. On hot water massage the stimulus was subsided. Thus the
pain gets relieved.

37) If a force acting is conservative only when


a) work done by this force is zero when the particle moves once around any closed path
b) it obeys Newton’s third law
c) its work is the change in the K.E of the particle
d) it is not a frictional force

38) Sati was studying neutralisation reaction. She accidentally dropped ‘x’ grams of a sodium
carbonate monohydrate into a 100 mL solution of HCl whose concentration was 0.25 M.
10mL from this final solution was taken and titrated against 0.05M NaOH solution to yield a
titre value of 20mL. What is the value of ‘x’.
a) 186mg b) 93mg c) 1860mg d) 930mg

9
JS 521

39) 8888888 * 8888888 this fifteen digit number is divisible by 22. Find the eighth digit in the
number.
a) 1 b) 3 c) 5 d) 8

40) The various parts of the human respiratory system are given below:
(i) Nasal passage (ii) Pharynx (iii) Wind pipe
(iv) Bronchus (v) Bronchioles (vi) Alveoli
Identify the right sequence of air passage during exhalation.
a) vi, v, ii, iv, iii, I b) vi, iv,v, iii, ii, i c) vi, v, iv, iii, ii, i d) vi, v, ii, iii, iv, i

41) A lift is moving up at constant speed. Consider the following statements:


I. The tension in the string is constant
II. The K.E of the elevator is constant
III. The gravitational P.E of the earth lift system is constant
IV. The acceleration of the elevator is zero.
V. The mechanical energy of the earth – lift system is constant.
Choose the correct option
a) Only II and V are true b) Only IV and V are true
c) Only I, II and III are true d) Only I, II and IV are true

42) Amongst the sixteen  and  marks, how many cases are incorrect?

Strip of Metal 0.5M MgSO4 0.5M ZnSO4 0.5M PbSO4 0.5M FeSO4
i) Mg    
ii) Zn    
iii) Fe    
iv) Pb    
means displacement reaction occurs
 means no displacement reaction occurs
a) 7 b) 11 c) 10 d) 9

43) In the adjoining figure segment AD, BE and CF are the altitudes of triangle ABC. Find AD  BC
if AB  AC = 409.6, BE  CF = 202.5.
A
E

B D C
a) 225 b) 256 c) 288 d) 312

10
JS 521

44) The testes of men lie in a small muscular pouch called scrotum located outside the
abdominal cavity; choose the correct reason.
a) Sperm formation in testes requires a higher temperature than the normal body temperature.
b) Seminal vesicles requires lower temperature to produce nutrients for sperm.
c) Sperm formation in testes requires a lower temperature than the normal body temperature.
d) Sperm produced in scrotum is easily released out without going into abdominal cavity.

45) Two toy cars (a and b) fixed with spring at front, collide as shown in the figure below. ‘a’ has
a mass of 200 g and is initially moving to the right. Car ‘b’ has a mass of 300 g and is initially
at rest. When the separation between the cars is minimum,

a b
a) car b is at rest b) car a has come to rest
c) both cars have the same kinetic energy (K.E) d) the K.E of the system is at a minimum

46) What is the ratio of number of electrons gained by acidified KMnO4 and acidified K2Cr2O7 in
the reaction given below

a) 5:6 b) 6:5 c) 3:5 d) 5:3

47) If 338  288  m then m = ?


a) 50 b) 32 c) 18 d) 2

48) Which of the following is NOT produced by microbial activity?


a) Yoghurt b) Bread c) Vinegar d) Antiseptics

49) A concave spherical mirror has a focal length of 12 cm. if an object is placed 6 cm in front of
it, the position of the image is
a) 4 cm behind the mirror b) 4 cm in front of the mirror
c) 12 cm behind the mirror d) 12 cm in front of the mirror

50) Methylcyclobutane is a saturated alkane. What is its molecular formula?


a) C5H10 b) C5H8 c) C5H12 d) C5H6

51) 5901AB04 is an eight digit number divisible by 792. Find A + B = ?


a) 6 b) 7 c) 8 d) 9

52) Among the following, which is not true about vaccines?


a) Vaccines contain dead microbial cells or their parts.
b) Vaccines contain antibiotics to prevent diseases.

11
JS 521

c) Vaccine contain special proteins which evoke immune system against disease.
d) Vaccines contain inactivated micro-organisms.

In the following passage some concepts about electrical circuit analysis is discussed. Based on this
answer questions 53 to 56.
(i) When two or more conductors meet at a point it is called a junction. At junctions there
should not be any accumulation of charges. This implies that the net current entering the
junction should be equal to net current leaving a junction.
(ii) In a closed circuit having resistors and batteries, the sum of voltages across the resistors
will be equal to net electromotive force (emf). This follows from the principle of
conservation of energy.
With the help of above two concepts the current and voltage across different circuit elements can be
determined.
53) The current I in the branch shown in
the adjacent figure is

a) 3.0A b) 2.5A
c) 4.0A d) 4.5A

54) A closed circuit shown in the


adjacent figure includes a resistor of resistance and battery
of emf 3.0V and internal resistance of . The internal resistance
of the battery can be considered in series with it. The voltage drop
across the resistor of resistance is

a) 2.857 V b) 3.000 V c) 2.500 V d) 1.567 V

55) A circuit which is used for charging a battery is shown in the


adjacent figure. The battery ‘B’ has emf 6V and internal
resistance of . The charging battery ‘A’ has an emf of 9V and
internal resistance of . The voltage across the points Pand Q

a) 8V b) 7V c) 4V d) 4.2V

56) The circuit shown in adjacent figure consists of an external


resistance of connected across two batteries of emfs
6.0V and 9.0V with internal resistance of each. Find the
power dissipated by the resistor.

a) 6.5W b)5.1W c) 3.5W


d) 5.5W

57) Choose the correct sets which represent the oxides as acidic:basic:neutral:amphoteric
respectively
(i) CO2: MgO: N2O: H2O (iii) P2O5: ZnO: NO: Al2O3
(ii) SO2: NO: CO: Al2O3 (iv) SO3: CaO : N2O : PbO

12
JS 521

a) i & ii b) ii & iii c) iii & iv d) iv & i


58) What is the smallest natural number when multiplied by 15 and divided by 63 gives
remainder 21?
a) 13 b) 14 c) 17 d) 20

59) Gram positive bacteria will have one of the specific characters. Identify it.
a) They have more peptidoglycon in their cell walls.
b) They show red colour on gram staining.
c) Flagella found all over the body.
d) They will have mesosomes as the extension of cell membrane.

60) A silver article turns black when kept open in air for few days. The article when rubbed with
toothpaste again starts shining because
a) Hydrogen peroxide present in the toothpaste reacts to give silver sulphate.
b) Hydrated silica present in the toothpaste reacts to give silver oxide.
c) Aluminium hydroxide present in the toothpaste reacts to give silver hydroxide.
d) Calcium carbonate present in the toothpaste reacts to give silver carbonate.

61) The sum of first four terms of an A.P is 56. The sum of last four terms of same A.P is 112. The
first term of the A.P is 11. Find the number of terms in that A.P.
a) 7 b) 8 c) 11 d) 13

62) Mitochondrial equivalent in prokaryotic bacterial cell is


a) ribosomes b) thylakoid
c) cytoplasmic plasma membrane d) cyanosomes

63) A compound ‘A’ when treated with a dilute mineral acid gives a gas which when passed
through a solution of B regenerates A. Further, a gas (C) that is obtained from the addition of
Concentrated HCl to KMnO4 crystals is used to react with B to give D. Identify A, B, C and D.
a) Na2CO3, NaOH, O2, Na2O. b) MgCO3, Mg(OH)2, OCl2 , Mg(OCl)2
c) CaCO3, Ca(OH)2, Cl2, CaOCl2 d) Al2(CO3)3, Al(OH)3, Cl2 , Al2O3.

64) If a : b = c : d then how many of the following statements are true?


(i) c(a + b) = a(c + d) (ii) d(a  b) = b(c  d)
 a 2   c2   2ac 
(iii) (a2 + b2) (ac  bd) = (a2  b2) (ac + bd) (iv)  2    2    
 b   d   bd 
a) 1 b) 2 c) 3 d) All

65) Endemic species refers to


a) threatened wild animals and plants which belong to different areas.
b) species which are capable of inter breeding.
c) those species of flora and fauna which are found exclusively in a particular area.
d) all the plants and animals mentioned in the red data book.

13
JS 521

66) 5 g of a gas at 27 °C occupied the same volume as 0.250 g of dihydrogen at 17 °C at the same
pressure. Find out molecular weight of the gas.
a) 41.37 b) 20.68 c) 25.80 d) 39.87

67) Select any three distinct digits. Form a three digit number. Form the another number by
reversing the digits. Find the difference of these two numbers. What is the G.C.D of all such
differences?
a) 9 b) 11 c) 33 d) 99

68) Which of the following option is not true about the viruses?
a) Viruses have either DNA or RNA as these genetic material.
b) Viruses will not infect bacteria, fungi and algae.
c) Viruses use host machinery to produce their own proteins.
d) Viruses are useful in the preparation of vaccines.

69) The coefficient of linear thermal expansion of steel is 11X10-6/0C. The percentage change in
the length of the rod when temperature changes by 700C.
a) 0.077% b) 0.085% c) 0.0576% d) 0.00077%

70) An acid-base titration is a technique where a solution of known concentration of acid/base is


used to determine the concentration of an unknown solution of acid/base. These titrations
typically use a pH indicator solution to denote the end point of the reaction. A pH indicator is
a compound added in small quantities to
a solution to indicate the pH visually
(generally by appearance/disappearance
or change in colour). A typical procedure 13

is as follows: A certain volume ‘V1’ of


unknown concentration ‘M1’ of HCl is pH
taken in a conical flask, to which a few
7
drops of phenolphthalein indicator
solution is added. The solution remains Equivalence
colourless. From a burette (a graduated point
dropper) a solution of NaOH, whose 1
concentration is known, ‘M2’, is added Volume run down from burette

dropwise into the conical flask until a pale Volume run down from burette

pink colour is obtained and is termed as the end point. The amount of solution dispensed
from the burette to obtain the end point is noted as ‘V2’. Phenolphthalein indicator changes
its colour to pink only when the pH of the solution is above 9.5. Similarly, another indicator,
methyl orange, is red in colour below pH 3.7 and yellow above. Given below is a graph of pH
of the solution in the conical flask and the reading of the burette in the course of the
titration. The equivalent point is theoretically defined as the point in the graph where the
number of moles HCl in the conical flask becomes equal to the number of moles of NaOH
run down the burette. Note the difference between end point and equivalence point.

14
JS 521

Which is the correct graph that represents the titration of NH4OH (from burette) with HCl?

13
13
pH
pH
7
7

1
Volume run down from burette
Volume run down from burette 1
Volume run down from burette
a) Volume run down from burette
b)

Volume run down from burette


13

13
pH
pH
7
7

1
1
Volume run down from burette
Volume run down from burette Volume run down from burette
Volume run down from burette
c) d)
1
71) There are ten numbers in a certain A.P. The sum of first three terms is 321. The sum of last
three numbers is 405. Find the sum of all the ten numbers.
a) 1165 b) 1210 c) 1221 d) 1252

72) Regeneration in animal kingdom is observed in which of the following animals?


a) Frog b) Planaria (C) Birds (D) Snakes

73) A cube of side 4 cm made of wood is floating in water of density 1.00gcc-1. When a small
steel ball is embedded in the cube it just immerses in water. If density of wood is 0.76gcc-1,
then mass of the steel ball is
a) 12.65g b) 3.84g c) 15.36g d) 22.98g

74) Given below is the pH vs volume


curve for titration of Na2CO3 (in
conical flask) with HCl. 13

p
7
H

Volume run down from

15
st
burette 2nd Equivalence point
1 Equivalence point
JS 521

Choose the correct statement based on the above graph


a) 1st equivalence point represents NaHCO3 getting protonated to H2CO3.
b) 2nd equivalence point represents NaHCO3 getting protonated to H2CO3.
c) 1st equivalence point represents deprotonation of HCl by NaHCO3 alone.
d) 2nd equivalence point represents deprotonation of HCl by Na2CO3 alone.

75) How many three digit numbers are divisible by 13 and having middle digit 5?
a) 5 b) 7 c) 10 d) 13

76) In biology, Polymerase Chain Reaction (PCR) refers to which of the following option?
a) In vitro multiplication of nucleic acids molecules.
b) In vivo multiplication of nucleic acids molecules.
c) Continuous protein synthesis from peptide.
d) Synthesis of mRNA from DNA in vitro.

77) A swing playing with small amplitude can be considered as a simple pendulum. Such a swing
is set to oscillate with an amplitude a and frequency f. When it is at its mean position, a box
of same mass as that of the seat of the swing is dropped on it and its starts moving with the
swing. Choose the correct statement
a) Amplitude is reduced to half its initial value and frequency is doubled
b) Amplitude is reduced to half its initial value and frequency is unchanged
c) Amplitude doubles and frequency is unchanged
d) Amplitude remains same and frequency is half its initial value

78) The pH of the 10-8 M HCl solution is


a) Greater than 7 and less than 8 b) Exactly 8
c) Exactly 6 d) Greater than 6 less than 7

79) Two parallel chords 96 cm and 28 cm long are on the opposite side of the centre of the circle
with radius 50 cm. Find the area of the quadrilateral whose vertices are the end points of
the chords.
a) 3488 b) 3848 c) 3844 d) 3484

80) Each chromosome contains


a) one long DNA molecule b) one long RNA molecule
c) one long sequence of amino acids d) a single gene for a protein

16
NSEJS 2015
JS 521 JS 532 JS 543 JS 554
Answer answer answer answer
Q no q no q no q no
key key key key
1 b 1 b 1 d 1 c
2 a 2 b 2 d 2 c
3 c 3 c 3 c 3 a
4 b 4 b 4 c 4 a
5 c 5 b 5 d 5 c
6 b 6 a 6 a 6 b
7 d 7 b 7 d 7 c
8 c 8 c 8 d 8 a
9 d 9 c 9 c 9 c
10 b 10 a 10 a 10 d
11 b 11 c 11 b 11 c
12 b 12 c 12 a 12 c
13 d 13 c 13 c 13 c
14 d 14 d 14 b 14 d
15 c 15 c 15 c 15 c
16 a 16 a 16 b 16 a
17 a 17 d 17 d 17 d
18 c 18 b 18 c 18 b
19 b 19 a 19 d 19 a
NSEJS 2015
JS 521 JS 532 JS 543 JS 554
Answer answer answer answer
Q no q no q no q no
key key key key
20 c 20 a 20 b 20 a
21 c 21 c 21 c 21 b
22 c 22 b 22 b 22 b
23 a 23 c 23 c 23 c
24 a 24 a 24 a 24 a
25 c 25 a 25 a 25 d
26 b 26 b 26 b 26 c
27 c 27 d 27 d 27 a
28 a 28 b 28 b 28 d
29 c 29 a 29 a 29 c
30 d 30 a 30 a 30 c
31 b 31 d 31 b 31 b
32 b 32 d 32 b 32 b
33 c 33 c 33 d 33 c
34 a 34 c 34 d 34 b
35 d 35 d 35 c 35 b
36 c 36 a 36 a 36 a
37 a 37 d 37 a 37 b
38 d 38 d 38 c 38 c
NSEJS 2015
JS 521 JS 532 JS 543 JS 554
Answer answer answer answer
Q no q no q no q no
key key key key
39 c 39 c 39 b 39 c
40 c 40 a 40 c 40 a
41 d 41 b 41 c 41 c
42 d 42 b 42 c 42 b
43 c 43 c 43 c 43 c
44 c 44 a 44 d 44 a
45 d 45 d 45 c 45 a
46 a 46 c 46 a 46 b
47 d 47 a 47 d 47 d
48 d 48 d 48 b 48 b
49 c 49 c 49 a 49 a
50 a 50 c 50 a 50 a
51 c 51 c 51 c 51 d
52 b 52 c 52 c 52 d
53 c 53 a 53 a 53 c
54 a 54 a 54 a 54 c
55 a 55 c 55 c 55 d
56 b 56 b 56 b 56 a
57 d 57 c 57 c 57 d
NSEJS 2015
JS 521 JS 532 JS 543 JS 554
Answer answer answer answer
Q no q no q no q no
key key key key
58 b 58 a 58 a 58 d
59 a 59 c 59 c 59 c
60 a 60 d 60 d 60 a
61 c 61 b 61 b 61 b
62 c 62 b 62 b 62 b
63 c 63 d 63 c 63 d
64 d 64 d 64 b 64 d
65 c 65 c 65 b 65 c
66 a 66 a 66 a 66 a
67 d 67 a 67 b 67 a
68 b 68 c 68 c 68 c
69 a 69 b 69 c 69 b
70 a 70 c 70 a 70 c
71 b 71 b 71 b 71 b
72 b 72 a 72 b 72 a
73 c 73 c 73 c 73 c
74 b 74 b 74 a 74 b
75 b 75 c 75 d 75 c
76 a 76 b 76 c 76 b
NSEJS 2015
JS 521 JS 532 JS 543 JS 554
Answer answer answer answer
Q no q no q no q no
key key key key
77 b 77 d 77 a 77 d
78 c 78 c 78 d 78 c
79 c 79 d 79 c 79 d
80 a 80 b 80 c 80 b
All quires regarding the answer keys should be sent only to
iaptpune@gmail.com with in December 2nd 2015
JS 530

INDIAN ASSOCIATION OF PHYSICS TEACHERS


NATIONAL STANDARD EXAMINATION IN JUNIOR SCIENCE 2014 -15
Date of Examination: 23rd November, 2014

Time: 1500 to 1700 Hrs

Q. Paper Code: JS 530


Write the question paper code mentioned above on YOUR answer sheet (in
the space provided), otherwise your answer sheet will NOT be assessed. Note
that the same Q. P. Code appears on each page of the question paper.
Instructions to Candidates –

1. Use of mobile phones, smartphones, ipads during examination is STRICTLY PROHIBITED.


2. In addition to this question paper, you are given answer sheet along with Candidate’s copy.
3. On the answer sheet, make all the entries carefully in the space provided ONLY in BLOCK
CAPITALS as well as by properly darkening the appropriate bubbles.
Incomplete/ incorrect/carelessly filled information may disqualify your candidature.
4. On the answer sheet, use only BLUE or BLACK BALL POINT PEN for making entries and filling
the bubbles.
5. Question paper has 80 multiple choice questions. Each question has four alternatives, out of
which only one is correct. Choose the correct alternative and fill the appropriate bubble, as
shown.

6. A correct answer carries 3 marks whereas 1 mark will be deducted for each wrong answer.
7. Any rough work should be done only in the space provided.
8. Use of non-programmable calculator is allowed.
9. No candidate should leave the examination hall before the completion of the examination.
10. After submitting your answer paper, take away the Candidate’s copy for your reference.

Please DO NOT make any mark other than filling the appropriate bubbles
properly in the space provided on the answer sheet.

Answer sheets are evaluated using machine, hence CHANGE OF ENTRY IS


NOT ALLOWED.

Scratching or overwriting may result in a wrong score.

DO NOT WRITE ON THE BACK SIDE OF THE ANSWER SHEET.

1
JS 530

Instructions to Candidates (continued) –

Read the following instructions after submitting the answer sheet.

11. Comments regarding this question paper, if any, may be sent by email only to
iaptpune@gmail.com till 25th November, 2014.
12. The answers/solutions to this question paper will be available on our website –
www.iapt.org.in by 3rd December, 2014.
13. CERTIFICATES and AWARDS –
Following certificates are awarded by the IAPT to students successful in NSEs

(i)Certificates to “Centre Top 10%” students

(ii)Merit Certificates to “Statewise Top 1%” students

(iii)Merit Certificates and a book prize to “National Top 1%” students

14. Result sheets and the “Centre Top 10%” certificates will be dispatched to the Prof-in-charge
of the centre by January, 2015.
15. List of students (with centre number and roll number only) having score above MAS will be
displayed on our website (www.iapt.org.in) by 22nd December, 2014. See the Eligibility
Clause in the Student’s brochure on our website.
16. Students eligible for the INO Examination on the basis of selection criteria mentioned in
Student’s brochure will be informed accordingly.

2
JS 530

Indian Association of Physics Teachers

NATIONAL STANDARD EXAMINATION IN JUNIOR SCIENCE 2014-2015


Total time: 120 minutes Marks: 240

Only one out of four options is correct

1) Three identical vessels carrying equal amount of water are placed in three lifts. Lift A is
accelerating upwards, lift B is accelerating downwards while lift C is moving up with
constant velocity. The pressure at a depth h from free surface in the three vessel is
measured as pA , pB and pc then which of the following is true
a) pA = pC > pB b) pA > pC > pB c) pA > pC = pB d) pA = pC = pB

2) In the reaction,
2KClO3 →2KCl + 3O2
What is the volume of oxygen released under NTP conditions when 36.75g of KClO3 is
heated?
a) 3.6 litres b) 7.2 litres c) 10 litres d) 1.8 litres

3) Figure shows a square grid of order 3, which of the following is correct


formula for the total number of squares in a similar grid of order n.
(a) b) c) d)

4) If the distance between genes - W. X. Y, and Z on a chromosome are as follows: from W-Y is
18 units, W-X is 26 units, W-Z is 40 units, X-Y is 8 units and X-Z is 14 units, the sequence of
W, X, Y, Z genes on the chromosome would be:

a) W, X, Y, Z. b) X, Y, W, Z. c) Y, W, X, Z. d) W, Y, X, Z.

5) In a plant, 30 megaspore mother cells are generated. If all the ovules are fertilised, how
many seeds are expected to be formed?

a) 30 b) 60 c) 90 d) 120

6) A water filter advertisement claims to provide 8 litres of water per hour. How much time
does it take to fill four bottles of 1.5 litres each?

a) 2 hr b) 1 hr c) 45 min d) 30 min

7) Which among the following salts will not change the pH of water on addition
a)Sodium chloride b) Sodium cyanide
c) Sodium bicarbonate d) Sodium carbonate

8) A particle starting from rest is moving with uniform acceleration in a straight line. The
percentage increase of the displacement of the particle in 9th second compared to that in
the immediate previous second is about
a) 8.3% b) 13.3% c) 20.6% d) 24.5%

3
JS 530

9) An inflated balloon with a heavy rock tied to it submerges in water. As the balloon sinks
deeper and deeper, the buoyant force acting on it

a) increases b) decreases
c) remains nearly unchanged d) Initially increases and then decreases

10) For a first order reaction, the ratio of the times taken for completion of 99.9% and 50% of
the reaction is
a) 8 b) 9 c) 10 d) 12

11) If set of marbles, of radius 5 cm, is poured into a cube of side 1 m. The maximum number
of marbles that can be filled into the box are

a) 1000 b) 2000 c) 1500 d) 3000

12) Most of the insects have egg, larva, pupa and adult stages in the life cycle. This is primarily
due to :

a) relatively short adult phase. b) terrestrial habitat they have adapted to.

c) eggs storing little reserved food. d) flying mode of locomotion majority have.

13) Which of the following has been proved to contribute to the transport of water in vascular
plants?

i. Positive root pressure


ii. Hyrophilic cell walls
iii. Capillarity
iv. Transpirational pull
v. Cohesion between water molecules
a) i, ii, iii, iv and v b) only I, iii and v

c) only ii, iv and v d) only I, ii, iv and v

14) A round table cover has six equal designs as shown in the adjacent
figure. If the radius of the cover is 4 cm, then cost of making the
designs at the rate of Rs 10.00 per cm2 (round off your answer to a
nearest rupee) is

a) Rs 85 b) 86 c) 87 (d) 90

15) Which of the following series of elements have nearly the same atomic radii?
a) F, Cl, Br, I b) Na, K, Rb, Cs c) Li, Be, B, C d) Fe, Co, Ni, Cu

16) A particle is moving along a straight line. Its velocity time


graph is as shown in the adjacent figure. Then
Match the following
Physical quantity Remarks
(i) Acceleration at 4 second (p) Positive
(ii) Velocity at 4 second (q) Negative
(iii) Direction of motion at 2 second (r) Zero

4
JS 530

a) (i) is (p); (ii) is (q) and (iii) is (r)


b) (i) is (q); (ii) is (r) and (iii) is (p)
c) (i) is (r); (ii) is (r) and (iii) is (p)
d) (i) is (q); (ii) is (p) and (iii) is (r)

17) A rectangular parallelepiped with sides a, b and c in the ratio 3:2:1 is kept on a uniformly
rough horizontal surface as shown in the figures below. The value of limiting friction is

(i) (ii) (iii)

a) Minimum in (i) b) Minimum in (ii) c) Minimum in (iii) d) Same in all cases

18) Which of the following has the maximum number of unpaired electrons?
a) Ti 3+ b) V 3+ c) Fe3+ d) Fe2+

19)The houses of a row are numbered consecutively from 1 to 49. Find the value of x such that
the sum of the numbers of houses preceding the house numbered x is equal to the sum of
the numbers of the houses following it.

a) 25 b) 37 c) 35 d) No such value exists

20) Urea is the principle excretary waste in larval as well as adult phases of:

a) Cockroach b) Frog c) Crab d) Starfish

21) Use of excessive NKP fertilizers has resulted in:

i. Reduction in number as well as species of nitrogen fixing bacteria


ii. Increase in number as well as types of denitrifying bacteria
iii. Increase in the proportion of coarse particles in soil.
iv. Increase in number as well as types of ammonifying microbes
v. Increase in number as well as types of nitrifying bacteria
a) i, ii, iii, iv and v b) only ii, iv and v

c) only i and ii d) only i, ii and iii

22)Along a road lie an odd number of stones placed at intervals of 10 metre. These stones have
to be assembled around the middle stone. A person can carry only one stone at a time. If a
man starts from one of the end stones, and by carrying them in succession he covers 3 km
to pile all stones at the centre. The number of stones is then

a) 12 b) 15 c) 25 d) 30

5
JS 530

23) The following variation of properties is generally seen in the periodic table.
a) Atomic radius and ionization energy both increase across a period.
b) Atomic radius increases and ionization energy decreases across a period.
c) Atomic radius decreases and ionization energy increases across a period.
d) Atomic radius and ionization energy both decreases across a period.

24) The erythrocytes separated from human blood were mixed with certain fluids on a slide and
observed under the microscope. Which of the following will be the expected result?

a) With distilled water the cells swell and eventually burst.


b) With serum the cells clump and coagulate.
c) With sea water the cells undergo no apparent change.
d) With tap water cells shrink and appear cremated.

25) The largest of the jelly-fishes grow over 1 meter in diameter and can survive without any
skeletal support due to:

a) rapid beating of cilia creating an upthrust.


b) the bottom dwelling habit.
c) high salinity and subsequent buoyancy of sea water.
d) upwelling currents in water.

26)The diagram shows a road


network. All vehicles drive in
one direction from A to B.
Numbers represent the maximum
flow rate (capacity of roads) in
vehicles per hour. The maximum
number of vehicles that can drive
through the network every hour is

a) 315 b) 215 c) 240 d) 340

27) An excess of NaOH solution is added gradually to an aqueous solution of ZnSO4. Which of
the following will happen?
a) A white precipitate is formed which does not dissolve in excess NaOH.
b) A green precipitate is formed which dissolves in excess NaOH.
c) A white precipitate is formed which dissolves in excess NaOH.
d) No observable change occurs.
28) If two bodies of different masses, initially at rest, are acted upon by the same force for the
same time, then both bodies acquire the same
a) Velocity b) momentum c) acceleration d) kinetic
energy
29) It is more difficult to walk on a sandy road than on a concrete road. The most appropriate
reason for this is

a) sand is soft and concrete is hard


b) the friction between sand and feet is less than that between concrete and feet
c) the friction between sand and feet is more than that between concrete and feet
d) the sand is grainy but concrete is solid

6
JS 530

30) In which of the following series of transition metal ions, all metal ions have 3d2 electronic
configuration?
a) Ti+, V4+, Cr6+, Mn7+ b) Ti2+, V3+, Cr4+, Mn5+ c) Ti3+, V2+, Cr3+, Mn4+ d) Ti4+, V3+, Cr2+, Mn3+

31)A piece of wire 60 cm long is cut into two parts, one of them being 24 cm long. Each part is
then bent to form a square. The ratio of the area of the larger square to the smaller square
is

a) 11/3 b) 7/4 c) 3/2 d) 9/4

32) In the cells of oil seeds which of the cell organelles have to be more active?

a) Mitochondria. b) Smooth Endoplasmic Reticulum.

c) Rough Endoplasmic Reticulum. d) Nucleoli.

33) Which of the following sugars tastes most sweet?

a) Fructose. b) Ribose. c) Sucrose. d) Lactose.

34)Scientists in an R & D company made three design improvements on a car: the first saves
50% of fuel, the second saves 30% of fuel and the third saves 20%. If the company
implements all three design changes at once, the new car will consume fuel that is ____ %
of the fuel consumption of normal car.

a) 50% b) 28% c) 100% d) 20%

35) Aluminum is extracted from its oxide by


a) Calcination b) Reduction
c) Thermal decomposition d) Electrolysis

36) The magnetic force on a moving charged particle can change the particle’s

a) speed only b) direction only


c) Both speed and direction d) neither of speed nor direction

37) A ray of light is incident on system of mirror as shown in the adjacent


figure. What is the total deflection ( ) of the ray when it emerges out
after two reflections?
a) 2200 b)1800 c)1400 d) 1200

38) The oxidation number of sulphur in sodium thiosulphate (Na2S2O3) is


a) +1 b) +2 c) +3 d) +4

39)The adjacent figure is a modification of the Switzerland flag to suit the


problem! Five identical small squares form the central cross. The length of
each side of the big square is 10 m. If the area of the white cross is 20% of
the area of the square flag, then the length of the side of the small square
is

a) 1.75 m b) 2.25 m c) 1.6 m d) 2 m

7
JS 530

40) The algae belonging to which group can sustain normal growth at the greater depth of
ocean?

a) Green algae. b) Blue-green algae. c) Brown algae. d) Red algae.

41) Snakes, the cold blooded animals, flick their bifid tounge often to:

a) sample air for chemoreceptors. b) sense vibrations in earth.


c) sense the nature of substratum. d) sense the temperature of air.

42)We all know that the sky appears to touch the ground at a distance. The distance at which
we perceive the sky to touch the ground is called horizon. The reason for the perception is
due to the fact that the Earth is a sphere (almost) and not a flat surface. Which of these
pictures below accurately depict the horizon for a person standing on a high rise building
like Burj Khalifa in Dubai? (Here, ‘h’ represents the height of the building while line ‘H’
represents the horizon)

43) Sulphuric acid is manufactured by the contact process in which sulphur dioxide reacts with
oxygen in presence of a catalyst. If 5.6 moles of SO2 reacts with 4.8 moles of O2 and a large
excess of water, the maximum number of moles of H2SO4 that can be obtained is.
a) 5.6 b) 11.2 c) 4.8 d) 1.4

44) The element essential for determining the three dimentional structure of proteins is:

a) carbon. b) hydrogen. c) nitrogen. d) sulphur.

45) The general indigestion experienced by a patient suffering from obstructive jaundice is due
to:

a) the lack of emulsification of lipids.


b) the low pH in the intestine not supporting optimal activity of enzymes.
c) the acceleration of intestinal peristalsis reducing the retention time for food.
d) the diffusion of bile pigments in blood suppressing secretion of digestive juices.

46) A number is said to be a triangular number if it is the sum of consecutive numbers


beginning with 1. Which one of the following is not a triangular number

a) 1431 b) 190 c) 506 d) 28

47) The equivalent weight of MnSO4 is half its molecular weight when it is converted to
a) Mn2O3 b) MnO2 c) MnO4- d)MnO42-

8
JS 530

48) A light source of diameter 2 cm is placed 20 cm behind a circular opaque disc of diameter
4cm. Shadow is formed on a screen at a distance of 80cm. The ratio of the area of umbra
and penumbra shadow regions is equal to.
a) 0.22 b) 0.18
c) 0.58 d) 0.11

49) Consider the following two statements.


Statement 1: The direction of acceleration of a particle must be always same as that of
velocity.
Statement 2: Acceleration is the rate of change of velocity.
Choose the correct option
a) Statement (1) is correct while statement (2) is wrong
b) Statement (1) and (2) are correct
c) Statement (1) is wrong while statement (2) is correct
d) Statement (1) and (2) are wrong.

50) Rust is a mixture of


a)FeO + Fe(OH)2 b) FeO + Fe(OH)3
c) Fe2O3 + Fe(OH)3 d) Fe3O4 + Fe(OH)3

51) If the distance between A and B is 230 km, B and C is 120 km, C and A is 350 km. Also, if the
distance between C and D is 200 km, distance between D and B is 330 km and distance from
A to E is 100 km and distance between D and E is 570 km. The diagram (not drawn to scale)
that represents this graphically is

52) Which of the following contains the same number of atoms as 13.5 grams of aluminum?
a)10 g of sodium b) 10 g of magnesium
c) 20 g of potassium d) 20 g of calcium

9
JS 530

53) Consider the following two statements. Statement 1 is an assertion of a concept while
Statement 2 is the reason.
Statement (1): When red light travels from air to water, for observer in water it appears to
be still red.
Statement (2): Colour of light is associated with frequency and frequency does not change,
when it travels in different medium.
Choose the correct option
a) Statement (1) is correct while statement (2) is wrong
b) Statement (1) and (2) are correct
c) Statement (1) is wrong while statement (2) is correct
d) Statement (1) and (2) are wrong.

54) A spring of spring constant 7600 Nm-1 is attached to a block of


mass 0.25 kg as shown in figure. Frequency of oscillation on
frictionless surface is
(a) 27.76 Hz (b) 39.26 Hz (c) 9681.5 Hz (d) 98.39 Hz

55) The following data was recorded for the reaction A + B →Product at 298K.
Experiment
No. [A] [B] Rate of reaction

1 1.00M 0.15M 4.20 x 10-3


2 2.00M 0.15M 8.40 x 10-3
3 1.00M 0.30M 8.40 x 10-3
From the above data one can conclude that
a) Rate ∝ [A]2[B] b) Rate ∝ [A][B]2 c) Rate ∝ [A][B] d) Rate ∝ [A]2[B]2

56) The sum of 2 digits x and y is divisible by 7. What can one say about a 3 digit number
formed by these two digits.

a) xyx is divisible by 7 b) xxy is divisible by 7


c) xyx is divisible by 72 d) yyx is divisible by 7

57) Most of the microbes employed in commercial fermentation for producing antibodies are:

a) thread bacteria. b) yeasts.


c) eubacteria. d) ascomycete fungi.

58) Most of the cellular RNA is synthesised and stored respectively in:
a) cytoplasm and ribosomes. b) ribosomes and cytoplasm.
c) nucleus and ribosomes. d) ribosomes and nucleus.

59)A number of bacteria are placed in a glass. 1 second later each bacterium divides in three,
the next second each of the resulting bacteria divides in three again, and so on. After one
minute the glass is full. When was 1/9th of the glass full?

a) 15 sec b) 58 sec c) 45 sec d) 38 sec

10
JS 530

60)A number x is a rational numbe If there exists integers p and q such that x = p/q. This is the
definition of rational numbers in which,

a) both p & q can be zero b) both p & q should not be zero


c) p can be zero but not q d) q can be zero but not p

61) There is a solution of 1 Litre HCl of pH 5. When 9 L of water is added to this solution, the pH
turns out to be
a) pH 5 itself b) pH 10 c) pH 4 d) pH 6

62) A wave is sent along a string by oscillating at one end. If the tension in the string is
increased then speed of the wave and wavelength of the wave

a) both increase b) speed increases, wavelength decreases


c) both decreases d) wavelength increases, speed decreases

63) Clock A based on oscillations of spring and clock B is based on pendulum motion. Both the
clocks keep the same time on earth. If they are taken to a planet having half the density of
earth and twice the radius
a) then A runs faster than B. b) B runs faster than A.
c) both will run at same rate as earth d) both will run at equal faster rate than earth.

64) Assuming ideal gas behavior, which among the following gases will have the least density
under room temperature and pressure.
a) Nitrogen b) Oxygen c) Ozone d) Fluorine

65)The least positive integer, , such that 2 divides , 3 divides , 4 divides , 5 divides
and 6 divides is

a) 52 b) 120 c) 720 d) 62

66) Which of the following places having same number of species is considered most
biodiverse?

a) species belonging to more taxa.


b) many of the species endemic.
c) many of the species economically important.
d) species adapted to greater number of habitats.

67) Axolotl, the Mexican Salamander, shows ‘neoteny’ or larva becoming sexually mature (adult).
Which of the following characters indicate larval features in it?

i. Naked skin
ii. External gills
iii. Lidless eyes
iv. Laterally compressed tail
v. Clawless digits

a) i, ii, iii, iv and v. b) only i, ii, iv and v


c) only ii, iii, iv and v. d) only ii and iv.

11
JS 530

68) The solution set of the inequality is

a) Set of all positive real numbers b) set of all non-negative real numbers
c) set of all real numbers except -1 d) Set of all numbers satisfying

69) Which among the following organic compounds is likely to have more than one possible
structure?
a) CH4 b) C3H8 c) C2H4 d) C3H6

70) In the circuit B1, B2, and B3 represent identical bulbs.


Consider the case
(i) With resistance R4 (ii) without the resistance R4
(R4 comparable with resistance of bulb)
a) B1,B2 and B3 glow with equal brightness in both
cases.
b) B1 brightest in (i) and in (ii) B2 and B3 become brighter and B1 dimmer compared to case
(i).
c) B2B3 brightest in case (i) and B1 becomes brighter in (ii).
d) B1brightest in (i) and B2 becomes brighter in comparison to B3 in (ii).

71) Three identical resistors each of resistance R are


connected in the following four configurations.
Rank the arrangement in the order of their
equivalent resistors from highest to lowest.

a) i,ii,iii & iv b) iv,iii,ii & i

c) i,iii,iv & ii d) ii,iv,iii & i

72) Given below are the structures of the famous molecules called Aspirin and Paracetamol.
Which among the listed functional groups do the two molecules put together NOT contain?

a) Ketone b) Ester c) Alcohol d) Carboxylic acid

12
JS 530

73) Number plate of a vehicle consists of 4 digits. The first digit is the square of second. The
third digit is thrice the second and the fourth digit is twice the second. The sum of all 4
digits is thrice the first. The number is

a) 1132 b) 4264 c) 9396 d) 1642

74) The pteridophytic character that is considered to have led to the evolution of gymnosperms
is:

a) homospory. b) heterospory.

c) furcate venation. d) sporophylls distinct from vegetative leaves.

75) Seeds trapped in crevices of rocks soak in water, swell and cause fragmentation of rock.
The process involved is termed:

a) imbibition. b) osmosis. c) Tyndall effect. d) water potential.

76) If the highest common factor of a, b and c is 1, where a, b and c belong to the set of natural
numbers, then the highest common factor of (a X b) and c is

a) c b) a X b c) 1 d) Insufficient data

77) If a firecracker burns with emission of red colour light, which cation is it likely to contain?
a) Sodium b) Copper c) Iron d) Lithium

78) A positively charged insulator is brought in contact with an uncharged conductor then
a) conductor acquires positive charge due to conduction
b) conductor acquires negative charge due to induction
c) conductor acquires positive charge due to induction
d) conductor cannot acquire any charge.

79) Two infinite wires carrying identical current are placed at position A and C
normal to plane of the paper as shown in the adjacent figure. The resultant
magnetic field (B) at a point P on the perpendicular bisector is
a) Along perpendicular bisector pointing towards line AC
b) Along Line joining PC and pointing towards C
c) Along line joining PA and pointing towards A
d) Along line parallel to AC and pointing towards right

80) When an incandescent bulb is switched on and the outer glass bulb also gets heated up.
This is due to
a) Conduction and convection of heat from filament to the bulb by the medium inside the
bulb at lower temperatures and by radiation of heat at higher temperature
b) Convection of heat from filament to the bulb by the medium inside the bulb at all
temperatures
c) Radiation of heat from filament to the bulb at all temperatures
d) Conduction of heat from filament to the bulb by the medium inside the bulb at higher
temperatures and by radiation of heat at lower temperature

13
514

Indian Association of Physics Teachers


NATIONAL STANDARD EXAMINATION IN JUNIOR SCIENCE 2013-2014
Date of Examination 24th November 2013
Time: 15.00 to 17.00 Hrs

Question paper code 5 1 4


Five One Four

Please read the instruction carefully before answering


INSTRUCTIONS TO CANDIDATES
1. On the answer sheet, fill up all the entries carefully in the space provided, ONLY IN BLOCK CAPITALS. Use only BLUE or BLACK
BALL PEN for making entries and marking answers. Incomplete / incorrect / carelessly filled information may disqualify your
candidature.

2. Write the Q. P. Code No. mentioned above on YOUR answer sheet (in the space provided). Otherwise your answer sheet will
NOT be valued..

3. The question paper has 80 multiple choice questions. Each question has 4 options, out of which only one is correct. Choose the
correct answer and shade the oval in the corresponding box on the answer sheet as shown below:

Correct Method

4. A correct answer carries 3 marks and 1 mark will be deducted for each wrong answers.

5. All rough work may be done on the blank sheet provided at the end of the question paper.

6. PLEASE DO NOT MAKE ANY MARKS OTHER THAN SHADING IN THE SPACE PROVIDED ON THE ANSWER SHEET. Answer
sheets are evaluated with the help of a machine. Due to this, CHANGE OF ENTRY IS NOT ALLOWED.
7. Scratching or overwriting may result in wrong score. DO NOT WRITE ANYTHING ON THE BACK OF ANSWER SHEET.

8. Use of a nonprogrammable calculator is allowed.

9. The answers / solutions to this question paper will be available on our website - www.iapt.org.in by 30th November 2013.

CERTIFICATES & AWARDS

i) Certficates to top 10% students of each centre.


ii) Merit certificates to statewise Top 1% students.
iii) Merit certificate to Nationwise Top 1% students.

10. Result sheets and the “centre top 10%” certificates of NSEJS are dispatched to the Professor in charge of the centre. You will
get your marks from the Professor in charge of your centre by January 2014 end.

11. 300 (or so) students are selected for the next examination - Indian National Junior Science Olympiads (INJSO). Individual letters
are sent to these students ONLY.

12. No querries will be entertained in this regard.

1
514

Indian Association of Physics Teachers


NATIONAL STANDARD EXAMINATION IN JUNIOR SCIENCE 2013-2014

Total time: 120 minutes Marks: 240


Only one out of four options is correct

1) One side of a glass slab is silvered as shown in the figure. A ray of


light is incident on the other side at angle of incidence i 45o .
Refractive index of glass is given as 1.5 . The deviation of the ray of
light from its initial path when it comes out of the slab is (Given
2
sin 1
28 )
3
o o o o
(a) 90 (b) 180 (c) 120 (d) 45
2) The number 5 41 lies between
a) 29 and 30 b) 30 and 31 c) 31 and 32 d) 32 and 33

3) Oxidation number and co-ordination number of Pt in cisplatin PtCl2(NH3)2 are respectively


a) + 4 and 2 b) +2 and 4 c)0 and 4 d)+2 and 6

Passage for Q(4-6): In a field one summer's day a Grasshopper was hopping about, chirping and
singing to its heart's content. An Ant passed by, bearing along with great toil an ear of pea he
was taking to the nest. "Why not come and chat with me," said the Grasshopper, "instead of
toiling and moiling in that way?" "I am helping to lay up food for the winter," said the Ant, "and
recommend you to do the same." "Why bother about winter?" said the Grasshopper; "We
have got plenty of food at present." But the Ant went on its way and continued its toil. When
the winter came the Grasshopper had no food and found itself dying of hunger - while it saw
the ants distributing every day corn and grain from the stores they had collected in the
summer. Then the Grasshopper knew: It is best to prepare for days of need

4) In the passage given above there seems to be a factual error with respect to the ant
carrying the food to the nest. The most probable reason for this mistake would be

a) Pea pod is too heavy for an ant to carry to its nest.


b) Pea cannot be carried by an ant in the summer because it is a Rabi crop.
c) Ant couldn’t have passed by easily since it is the favourite food of grasshoppers.
d) Grasshoppers avoid coming out in summer and thus there cannot be
grasshopper in the story.

3
514

5) What could be the most plausible reason that all the ants that toiled and moiled in the
summer were happy and content in the winter?

a) Ants were probably happy since their food was not shared with Grasshopper.
b) Ants need not worry to work anymore since they had food stocked.
c) Ants were happy since they enjoyed working together in summer.
d) Food that was procured was efficiently distributed and managed so that all the
ants were fed equally.

6) Grasshopper was at fault in this story mostly because

a) Of its attitude towards ants who were working tirelessly.


b) Of not having a forethought to store food for the upcoming winter season.
c) Of chirping and singing to its heart's content in the summer.
d) It should have asked ants for the food and managed to surpass the winter
somehow.

7) An object of mass 1 kg is made to slide down a smooth inclined plane of length 20 m . If the
kinetic energy possessed by the body at the bottom of the plane is 100 J , then the inclination of
the plane with the horizontal is (take g=10ms-2)
(a) 45  (b) 37  (c) 60  (d) 30 

8) Two circles each of radius 3 touch each other externally in the plane. In how many ways can a
circle of radius 8 be placed in the plane touching each of these two circles?
a) 2 b) 4 c) 6 d) 8

9) Which of these elements has the greatest electronegativity?


a) Br b) N c) O d) S

10) An inclined plane of inclination θ is placed in water as


shown in figure given below. Consider a small area
h
(∆A) around point P at a depth h. If Density of water is P
∆A
ρ and acceleration due to gravity is g the force
θ
experienced by ∆A due to hydrostatic pressure is
a) ρgh(∆A) b) ρgh(∆A)sinθ c) ρgh(∆A)cosθ d) ρgh(∆A)secθ

11) If 3x + 3y – 1, 4x2 + y -5, 4x + 2y are the sides of an equilateral triangle, its area is closest to the
integer
a) 84 b) 85 c) 86 d) 87

4
514

12) The pH of a 0.025 M solution of KOH is


a) 1.60 b) 3.69 c) 10.31 d) 12.40

13) Consider the following two statements about light &sound. Choose the most appropriate option
(i) When light and sound travel from air to water, light may bend towards normal while sound
may bend away from normal.
(ii) Sound is longitudinal wave while light is transverse wave.
a) Statement (i) is correct while statement (ii) is incorrect
b) Statement (i) and statement (ii) are both correct and statement (ii) is not the reason for
statement (i)
c) Statement (i) and statement (ii) are incorrect
d) Statement (i) and statement (ii) are correct and Statement (ii) is the correct reason for
statement (i)

14) If xy2 =a3, yz2=b3 and zx2 = c3 then z3 equals


bc 4 b4c b2 c 4 ab 4
a) b) c) d)
a2 a2 a2 c2

15) How many H atoms are in 3.4 g of C12H22O11?


a) 6.0 1023 b) 1.3 1023 c) 3.8 1022 d) 6.0 1021

Passage Q (16-18): Diatoms are the most common photosynthetic aquatic microorganisms
group of algae which are unicellular and can exist as colonies in the shape of filaments or
ribbons, fans, zigzags or stars depending on the quality of the water. Diatom communities
are a popular tool for studies of water quality and pollution management. Karthik from
Bangalore recently went on a field trip from Bangalore to Mysore. On the way he stopped
his car at a sewage canal, a lake and a mountain stream and collected water samples from
all of these places for his lab work. After a careful analysis of his water samples, he observed
that diatoms came with varying size/shape and the size /shape increases has (have?)
something to do with the water quality.

16) Below are the diatoms observed under a microscope by Karthik. Help him to recognise
the correct order of sample localities (Viz., Canal, Stream and lake).

A B C

5
514

a) A- Mountain stream, B- Sewage Canal, C- Lake


b) A- Sewage Canal, B- Lake, C- Mountain Stream
c) A- Lake, B- Mountain stream, C- Sewage Canal
d) A- Mountain stream, B- Lake, C- Sewage canal

17) What is the take home message from the above experiment

a) Diatoms come in different sizes and shapes


b) The difference in size and shapes from different water samples is
suggestive of the intensity of water pollution.
c) Karthik enjoyed collecting samples from different locations.
d) Nothing can be inferred from the above experiment.

18) In the above experiment, difference in sizes and shapes of diatoms should be inferred as
a) Different species of diatoms.
b) Different genera of diatoms.
c) Different families of diatoms.
d) Different orders of diatoms.
19) The percentage change in acceleration due to gravity at an altitude equal to radius of earth
compared to that on the surface of earth is given by
a) 25% increase b) 35% decrease c) 75% decrease d) 25% decrease

1 1 1
20) Let a, b be two positive real numbers such that a < b < and let x (a ) (b ) . Then
a a b
a) x is always greater than zero b) x is always less than zero
c) x =0 d) No such definite conclusion can be drawn about x

21) Which of the following species has standard enthalpy of formation as 0 kJ mol–1?
a) H2O(l) b) Na(g) c) Na(s) d)CO2(g)

22) A particle accelerates with a constant acceleration a0 and reaches a maximum velocity and then
decelerates with a0 and comes to rest. The position time graph describing the situation is best
represented by

(a) (b)

6
514

(c) (d)

23) Let m be the number of distinct (non congruent) integer-sided triangles each with perimeter 15
and n be the number of distinct (non congruent) integer sided triangles each with perimeter 16.
Then m-n equals
a) -2 b) 0 c) 2 d) -4

24) What is the molality of a solution made by dissolving 100 g of bromothymol blue (C27H28Br2O5S)
in 1.00 L of ethanol on a winter’s day at 10ºC? The density of ethanol at this temperature is
0.7979 kg L-1.
a)0.100 mol kg–1 b) 0.128 mol kg–1
c) 0.160 mol kg–1 d) 0.201 mol kg–1

25) Two bulbs of specifications 50W, 220V and a 100 W, 200V are connected first (i) In parallel and
then (ii) In series across 220V power supply. Choose the correct statement

(a) In (i) 50 W will glow brighter and in (ii) 100 W will glow brighter

(b) In (i) 50 W will glow brighter and (ii) both will glow equally brighter.

(c) In (i) 100 W will glow brighter and (ii) 50 W will glow brighter

(d) In both cases the 50 W bulb will glow brighter.

26) Let T be the number of 4- digit integers, each ending in 3 (in units place) and each divisible by
11. Then
a) 70 ≤ T ≤ 79 b) 80 ≤ T ≤ 89 c) 90 ≤ T ≤ 99 d) T ≥ 100

27) 1.000mL of 0.1000 mol L–1 hydrochloric acid was diluted to 100.0 mL with deionised water.
10.00 mL of this solution was diluted to 100.0 mL again using deionised water. What is the pH of
the final solution?
a) 2 b) 3 c) 4 d)5

7
514

28) While playing football, Dimple fell down and was badly wounded on her left ankle. The Doctor
prescribed her antibiotics for a week which should have healed her of the wound in a week.
However, Dimple’s wound did not heal in a week. What among the following could have been
the reason for inability of the wound to heal in the prescribed time frame given by the doctor?
a) Prescribed medicine’s date was expired.
b) Dimple wouldn’t have taken the full course of the antibiotics.
c) Both a & b could be the reason
d) Doctor’s inability to prescribe the correct medicine for the wound.

29) Which of the following situation is impossible?


a) A body having velocity and acceleration in opposite directions.
b) A body having zero velocity and non zero acceleration.
c) A body having constant acceleration and variable velocity.
d) A body having constant velocity and variable acceleration.

30) At what time (to the nearest second) immediately after 4 O’ clock will angle between the hands
of the clock be the same as that at 4 O’ clock?
a) 4h 42m50s b) 4h 43m38s c) 4h 43m40s d) 5h 5m27s

31) 0.5755 g of a compound, containing sulfur and fluorine only, has a volume of 255.0 mL at
288.0 K and 50.01 kPa. What is the molecular formula of this compound?
a) S2F2 b) SF2 c) SF4 d) SF6

32) Given here is a phylogenetic tree (family tree) of


greater apes. Which of the following statements
cannot be true from the tree? (mya- million
years ago)

a) Humans did not evolve from chimpanzees.


b) Humans and chimpanzees are evolutionary cousins.
c) Orangutans evolved much earlier than Humans.
d) Humans are highly evolved among great apes.

33) A liquid (A) of density 1.6gcm-3 and liquid (B) of unknown density is A B
poured into a U-tube as shown in the figure. The liquids are immiscible. If
height of A is hA= 26.6cm and height of B is hB=50 cm the density of B is hb
ha
a) 0.85 g cm-3 b) 3.01 g cm-3
c) 0.33 g cm-3 d)1.18 g cm-3

a2 b2 a
34) If a and b are two positive real numbers such that 6 , then a positive value of lies
ab b
between
a) 2 and 3 b) 3 and 4 c) 4 and 5 d) 5 and 6

8
514

35) The isomerism which exists between CH3CHCI2 and CH2CICH2CI is


a) chain isomerism b) functional group isomerism
c) positional isomerism d) metamerism

36) The term Biodiversity refers to


a)Species Diversity b) Genetic diversity
c) Ecosystem diversity d) All of the above

37) Diagram shows trajectory of a cricket ball. The set of arrows which show the direction of
the acceleration of ball at points P and
Q respectively is
(a) (b) P Q
y

(c) (d)

X
38) Sucharitha purchases x pencils at Rs x each, y pens at Rs y each and z notebooks at Rs z each.
SHe purchases altogether 50 items and pays Rs. 1000/=. The cost of y pencils, z pens and x
notebooks is
a) Rs 600/= b) Rs. 750/= c) Rs. 500/= d) Rs. 350/=

39) The metal that does not give H2 on treatment with dilute HCl is
a) Zn b) Fe c) Ag d) Ca

40) On a field trip in North America, students noticed that when threatened, Horned lizards (Genus:
Phrynosoma) squirt blood at the attackers. When the professor asked what could have been the
reason behind such behaviour of Horned lizards, one student said that certain sensory receptors
had fired and triggered a neuronal reflex culminating in increasing the pressure in their sinus
cavities until the blood vessels in the corners of the eyes burst. Another student said that it was
just an act to frighten off the predator. Thus it can be said that
a) The first response is correct, while the second is incorrect.
b) Both explanations are reasonable and can be scientifically tested.
c) The first response is biological, while the second is philosophical.
d) The first explanation is testable as a scientific hypothesis, while the second is not.

41) If temperature of a certain mass of aluminum having specific heat capacity of 0.8 J/g0C is
lowered by 60C and heat lost is 96J, then mass of aluminum is
a) 16g b) 48 g c) 60 g d) 20g

42) The number of real values of a for which the cubic equation x3 - 3ax2 + 3ax – a = 0 has all real
roots one of which is a itself, is
a) 0 b) 1 c) 2 d) 3

9
514

43) The maximum number of isomers for an alkene with molecular formula C4H8 is
a) 5 b) 4 c) 2 d) 3

44) People residing in coastal area usually do not face the problem of Thyroxin hormone deficiency
because their food intake will be rich in one of the following minerals.
a) Sodium b) Chlorine c) Iodine d) Phosphorus

45) In photoelectric effect, the maximum kinetic energy (EK) of photoelectrons depends on
frequency (f) of light incident on a metal surface of work function (ϕ). In an experiment f is
varied and EK is measured. To determine value for Planck’s constant (h)
(a) Plot Ek against φ and find intercept of best fitted line.
(b) Plot Ek against f and find slope of line of best fit.
(c) Plot Ek against φ and find slope of line of best fit.
(d) Plot Ek against f and find intercept of best fitted line.

46) Around a lawn which is of semicircular shape a pavement of uniform width is laid. If the ratio of
the area of the lawn to the area of the pavement is 25:24, then the ratio of the outer and inner
perimeters of the pavement is
7 6 5 5
a) b) c) d)
5 5 4 2 6

47) The method that cannot be used for removing permanent hardness of water is
a) adding sodium carbonate b) distillation
c) adding caustic soda d) boiling

48) In angiosperm plants, companion cell is associated with which one of the following elements?
a) Sieve tube b) Tracheids c) Vessels d) Xylem fibre

49) Essential requirement for the operation of a step down transformer is


(a) Laminated iron core
(b) Electrical connection between primary and secondary coils.
(c) Magnetic interaction between primary and secondary coils
(d) Non magnetic core.

50) Let ABC be a triangle in which AB = AC. Let D be a point on AC such that BD bisects angle B.
AB
Value of the ratio is between
BC
a) 1.0 and 1.5 b)1.5 and 2.0 c) 2.0 and 2.5 d) 2.5 and 3.0

51) Consider the following reaction: 4 PCl3(g) --> P4(g) + 6 Cl2(g). If the initial concentration of PCl3(g)
is 1.0 M, and "x" is the equilibrium concentration of P4(g), what is the correct equilibrium
relation?
a) Kc = 6x7 b) Kc = 6x7/(1.0 - x)4
c) Kc = (x)(6x) 6/(1.0 - 4x)4 d) Kc = x7/(1.0 - x)4

10
514

52) In pregnant women, foetus’s physiological functions like nourishment, respiration and excretion
are taken up by
a) Stomach of mother b) Placenta
c)Umbilical cord d) Uterus

53) On a rainy wet day, a thunder is heard 6 second after lightening. If speed of sound is 350ms-1 the
altitude of the clouds is
a) 1.8 km b) 1.9 km c) 2.1 km d) 2.5 km

54) A certain principal becomes Rs. 96800/= in 2 years if compounded annually at a certain rate of
interest. The same principal becomes Rs. 97240/= in two years if compounded half yearly at the
same rate of interest. The rate of interest is
1
a) 8% b) 8 % c) 10% d) 12 ½ %
3

55) Which properties of plastics make their disposal difficult: (I) PVC produces harmful combustion
products; (II) polyalkenes are highly flammable; (III) polyalkenes are non-biodegradable
a) I and II only b) I and III only c) II and III only d)I, II, III

56) Suresh accidentally touched silencer of his two wheeler while parking and withdrew his leg
immediately. Identify the correct order of the flow of message to the brain?
a) Receptor →Sensory neuron→CNS→Motor neuron→Effectors
b) Sensory neuron→CNS→Motor neuron→Effectors→Receptors
c) CNS→ Motor neuron→Effectors→Receptors→Sensory neuron
d) Effectors→Receptors→Sensory neuron→Motor neuron

57) There are three bodies A , B and C. Body A when brought closer to B, attract. When body B is
brought closer to C they repel. We can then conclude
a) Body A and B should have opposite charges while Body C should have the same charge as B
b) Body B and C should have same kind of charge while Body A may have opposite charge.
c) Body A and B should have same charge while Body C may have opposite charge.
d) Body C is neutral while Body A and B should have opposite charge.

58) A pen costs Rs 13/= and a note book costs Rs. 35/=. Let m be the maximum number of items
that can be bought for Rs 1000/= and n be the minimum number of items that can be bought for
the same amount. Then m + n is
a) 76 b) 88 c) 96 d) 98

59) When the pH of the environment of a protein is changed, it is said to be denatured. This is due
to:
a) breakage of peptide bonds b) breakage of disulfide links
c) loss of tertiary structure d) breakdown of R groups

60) If Brain is controlling unit of an organism, then at cellular level which cell organelle can be
comparable to Brain?
a) Chloroplast b) Ribosome c) Nucleus d) Lysosome

11
514

61) Aldebaron, the brightest star in the constellation Taurus rises at local time 7:00 pm on 1st of
October. On November 1st the star will rise at
a) 5:00 pm b) 6:00 pm c) 9:00 pm d) 8:34 pm
x
62) In the xy-plane let A be the point (5,0) and L be the line y . The number of points P on the
3
line L such that triangle OAP is isosceles is (O being the origin)
a) 1 b) 2 c) 3 d) infinitely many

63) For the reaction, 2A + B C which relationship is correct?


a) ∆*A+ = ∆*C+ b) -∆*A+ = ∆*C+
c) -2∆*A+ = ∆*C+ d) -∆*A+ = 2∆*C+

64) In some societies, ‘‘Women were solely held responsible for giving birth to female baby’’
assuming no role for men. But scientific advancement has proved men equally responsible for
the birth of either sex. Armed with this information which of the following would be the most
appropriate scenario for the birth of female child?
a) Ovum with X chromosome and Sperm with Y chromosome is FEMALE
b) Ovum with Y chromosome and Sperm with Y chromosome is MALE
c) Ovum with X chromosome and Sperm with X chromosome is FEMALE
d) Ovum with X chromosome and Sperm without chromosome is FEMALE

65) A block of mass 2 Kg placed on a floor experiences an external force in horizontal direction of
20N, frictional force of 6N and normal force of 20N. The body travels a distance of 10m under
the combined effect of all these force. If Initially body is at rest then what is the kinetic energy of
the body at the end of 4m
a) 140J b) 260J c) 60J d) 460J

66) If x3 = a + 1 and x + (b/x) =a, then x equals


a(b 1) ab 1 ab a 1 ab a 1
a) b) c) d)
a2 b a2 b a2 b a2 b

67) An electrochemical cell constructed for the reaction: Cu2+(aq) + M(s) Cu(s)+ M2+(aq) has an E =
0.75 V. The standard reduction potential for Cu2+(aq) is 0.34 V. What is the standard reduction
potential for M2+(aq)?
a) 1.09 V b) 0.410 V c) 0.410 V d) 1.09 V

68) Which one of the following is said to produce seeds exposed and they are called naked seed
plant
a)Deodar & Pinus b) Marsilea & Nostoc
c) Maize & Garden pea plant d) Spirogyra & Funaria

69) A body of mass 2 kg moving in the positive X-direction with a speed 4ms-1 collides head on with
an another body of mass 3kg moving in the negative X-direction with a speed of 1ms-1. During
collision a loud sound is heard and they both start moving together. The sound energy cannot
be greater than
a) 12J b) 14J c)15J d)17.5J

12
514

70) Let a, b, c be positive real numbers such that abc≠ 1, (ab)2 = (bc)4 = (ca)x = abc. Then x equals
a) 1 b) 2 c) 3 d) 4

71) Which radiation is the easiest to shield?


a) alpha b) beta c) gamma d) neutron

72) Identify the correct order of sequence from exterior to interior.


a) CellNucleusChromosomeDNAProtein
b) NucleusCellChromosomeDNAProtein
c) Cell NucleusDNAChromosomeProtein
d) CellNucleusProteinDNAChromosome

73) In nuclear reactor, the electrons and protons are moving in opposite direction across a small
hole in 2 second. If number of electron and protons are 2X1016 each , the current through the
hole is given by
a) 1.6 mA b) 0mA c) 6.4mA d) 3.2mA

1 1 1 1 1 1 1
74) The sum 1 ................. equals
2 3 4 5 6 2012 2013
1 1 1 1
a) ...............
1006 1007 1008 2013
1 1 1 1
b) ...............
1007 1008 1009 2013
1 1 1 1
c) ...............
1006 1007 1008 2012
1 1 1 1
d) ...............
1007 1008 1009 2012

75) Which species below has the same general shape as NH3?

a) SO32- b) CO32- c) NO3- d)SO3

76) After hearing to an influential lecture on “how to conserve environment by avoiding usage of
plastic”? Ghan Shyam resolved that he should also contribute towards protecting the
environment from plastic menace. Can you suggest him the first step how should he go about
doing this effectively
a) He should urge his parents to stop using plastic materials at home.
b) He should write a letter to the local civic body against selling plastic materials around
his locality.
c) He should practice minimising plastic usage himself.
d) He should ask his teacher to advice people on his behalf to stop usage of plastics.

13
514

77) Two infinite wires are placed parallel to each other. They carry current I1 and I2 (I2 = I1). The
magnetic field is B1 and B2 respectively. Different situation are given in column 1. The comments
on the direction and strength of magnetic field are given in column II. Match the following

Column 1 Column 2
1) The point P is at the mid P) B2>B1; Thus B2 + B1 getting into the plane of
point of the two the paper.
conductors carrying I1 I2
current in same direction.

2) The point P is on right Q) B2>B1; B2 – B1 coming out of the plane of


side of second the paper.
conductor carrying I1
current in same I2
direction

3) The point P is at the R) B1= -B2 ; B2 - B1 =0


mid point of the two
conductors carrying I1
current in opposite I2
direction.

S) B1=B2; Thus B1+B2 = 2B getting into the


4) The point P is at the plane of paper
right of the two
I1
conductors carrying I2
current in opposite
direction

a) b) c) d)
1 P 1 R 1 S 1 S
2 Q 2 P 2 R 2 R
3 R 3 S 3 P 3 Q
4 S 4 Q 4 Q 4 P

78) If the radius of the base of a cone is doubled then the slant area becomes 3 times the original
slant area. Suppose when the radius of the base of the cone is quadrupled (that is increased to 4
times), the slant area becomes k times the original slant area. Then the integer closest to k is
a) 6 b) 8 c) 10 d) 11

79) The mass of 0.2 mole of Oxygen molecule is


a) 6.4g b) 3.2g c) 1.6g d) 2.75g

80) Wuchereria is an example of


a) Arthropoda b) Annelida c) Arthropoda d) Nematoda

14
A
Indian Association of Physics Teachers
NATIONAL STANDARD EXAMINATION IN JUNIOR SCIENCE 2012-2013
Date of Examination 24th November 2012
Time: 15.00 to 17.00 Hrs

Q.P code No. 5 0 2


Five ZERO TWO

INSTRUCTIONS TO CANDIDATES
1. On the answer sheet, fill up all the entries carefully in the space provided, ONLY IN BLOCK CAPITALS. Use only BLUE or BLACK
BALL PEN for making entries and marking answers. Incomplete / incorrect / carelessly filled information may disqualify your
candidature.

2. Write the Q. P. Code No. mentioned above on YOUR answer sheet (in the space provided). Otherwise your answer sheet will
NOT be examined.

3. The question paper has 80 multiple choice questions. Each question has 4 options, out of which only one is correct. Choose the
correct answer and mark a cross in the corresponding box on the answer sheet as shown below:

Q. a b c d

22

4. A correct answer carries 3 marks and 1 mark will be deducted for each wrong answers.

5. All rough work may be done on the blank sheet provided at the end of the question paper.

6. PLEASE DO NOT MAKE ANY MARKS OTHER THAN (X) IN THE SPACE PROVIDED ON THE ANSWER SHEET. Answer sheets are
evaluated with the help of a machine. Due to this, CHANGE OF ENTRY IS NOT ALLOWED.
7. Scratching or overwriting may result in wrong score. DO NOT WRITE ANYTHING ON THE BACK OF ANSWER SHEET.

8. Use of a nonprogrammable calculator is allowed.

9. The answers / solutions to this question paper will be available on our website - www.iapt.org.in by 30th November 2012.

CERTIFICATES & AWARDS

i) Certficates to top 10% students of each centre.


ii) Merit certificates to statewise Top 1% students.
iii) Merit certificate to Nationwise Top 1% students.

10. Result sheets and the “centre top 10%” certificates of NSEJS are dispatched to the Professor in charge of the centre. Thus you
will get your marks from the Professor in charge of your centre by January 2013 end.

11. 300 (or so) students are called for the next examination - Indian National Junior Science Olympiads (INJSO). Individual letters
are sent to these students ONLY.

12. No querries will be entertained in this regard.

1
Indian Association of Physics Teachers
NATIONAL STANDARD EXAMINATION IN JUNIOR SCIENCE 2012-2013

Total time: 120 minutes Marks: 240


Only one out of four options is correct

1) If equal weights of oxygen and nitrogen are kept in separate containers at the same temperature
then.

a) Both the containers have the same number of molecules.


b) More molecules are present in the oxygen container.
c) The pressure of the nitrogen container is greater than that of the oxygen container.
d) The pressure of the oxygen container is greater than that of the nitrogen container.

2) A boy is standing on a truck which is moving with constant


speed along a straight road. On a day when wind is negligible,
the boy throws a ball vertically up with some velocity. The ball
comes back and falls
a) into boy’s hand
b) Behind the boy
c) In Front of the boy
d) Behind or in front of the boy depending on the speed of the truck and ball.

y
3) If x
75 45 z
15 0 , then which of the statement is true
a) x + y = 2z b) x + y=3z c) x – y= 2z d) x – y = 3Z

4) The pressure at the bottom of the four vessels filled with water to the same level is P1, P2, P3 and
P4 respectively. Then which of the following conclusion is correct.

P1 P2 P3 P4
a) P1 > P2> P3 > P4 b) P1 < P2< P3 < P4 c) P1 = P4 =P2> P3 d) P1 = P2= P3 = P4

1 1 1
5) If x y z 1;x y z ; x, y, z are real numbers and m . How many of the
x y z
following values of m are always possible.
i) m=6 , ii) m=8, iii) m=10 and iv) m=12
a) 1 b) 2 c) 3 d) all

3
6) Two liters of oxygen gas diffused through a membrane in 600 seconds. 0.6 liter of an unknown
gas diffused through the same membrane in half the time required for oxygen to diffuse. The
molecular weight of the unknown gas is
a) 16 b) 44 c)89 d)64

Read the following carefully and answer the questions from 7 to 10

A student treats some onion root tips with colchicine that is responsible
for arresting cell division at the metaphase stage (by dissolving spindle
fibres) and further prepared a slide of the root tip staining with
acetoorcein (stains chromatin) and observed under high power of
compound microscope. He is well aware of the cell cycle given alongside.

7) Which of the following is not true about his observation?


a) Most cells are in Interphase
b) Most cells are in the metaphase
c) No cells are in anaphase or telophase
d) Chromosomes could be observed better than a slide prepared without colchicine
treatment.

8) Why did the student choose root tips of onion


a) Roots grow fast and considerable length of tips can be used.
b) Root tips are easy to smear and stain.
c) Root tips have meristematic tissue.
d) Cell division occurs only at the root tips in plants

9) What might be the purpose of the student?

a) Observing chromosomes
b) Observing stages of cell division
c) Comparing number of cells in various stages of cell division
d) Preventing further growth of the root tips.

10) Considering the action of colchicine, it may be considered for the treatment of:
a) Hairfall b) Anemia c) Cancer d) Bacterial infection

11) The element with electronic configuration 1s2 2s2 2p6 3s2 is a/an
a) Metal b) Non- metal c) Metalloid d) Inert gas

4
12) A ball is thrown up vertically in still air with a velocity of 20ms-1. It comes back to ground. The
velocity –time graph is (g=10ms-2).
a) v
b) v
20 20
15 15
10 10
5 5
t t
1 2 3 4 -5 1 2 3 4
-10
-15
-20

v
c) d)
20 v
15
10 20
5 15
t 10
-5 1 2 3 4 5
-10
t
1 2 3 4
-15
-20

13) The sixty sixth Independence Day was on Wednesday. After how many years the next
Independence Day will be on Wednesday?
a) 6years b) 7years c)11 years d) 28 years

14) Sound waves traveling in air enters water at an angle i with the normal. It gets refracted at angle
r with
a) i>r b) r>i c) i=r d) Sound waves do not get refracted

15) What will be the remainder if the number 72012 is divided by 25?
a) 24 b)18 c) 7 d)1

16) The last electron of the element of atomic member 31 will have the following quantum numbers

n l m s

a) 3 0 0 -1/2
b) 3 1 1 +1/2
c) 4 1 -1 -1/2
d) 4 0 0 +1/2

5
Study the following diagram and answer the questions 17 to 21

17) Which of the following are tertiary consumers?

a) Snake, toads and spiders b) Foxes, Hawks and Snakes


c) Rabbit, Squirrels and Mice d) Spiders, Predacious insects and herbivorous insects.

18) The shortest and the longest food chains have _____ and ____ number of organisms respectively.
a) 2 and 6 b) 2 and 5 c) 3 and 5 d) 3 and 6

19) An insecticide is sprayed to protect the plants. Which of the following statements is true?
a) Toads and insectivorous birds will prosper as they will get ample supply of dead insects.
b) Herbivores will be greatly affected, plants will be safe and carnivores will move to
other areas and will not be affected greatly.
c) Some insects will die, some will become resistant and prolifer more and top carnivore
will be affected most.
d) Some insects will die, but there will be no long term effects as the pesticides will get
washed away.

20) What should be the preferred food of snakes to ensure minimum loss of solar energy.
a) Mice b) Toads c) Insectivorous birds d) Foxes

21) Organisms having low chances of survival produce larger number of offsprings to ensure their
survival. Which of the following can be a characteristic feature of such organisms.

a) Short lifecycle b) Better defense strategies


c)Large body size d) Good parental care

6
22) The compound which contains both ionic and covalent bonds is
a ) KCl b) CS2 c) C2H6 d) KCN

23) A particle of mass 0.5kg travelling with a velocity of 2ms-1 experiences acceleration of 2m-2 for 9s.
The workdone by the force on the particle during this period is
a) 99J b) 101J c) 190J d)396J

24) The product of three consecutive natural numbers is 124850054994. What is their average?
a) 4993 b)4994 c)4997 d) 4998

25) What is the reading of the spring balance shown in the figure below?

a) 0 b)2N
c) 4N d)6N

26) If ABCD is a cyclic quadrilateral. AB=204, BC=104, CD=195, DA=85 and BD=221. Then find AC
a) 205 b) 210 c) 220 d) 225

27) Molality of a solution is the number of

a) moles of the solute per 1000 mL of the solution.


b) moles of the solute per 1000 mL of the solvent.
c ) moles of the solute per 1000 g of the solvent.
d ) moles of the solute per 100g of the solvent.

28) A scientist wanted to check the effect of a certain hair straightening procedure on the tensile
strength of hair. The scientist could take only 20 samples. Which of the following sampling
pattern should she use to ensure that maximum parameters are controlled and the results reflect
the effect of the straightening process alone.
a) 10 girls and 10 boys of age 18 before and after the straightening procedure.
b) 10 girls and 10 boys one each of age 15, 16, 17, and so on upto 25 before and after the
procedure.
c) 10 girls and 10 boys of age 18 after the straightening procedure and another similar set
of sample group who did not undergo the procedure.
d) 10 girls and 10 boys one each of age 15, 16, 17, and so on upto 25 after the procedure
and another similar set of sample group who did not undergo the procedure.

7
29) A lady has 4 kids with bloodgroup AB and 1 kid with blood group O. If the father of these kids
have blood group B, what is the possible genotype of the lady?
a) IA IB
b) IA IO
c) IAIA
d) IBIB

30) The oxidation number of chlorine in CaOCI2 is


a) 0 b) -1 c) +1 d) +3

31) The real image of an extended object placed in front of a concave mirror is formed at a distance
of 40 cm from the object. If the image is 3 times bigger than the object, the magnitude of focal
length of the mirror is.
a) 15cm b) 10cm c) 20cm d) 5cm

32) If cot 2 (1-3sec +2sec2 )=1 and θ>900, then θ is equal to


a) 1200 b)2100 c) 3000 d) 3300

33) A person suffering from short sight is advised to wear spectacles having concave lens of power-
1.25D. What is the farthest distance he can see clearly without spectacles?
a) 60cm b) 100cm c) 120cm d) 80cm

34) Consider triangles having integer sides such that no side is greater than 4 units. How many such
triangles are possible?
a) 13 b) 17 c) 24 d)64

35) Green house effect is related to


a ) Ozone layer depletion b ) Carbon dioxide emission and absorption
c ) Nitrogen radiation d ) Oxygen radiation

36) What is the major difference between Bactiria and Virus


a) Viruses are precursors to bacteria
b) Viruses lack proteins that are present in viruses
c) Viruses use host machinery to reproduce unlike bacteria
d) Viruses have proteins whereas bacteria do not.

37) Of the following the combination of processes related to sexual reproduction are:
i Conjugation ii Fragmentation iii Gamete formation iv Zygote
a) i, iii and iv b) i, ii and iv c) ii, iii and iv d) only iii and iv

8
38) Charring of sugar in concentrated sulphuric acid is due to
a) Oxidation of sugar b) Reduction of sugar
c) Hydrolysis of sugar d) Dehydration of sugar

39) Two wires made of same material have length l and 2l. If the masses of the wires are same, the
ratio of the resistance of shorter wire to that of longer wire is
a) 1/2 b) 2 c) 1/4 d) 4

40) Find x2 + y2 +z2 if x 2+ xy + xz=135 , y2 + yz + yx=351 and z2 + zx + zy=243


a) 225 b) 250 c) 275 d) 300

41) Current passing through a wire increases by 20%. Due to Joule heating the resistance increases
by 20%. The percentage increase in the power is
a) 72.8% b) 44% c)33% d) 40%

42) In a certain number system 363+1056=1452. Find the value of (654-456) in the same number
system
a) 156 b)165 c) 178 d) 198

43) A radioactive element 90R232 emits one alpha (α) particle and then two beta (β ) particles. The
daughter element will have
a) Atomic no 90, Mass No. 228 b) Atomic no. 90 , Mass no. 232
c) Atomic no. 88, Mass No. 228 d) Atomic no. 88 , Mass no. 232

44) Dwarfness is a desirable agronomic character since such cereals:


a) Produce grains faster
b) Produce grains of better quality
c) Prove easier to harvest
d) Require lesser nutrients and water

45) The combination of the following structures possessing a single set of genome is:
i. ovary ii anther iii egg iv. Zygote v. sepal
vi. petal vii. Pollen
a) i, ii, iv, v and vi b) ii, iii, iv and vii c) only iii and vii d) only ii, iii and vii

46) The equilibrium constant for the gaseous reaction N2 + O2 → 2NO is K. The equilibrium constant
for the formation of one mole of NO will be
a) K/2 b) K c) 2 K d) k

47) A bar magnet is placed on a table. There are n number of field lines connecting North pole to
South pole of the magnet. Another identical magnet is placed on the first magnet with North Pole
on North Pole and South Pole on South Pole. The number of field lines are now
a) n b) n2 c) n/2 d)2n

9
48) What is the radius of the circumcircle of a triangle whose sides are 30cm, 36cm and 30cm in
length
a) 18cm b) 18.25cm c) 18.50cm d) 18.75cm

49) A conducting wire shown in the figure carries current I. Segments AB, BC and CD are of same
length. The direction of the magnetic field at point P is given by B A
a) into the plane of the paper
b) out of the plane of the paper
P
c) towards right
d) towards left
C D

B
50) In the adjoining figure AQ=2, QB=4, BP=3, PC=5, CR=6 and RA=4.
3
Find the area of triangle PQR.
4
P
5
a) 4.8 b) 5.2 Q
c) 5.8 d) 6.2 2

A 4 R 6 C
1
51) The solubility of a salt B2D3 is X mol L-1. Its solubility product is

a) X5 b) 6X5 c) 36X5 d) 108X5

52) If and when proteins are oxidized during respiration the energy yield is lesser than when
carbohydrates or lipids are oxidized. This is primarily due to the fact that they have:
a) relatively more oxygen
b) relatively less carbon
c) nitrogen that is not oxidized
d) relatively less hydrogen

53) 8 Grams of oxygen at NTP contain

a) 1.5 × 1023 molecules b) 3.0 × 1023 molecules


c) 6.023 × 1023 molecules d) 1.5 × 1022 molecules

54) In a nuclear reactor the fission process of each 235U-atom gives out an energy of 200MeV.
According to Einstein’s equation the amount of mass getting converted to energy in this process
is
a) 3.55X10-30Kg b) 3.55X10-38Kg c) 3.55X10-28Kg d) 3.55X10-27Kg

10
1
55) If sin x sin y a and cos x - cos y b . Then find the value of (2 - a2 - b 2 )
2
a) cos(x+y) b) cos(x-y) c) sin(x+y) d) sin(x-y)

56) A ball is projected at an angle of 450 with horizontal. In the absence of air resistance, the ball
follows
a) Elliptical orbit b) sinusoidal path
c) parabolic path d) linear path

57) A circle is inscribed in an isosceles trapezium ABCD in which AB is parallel DC. If AB=10 and
DC=30. Find the area of the circle.
a) 45π b)50π c)60π d) 75π

58) When 1g of CaCo3 reacts with 50 mL of 0.1 M HCI, the volume of CO2 produced is

a) 11.2 mL b) 22.4 mL c) 112 mL d) 224 mL

59) Neoteny or larva becoming large and developing into adult retaining larval features is common in
amphibians since they are adapted to survive:
a) in fresh water bodies where temperature and/ or iodine content is less
b) on insects that fail to supply enough nutrients
c) on a high protein diet that induces early maturation
d) in dark places and lack of light induces early sexual maturation

60) Cheese is a colloidal system of

a) Gas in solid b) Gas in liquid c)Liquid in gas d) Liquid in solid

61) 60g of ice at 00C is added to 20g of water at 400. The final temperature attained by the mixture is
(given latent heat of melting of ice = 80 cal/g and specific heat of water is 1cal/g/0C)
a) 00C b) 200C c) 100C d) 50C

62) y varies inversely as x. If x is increased by 25%, then the value of percentage change to y is
a) 80% b) 75% c) 60% d) 62.5%
63) Wavelength is
(i) The distance traveled by the wave in one period of oscillation of particles in the
medium.
(ii) The distance between two particles, which are in the same phase.
(iii) Half of the distance between two particles, which are in the same phase.
The correct definitions are
a) (i) and (iii) b) (i) and (ii) c) (i), (ii) and (iii) d) (ii) and (iii)

11
2 2 2 2 2
64) Find the value + + + +----+
15 35 63 99 9999
a) 8/33 b) 2/11 c) 98/303 d) 222/909

65) When a dilute solution of sulphuric acid is electrolysed using platinum electrodes the gas
evolved at the positive electrode is
a) SO2 b) SO3 c) H2 d) O2

66) To avoid damage to electricity cables trees are often trimmed before monsoon. Excessive
trimming leaving only trunk often leads to death of a tree. The most probable reason is that:
a) no food can be synthesized
b) no buds are left to grow into new shoot/s
c) no auxins (growth promoters) can be synthesized
d) there succumbs to the excessive trauma of wounds

67) The pH of blood is maintained within the range 7.36 -7.42 by

a) CH3COONH4 b) CH3COONa/CH3COOH
c) HCO3- / CO3- d) CH3COOH 100Ω

68) An ideal cell of emf 10V is connected across the network of 10Ω
resistors as shown in the figure. The value of the resistance R for R
which the power dissipated by the parallel combination is same
as that in 10Ω resistance is.
a) 20Ω b) 30Ω
c) 22.22Ω d) 11.11Ω 10V

69) If one of the roots of the equation x2- px + q = 0 is m times the other root then m/(1+m2) is
equal to
q p q p
a) 2
b) 2
c) 2
d) 2
p 2q q 2p q 2p p 2q

70) Three particles each of mass m are placed at the vertices of a triangle of sides r. The force
experienced by each mass is
Gm 2 Gm 2 Gm 2 Gm 2
a) 3 ( ) b) 2 ( ) c) d) 2
r2 r2 r2 r2

71) If a+b+c=1, a2 + b2 + c2 = 21 and abc=8 then find the value of (1-a)(1-b)(1-c)


a) -10 b)-18 c)-24 d)-30
72) An alkaline solution of K2HgI4 is called

a) Fehling’s reagent b) Benedict’s reagent


c) Nessler’s reagent d) Tollen’s reagent

12
73) A film of oil on every water surface arrests the growth in mosquito population since:
a) it blocks sunlight and mosquito larvae cannot get food
b) mosquito larvae suffocate
c) mosquito eggs cannot float on oil
d) mosquitoes fail to mate if water surface is not available

74) At constant temperature and pressure which of the following statements is true for the reaction.
CO2 (g) + ½ O2 (g) → CO2 (g)
a) ΔE = Δ H b) ΔE > Δ H
c) ΔE < ΔH d) Δ H and ΔE are independent of each other

75) When a car turns on a curved road, you are pushed against one of the doors of the car because of
a) inertia b) the centripetal force
c) the centrifugal force d) the frictional force

76) The distance between two spots A & B on the same bank of the river is 75km. Speed of the boat
in still water is twice as much as that of the speed of the water current of the river. The boat
travels in the river from A to B and returns back to the spot in 16 hour. What is the speed of the
boat in still water?
a) 12.5kmph b) 15kmph c) 16kmph d)18kmph

77) Michael Faraday a book binder got an opportunity to work with a scientist and later succeeded
him. Name of the scientist is
a) Hans Christian Oersted b) Humphrey Davy
c) Heinrich Lenz d) James Clerk Maxwell

78) Find the equation of the line parallel to 4x +3y=5 and having x-intercept (-3)
a) 3x + 4y +12 =0 b) 3x +4y =12
c) 4x +3y-12 =0 d) 4x + 3y +12 =0

79) The green coloured substance produced during the burning of ammonium dichromate in
fireworks is
a) CrO3 b) Cr2O3 c) CrO(O2)2 d) Cr (OH)3

80) Mud flats with mangrove plants export a lot of organic matter to waters in contact. This is
primarily because:
a) there are fewer consumers in mangrove community
b) excreta of animals in mangrove community is richer in fibers
c) detritivores are lacking in mangrove community
d) aerobic decomposers cannot survive in waterlogged mud

13
INDIAN ASSOCIATION OF PHYSICS TEACHERS
NATIONAL STANDARD EXAMINATION IN JUNIOR SCIENCE 2011 - 2012
Date of examination: 27th November 2011
Time 15.00 to 17.00 Hrs.
Instructions to Candidates
1. In addition to this question paper, you are given a separate answer sheet.
2. On the answer sheet fill up all the entries carefully in the space provided, ONLY IN BLOCK
CAPITALS.
Incomplete / incorrect / carelessly filled information may disqualify your candidature.
3. On the answer sheet, use only BLUE or BLACK BALL PEN for making entries and marking
answers.
4. The question paper contains 80 multiple-choice questions. Each question has 4 options, out
of which only one is correct. Choose the correct answer and mark a cross in the corresponding
box on the answer sheet as shown below :

Q. a b c d
22

5. Any rough work should be done only on the sheet provided at the end of question paper.

6. A correct answer carries 3 marks and 1 mark will be deducted for each wrong answer.

7. Use of nonprogrammable calculator is allowed.

8. No candidate should leave the examination hall before the completion of the examination.

9. The answers / solutions to this question paper will be available on our website - www.iapt.org.in
by 3 rd December 2011.

10. Result sheets and the "centre top 10%" certificates of NSEJS are dispatched to the Professor
in charge of the centre. Thus you will get your marks from the Professor in charge of your
centre by January 2012 end.
11. TOP 300 (or so) students are called for the next examination-Indian National Junior Science
Olympiads (INJSO). Individual letters are sent to these students ONLY.

12. No querries will be entertained in this regard.


PLEASE DO NOT MAKE ANY MARK OTHER THAN (X) IN THE SPACE PROVIDED ON
THE ANSWER SHEET.

Answer sheets are evaluated with the help of a machine. Due to this, CHANGE OF ENTRY IS NOT
ALLOWED.
Scratching or overwriting may result in wrong score.
DO NOT WRITE ANYTHING ON BACK SIDE OF ANSWER SHEET.
CERTIFICATES & AWARDS
Following certificates are awarded by the I.A.P.T. to students successful in NSEJS.

i) Certificate for "Centre Top 10%" students.

ii) Merit certificates to statewise Top 1% students.


iii) Merit certificate and a prize in the form of a book to Nationwise Top 1% students.

TO TAKE THIS EXAMINATION YOU HAVE TO BE BORN ON OR AFTER JANUARY 1, 1997


--- 3 ---
--- 4 ---
--- 5 ---
--- 6 ---
--- 7 ---
--- 8 ---
--- 9 ---
--- 10 ---
--- 11 ---
--- 12 ---
--- 13 ---
--- 14 ---
--- 15 ---
--- 16 ---
--- 17 ---
INDIAN ASSOCIATION OF PHYSICS TEACHERS
NATIONAL STANDARD EXAMINATION IN JUNIOR SCIENCE 2010 - 2011
Date of examination: 28th November 2010
Time 15.00 to 17.00 Hrs.
Instructions to Candidates
1. In addition to this question paper, you are given a separate answer sheet.
2. On the answer sheet fill up all the entries carefully in the space provided, ONLY IN BLOCK
CAPITALS.
Incomplete / incorrect / carelessly filled information may disqualify your candidature.
3. On the answer sheet, use only BLUE or BLACK BALL PEN for making entries and marking
answers.
4. The question paper contains 80 multiple-choice questions. Each question has 4 options, out
of which only one is correct. Choose the correct answer and mark a cross in the corresponding
box on the answer sheet as shown below :

Q. a b c d
22

5. Any rough work should be done only on the sheet provided at the end of question paper.

6. A correct answer carries 3 marks and 1 mark will be deducted for each wrong answer.

7. Use of nonprogrammable calculator is allowed.

8. No candidate should leave the examination hall before the completion of the examination.

9. The answers / solutions to this question paper will be available on our website - www.iapt.org.in
by 3 rd December 2010.

10. Result sheets and the "centre top 10%" certificates of NSEJS are dispatched to the Professor
in charge of the centre. Thus you will get your marks from the Professor in charge of your
centre by January 2011 end.
11. TOP 300 (or so) students are called for the next examination-Indian National Junior Science
Olympiads (INJSO). Individual letters are sent to these students ONLY.

12. No querries will be entertained in this regard.


PLEASE DO NOT MAKE ANY MARK OTHER THAN (X) IN THE SPACE PROVIDED ON
THE ANSWER SHEET.

Answer sheets are evaluated with the help of a machine. Due to this, CHANGE OF ENTRY IS NOT
ALLOWED.
Scratching or overwriting may result in wrong score.
DO NOT WRITE ANYTHING ON BACK SIDE OF ANSWER SHEET.
CERTIFICATES & AWARDS
Following certificates are awarded by the I.A.P.T. to students successful in NSEJS.

i) Certificate for "Centre Top 10%" students.

ii) Merit certificates to statewise Top 1% students.


iii) Merit certificate and a prize in the form of a book to Nationwise Top 1% students.

TO TAKE THIS EXAMINATION YOU HAVE TO BE BORN ON OR AFTER JANUARY 1, 1996


--- 3 ---
--- 4 ---
--- 5 ---
--- 6 ---
--- 7 ---
--- 8 ---
--- 9 ---
--- 10 ---
--- 11 ---
--- 12 ---
--- 13 ---
--- 14 ---
--- 15 ---
--- 16 ---
--- 17 ---
GENERAL INSTRUCTIONS FOR NATIONAL STANDARD
EXAMINATION IN JUNIOR SCIENCE (IJSO) TEST PAPER

YEAR : 2009-10 (STAGE-I)

Time : 1 Hr. Max. Marks : 150

In addition to this question paper, you are given a separate answer sheet.

On the answer sheet fill up all the entries carefully in the space provided, ONLY IN BLOCK
CAPITALS.
Incomplete/incorrect/carelessly filled information may disqualify your candidature.

On the answer sheet, use only BLUE or BLACK BALL PEN for making entries and marking
answers.

This papers has two parts. In Part A1(Q. Number 1 to 30) each question has FOUR
alternatives, out of which only one is correct. Choose the correct alternative and mark a
cross (X) in the corresponding box on the answer sheet below.

Q.NO. A B C D

For Example
22
X
Part A2 (Q. Number 31 to 40) has four alternatives in each question, but any number of
these (4, 3,2,1) may be correct. You have to mark A L L correct alternatives and mark a cross

Q.NO. A B c D I
(X) for each like
32
X X
— — i...

Any rough work should be done only on the sheet provided at hte end of question paper.

For Part A1, each correct answer gets 3 marks, wrong answer gets a penalty of 1 mark. For
Part A2 full marks are 6 for each question, you get them when All correct answers only are
marked.

Use of nonprogrammable calculator is allowed.

Top 300 (or so) students are called for the next examination-Indian National Astronomy
Olmpiads (INJSO). Individual letters are sent to these students ONLY.
YEAR-2009-10_NSEJS(STAGE-l)

(PART A-1)
Only one out of four options is correct:

1. A beaker containing water is placed on the platform of a digital weighing machine. It reads 1100 g. A metal
body of density 8g /cc and mass 200 g is suspended in water in the beaker (without touching the walls of
the beaker). It is attached by a suitable string fixed to some support Now the reading of weighing machine
will b e :
(A)1100g (B)1125g (C)1275 (D)1300g

2. The sum of all natural numbers less than 400 which are NOT divisible by 6, is :
(A) 13266 (B)66534 (C) 79800 (D)93066

3. In the figure shown, the bigger circle has radius 1 unit. Therefore, the radius of smaller circle must be

(A) 42 +1

4. Three containers A, B and C of equal volume contain N , N O . and C 0 respectively at the same tempera-
2 2

ture and pressure, The ascending order of their masses i s :


(A) A, C, B (B) C, A, B (C)B.C.A (D) C, B, A

5. A 3.7 litre 1.0 M N a O H solution is mixed with another 5 litre 0 30 M NaOH solution. The molarity of the
resulting solution is :
(A) 0.80 M (B)0.10M (C)0.73M (D)0.59M

6. The animal body is formed of many cells, but the cells show no coordination to form tissues in
(A) protozoans (B) coelonterates (C) sponges (D) flat worms

7. An indicator of S 0 pollution is :
2

(A) moss (B) lichens (C) algae (D) pteridophytes

8. The one that C A N N O T be used for D N A fingerprints is :


(A) leucocytes (B) erythrocytes (C) hair bulbs (D) sperms
A rainbow has circular shape because:
(A) the earth is spherical. (B) rain drops are spherical
(C) the sun is spherical (D) none of these

An observer in a car P moving towards north with speed v observes that another car Q is moving towards
east with the same speed v. The true velocity of Q is :

(A) v towards east (B) 42 v towards north-east

(C) v towards south-east (D) v 42 towards south-west

A body describes uniform, accelerated motion along a straight line. Which of the following displacement
time graphs shown represent the motion correctly ?

(A) (I) only (B)(1) and (IV) only


(C) (I), (III) and (IV) only (D) (I), (II), (III) and (IV) all

Transition metal ion in amethyst is :


3 3+ 3 :
(AJCr * (B)Fe (C)Mn * (D)Co

Among the following the most powerful oxidizing agent is -


(A)0 2 (B)KCI0 + 0 3 2 (C)H 02 (D)O,

The half-life period of radioactive element is 14 hours. The fraction of the radioactive element that disintegrates
in 56 hours i s :
(A) 0.125 (B) 0.625 (C) 0.9025 (D) 0.9375
+ =
15. Three line segments in a plane have lengths a, b and c. No two of these are parallel. If V a Vb Vc •

then :
(A) can form an acute angled triangle. (B) can form a right angled triangle.
(C) can form an isosceles triangle (D) cannot form a triangle.

16. A cell, an ammeter and a voltmeter are all connected in series. The ammeter reads a current / and the
voltemeter a potential difference V. If a torch bulb is connected across the voltmeter, then.
(A) both / and V will increase (B) both / and V will decrease
(C) / will increase but V will decrease (D) / will decrease but V will increase

17. Dispersion of white light into its consitituent colours occurs during
(A) reflection at a plane mirror
(B) reflection at a concave mirror.
(C) internal reflection inside a spherical drop of water
(D) refraction at the boundary of a transparent medium.

18. Ice at 0° C is put in a closed container and heat is supplied to it continuously at a uniform rate. Which of
the following graphs gives the temperature variation with time correctly ?

19. Two circles of radii 2 and 3 cm touch each other externally The length of direct common tangent to the
two circles will be :

(A) 2 ^ 6 cm (B) V26 cm (C)5cm (D) 2.4 cm


20. Formalin is 4 0 % aqueous solution of:
(A) formic acid
(B) freon gas
(C) formaldehyde
(D) a mixture of formic acid and formaldehyde in a ratio 1:1

21. "Green fuel" means


(A) fuel obtained from plant leaves (B) green coloured fuel
(C) chemicals used for the growth of plants (D) fuel obtained from plastic waste

22. Which one of the following is a true fruit ?


(A) Pear (B) Coconut (C) Apple (D)Cashewnut

23. The number of chromosomes can be counted at:


(A) anaphase (B) interphase (C)metaphase (D) prophase

24. Which one of the following is true fish ?


(A) Star fish (B) Gold fish (C) Silver fish (D) Hag fish

25. Newton deduced the inverse square law of gravitation:


(A) by observing motion of an apple falling from a tree.
(B) by using kepler's laws of planetary motion.
(C) by studying motion of different objects in the laboratory.
(D) by using data obtained from Cavendish experiment.

26. If D = a + b + c where a and b are consecutive integers and c = ab, then VD is :


2 2 2

(A) always an even integer. (B) always an odd integer.


(C) an integer, odd or even (D) sometimes an irrational number

2 2 2 2 2 2
27. If a + 2b = 7, b + 4c = - 7 and c + 6a = - 14, then the value of (a + b + c ) is -
(A) 14 (B)25 (C)36 (D)47

28. The part of the digestive system that digests lipids in the food is :
(A) stomach (B) duodenum (C) ilium (D) large intestine

29. One mole of oxalic acid is equivalent to :


(A) 0.5 mole of N a O H (B) 1 mole of NaOH (C) 1.5 mole of N a O H (D) 2 mole of NaOH

2
"secG-1" " sinG-T
The trigonometric expression : cot 9 + sec 02
has the value
1 + sinG 1 + sec0

(A)-1 (B)0 (C)1 (D)2


(PART A-2)
In question 31 to 40 any number of options (1 or 2 or 3 or 4) may be correct. You are to identify
all of them correclty to get 6 marks. Even if one answer identified is incorrect or one correct
answer is missed, you get zero score.

31. During contraction of muscle fiber:


(A) I bands get reduced in length (B) A bands retain the length
(C) I bands retain the length (D) A bands get reduced in length

32. Out of the following the salt/ s that has /have pH value higher that 7.5 / is/are :
(A) sodium hydrogen carbonate (B) sodium sulphate
(C) sodium chloride (D) sodium carbonate

33. In case of the circuit shown below, which of the following statements is/are true ?

1'

VVVV
R, R 2 R 3

(A) R R and R are in series.


r 2 3

(B) R and R are in series.


2 3

(C) R and R are in parallel.


2 3

R2R3
(D) The equivalent resistance of the circuit is R,+ R R
2 + 3

34. Which of the following statements is/are true in case of waves ?


(A) Transverse vibrations are set on the string of a guitar.
6
(B) A s compared to light, a sound wave produced on the moon will take a time about 10 times longer to
reach the earth.
(C) A sound note of wavelegth 1 cm in air cannot be heard by a man.
(D) Sound travels faster in water than in air.

35. In a rectangle A B C D the lengths of sides A B , B C , C D and DA are (5x + 2y + 2) cm, (x + y + 4) cm,
(2x + 5y - 7) cm and (3x + 2y - 11) cm respectively. Which of the following statements is /are true ?
(A) One of the sides of the rectangle is 15 cm long.
(B) Each diagonal of the rectangle is 39 cm long.
(C) Perimeter of the rectangle is 102 cm.
2
(D) Area of the rectangle is 560 c m

36. What is true about white muscles ?


(A) Number of mitochondria is few.
(B) They possess very small quantity of myoglobin.
(C) The amount of sarcoplasmic reticulum is high
(D) They depend on anaerobic process of energy.

37. If U = {X| X is a point on straight line AB}, P = {M|M is a point on ray AB}, Q = {N|N is a point on ray BA}
and R = {L|L is point on segement AB}, then which of the following statements is/are true ?

(A)PflQ = R (B)P'UQ' = Q U P (C)[P'UQT = R (D)PriQ' = P ' U Q


38. In bacterial photosynthesis, usually :
(A) C 0 2 is not fixed (B) 0 is not released
2 (C) H 0 is oxidized
2 (D) chlorophyll taps light

39. A body traveling along a straight line with a uniform acceleration has velocities 5m/s at a point Aand 15 m/
s at a point B respectively. If M is the midpoint of A B , then :

V5-l"
(A) the ratio of times taken by the body to cover distance MB and A M is

(B) the velocity at M is 5 7 5 m/s

5(y5-l)
(C) average velocity over A M is m/s
2 2
(D) the product of the acceleration and the distance A B is 100 m / s

40. Out of the following the metals, the one/ s that is/ are sonorous is /are :
(A)Zn (B)Na (C)Hg (D)Cu
INDIAN ASSOCIATION OF PHYSICS TEACHERS
NATIONAL STANDARD EXAMINATION IN JUNIOR SCIENCE 2008 - 2009
Date of examination: 23rd November 2008
Time 16.15 to 17.15 Hrs.
Instructions to Candidates
1. In addition to this question paper, you are given a separate answer sheet.
2. On the answer sheet fill up all the entries carefully in the space provided, ONLY IN BLOCK
CAPITALS.
Incomplete / incorrect / carelessly filled information may disqualify your candidature.
3. On the answer sheet, use only BLUE or BLACK BALL PEN for making entries and marking
answers.
4. This paper has two part. In part A1 (Q. Nos. 1 to 30) each question has FOUR alternatives, out
of which only one is correct. Choose the correct alternative and mark a cross (X) in the
corresponding box on the answer sheet.

Q.No. a b c d
For example,
22
Part A2 (Q. Nos. 31 to 40) has four alternatives in each question, but any number of these (4,
3, 2, or 1) may be correct. You have to mark ALL correct alternatives and mark a cross (X) for
each, like
Q.No. a b c d
42
5. Any rough work should be done only on the sheet provided at the end of question paper.
6. For Part A1, each correct answer gets 3 marks; wrong answer gets a penaltyof 1 mark. For
Part A2 full marks are 6 for each question, you get them when ALL correct answers only are
marked.
7. Use of nonprogrammable calculator is allowed.
8. No candidate should leave the examination hall before the completion of the examination.
9. The answers / solutions of this question paper will be available on our website - www.iapt.org.in
by 30 th November 2008.
10. Result sheets and the "centre top 10%" certificates of NSEA are dispatched to the Professor
in charge of the centre. Thus you will get your marks from the Professor in charge of your
centre by January 2009 end.
11. TOP 300 (or so) students are called for the next examination-Indian National Astronomy
Olympiads (INAO). Individual letters are sent to these students ONLY.
12. No querries will be entertained in this regard.
PLEASE DO NOT MAKE ANY MARK OTHER THAN (X) IN THE SPACE PROVIDED ON
THE ANSWER SHEET.
Answer sheets are evaluated with the help of a machine. Due to this, CHANGE OF ENTRY IS NOT
ALLOWED.
Scratching or overwriting may result in wrong score.
DO NOT WRITE ANYTHING ON BACK SIDE OF ANSWER SHEET.
CERTIFICATES & AWARDS
Following certificates are awarded by the I.A.P.T. to students successful in NSEA.
i) Certificate for "Centre Top 10%" students.
ii) Merit certificates to statewise Top 1% students.
iii) Merit certificate and a prize in the form of a book to Nationwise Top 1% students.

YOU HAVE TO BE BORN BETWEEN JANUARY 1, 1994 AND DECEMBER 31, 1995
--- 1 ---
--- 2 ---
--- 3 ---
--- 4 ---
--- 5 ---
--- 6 ---
--- 7 ---
--- 8 ---
INDIAN ASSOCIATION OF PHYSICS TEACHERS
NATIONAL STANDARD EXAMINATION IN JUNIOR SCIENCE 2014 -15
Q. Paper Code: JS 530
1) Three identical vessels carrying equal amount of water are placed in three lifts. Lift A is
accelerating upwards, lift B is accelerating downwards while lift C is moving up with
constant velocity. The pressure at a depth h from free surface in the three vessel is
measured as pA , pB and pc then which of the following is true
a) pA = pC > pB b) pA > pC > pB c) pA > pC = pB d) pA = pC = pB
Solution (b): The pressure at a depth h is given by In case of lift moving up
Lift moving down Lift moving with constant velocity

2) In the reaction,
2KClO3 →2KCl + 3O2
What is the volume of oxygen released under NTP conditions when 36.75g of KClO3 is
heated?
a) 3.6 litres b) 7.2 litres c) 10 litres d) 1.8 litres

Solution (c) 245g of KClO3 → 67.2L of Oxygen gas


36.75g → 10 Litres
3) Figure shows a square grid of order 3, which of the following is correct
formula for the total number of squares in a similar grid of order n.
(a) b) c) d)

Solution (b)
4) If the distance between genes - W. X. Y, and Z on a chromosome are as follows: from W-Y is
18 units, W-X is 26 units, W-Z is 40 units, X-Y is 8 units and X-Z is 14 units, the sequence of
W, X, Y, Z genes on the chromosome would be:

a) W, X, Y, Z. b) X, Y, W, Z. c) Y, W, X, Z. d) W, Y, X, Z.

Solution : (d)

5) In a plant, 30 megaspore mother cells are generated. If all the ovules are fertilised, how
many seeds are expected to be formed?

a) 30 b) 60 c) 90 d) 120

Solution : (a)

6) A water filter advertisement claims to provide 8 litres of water per hour. How much time
does it take to fill four bottles of 1.5 litres each?

a) 2 hr b) 1 hr c) 45 min d) 30 min

Solution (c)

7) Which among the following salts will not change the pH of water on addition
a)Sodium chloride b) Sodium cyanide
c) Sodium bicarbonate d) Sodium carbonate
Solution (a): Apart from sodium chloride rest are salts of weak acids.

1
8) A particle starting from rest is moving with uniform acceleration in a straight line. The
percentage increase of the displacement of the particle in 9th second compared to that in
the immediate previous second is about
a) 8.3% b) 13.3% c) 20.6% d) 24.5%

Solution (b): The displacement in nth second is given by . Thus percentage change in
displacements

9) An inflated balloon with a heavy rock tied to it submerges in water. As the balloon sinks
deeper and deeper, the buoyant force acting on it

a) increases b) decreases
c) remains nearly unchanged d) Initially increases and then decreases

Solution (b): b is correct. Other choices indicate failure to see that the balloon is compressed by
water pressure—and compression is greater with greater depth—displacing less water.

10) For a first order reaction, the ratio of the times taken for completion of 99.9% and 50% of
the reaction is
a) 8 b) 9 c) 10 d) 12
Solution (c): 100→ 50 → 25 → 12.5 → 6.25→3.125 → 1.6 → 0.8 →0.4 → 0.2 → 0.1. Each arrow
corresponds to one t1/2 required. Numbers represent amount of reactant left after each t1/2

11) If set of marbles, of radius 5 cm, is poured into a cube of side 1 m. The maximum number
of marbles that can be filled into the box are

a) 1000 b) 2000 c) 1500 d) 3000

Solution (a)
12) Most of the insects have egg, larva, pupa and adult stages in the life cycle. This is primarily
due to :

a) relatively short adult phase. b) terrestrial habitat they have adapted to.

c) eggs storing little reserved food. d) flying mode of locomotion majority have.

Solution : (c)

13) Which of the following has been proved to contribute to the transport of water in vasular
plants?

i. Positive root pressure


ii. Hyrophilic cell walls
iii. Capillarity
iv. Transpirational pull
v. Cohesion between water molecules
a) i, ii, iii, iv and v b) only I, iii and v

c) only ii, iv and v d) only I, ii, iv and v

Solution : (c)

2
14) A round table cover has six equal designs as shown in the adjacent
figure. If the radius of the cover is 4 cm, then cost of making the
designs at the rate of Rs 10.00 per cm2 (round off your answer to a
nearest rupee) is

a) Rs 85 b) 86 c) 87 (d) 90

Solution (c)

15) Which of the following series of elements have nearly the same atomic radii?
a) F, Cl, Br, I b) Na, K, Rb, Cs c) Li, Be, B, C d) Fe, Co, Ni, Cu
Solution (d) In transition elements of one row, electrons are added to the same orbital i.e 3d in
this case.
16) A particle is moving along a straight line. Its velocity time
graph is as shown in the adjacent figure. Then
Match the following
Physical quantity Remarks
(i) Acceleration at 4 second (p) Positive
(ii) Velocity at 4 second (q) Negative
(iii) Direction of motion at 2 second (r) Zero

a) (i) is (p); (ii) is (q) and (iii) is (r)


b) (i) is (q); (ii) is (r) and (iii) is (p)
c) (i) is (r); (ii) is (r) and (iii) is (p)
d) (i) is (q); (ii) is (p) and (iii) is (r)

Solution (b): Slope of v-t graph represents acceleration and hence in this case negative
Velocity at 4 s from v-t graph is zero
V at 2 s is positive and hence direction of motion is positive

17) A rectangular parallelepiped with sides a, b and c in the ratio 3:2:1 is kept on a uniformly
rough horizontal surface as shown in the figures below. The value of limiting friction is

(i) (ii) (iii)

a) Minimum in (i) b) Minimum in (ii) c) Minimum in (iii) d) Same in all cases


Solution (d): Limiting value of friction depends on the mass of the object but not the area of
surface of the object in contact with the floor.

18) Which of the following has the maximum number of unpaired electrons?
a) Ti 3+ b) V 3+ c) Fe3+ d) Fe2+
Solution (c)Fe0 has s2 d6 configuration. Fe 3+ has s0 d5 configuration. All 5 are unpaired.

3
19)The houses of a row are numbered consecutively from 1 to 49. Find the value of x such that
the sum of the numbers of houses preceding the house numbered x is equal to the sum of
the numbers of the houses following it.

a) 25 b) 37 c) 35 d) No such value exists

Solution (c)
20) Urea is the principle excretary waste in larval as well as adult phases of:

a) Cockroach b) Frog c) Crab d) Starfish

Solution : (b)

21) Use of excessive NKP fertilizers has resulted in:

i. Reduction in number as well as species of nitrogen fixing bacteria


ii. Increase in number as well as types of denitrifying bacteria
iii. Increase in the proportion of coarse particles in soil.
iv. Increase in number as well as types of ammonifying microbes
v. Increase in number as well as types of nitrifying bacteria
a) i, ii, iii, iv and v b) only ii, iv and v c) only i and ii d) only i, ii and iii

Solution : (d)

22) Along a road lie an odd number of stones placed at intervals of 10 metre. These stones
have to be assembled around the middle stone. A person can carry only one stone at a
time. If a man starts from one of the end stones, and by carrying them in succession he
covers 3 km to pile all stones at the centre. The number of stones is then

a) 12 b) 15 c) 25 d) 30

Solution (c)

23) The following variation of properties is generally seen in the periodic table.
a) Atomic radius and ionization energy both increase across a period.
b) Atomic radius increases and ionization energy decreases across a period.
c) Atomic radius decreases and ionization energy increases across a period.
d) Atomic radius and ionization energy both decreases across a period.
Solution (c)

24) The erythrocytes separated from human blood were mixed with certain fluids on a slide and
observed under the microscope. Which of the following will be the expected result?

a) With distilled water the cells swell and eventually burst.


b) With serum the cells clump and coagulate.
c) With sea water the cells undergo no apparent change.
d) With tap water cells shrink and appear cremated.
Solution : (a)

25) The largest of the jelly-fishes grow over 1 meter in diameter and can survive without any
skeletal support due to:

a) rapid beating of cilia creating an upthrust.


b) the bottom dwelling habit.

4
c) high salinity and subsequent buoyancy of sea water.
d) upwelling currents in water.
Solution : (c)

26)The diagram shows a road


network. All vehicles drive in
one direction from A to B.
Numbers represent the maximum
flow rate (capacity of roads) in
vehicles per hour. The maximum
number of vehicles that can drive
through the network every hour is

a) 315 b) 215 c) 240 d) 340

Solution (b)

27) An excess of NaOH solution is added gradually to an aqueous solution of ZnSO4. Which of
the following will happen?
a) A white precipitate is formed which does not dissolve in excess NaOH.
b) A green precipitate is formed which dissolves in excess NaOH.
c) A white precipitate is formed which dissolves in excess NaOH.
d) No observable change occurs.

Solution (c): Forms Zinc hydroxide first (white ppt.) on excess addition of hydroxide, forms water
soluble zinc tetrahydroxide complex.
28) If two bodies of different masses, initially at rest, are acted upon by the same force for the
same time, then both bodies acquire the same
a) Velocity b) momentum c) acceleration d) kinetic
energy
Solution (b) Product of force and time (Impulse) is equal to increase in momentum.

29) It is more difficult to walk on a sandy road than on a concrete road. The most appropriate
reason for this is

a) sand is soft and concrete is hard


b) the friction between sand and feet is less than that between concrete and feet
c) the friction between sand and feet is more than that between concrete and feet
d) the sand is grainy but concrete is solid
Solution (d)

30) In which of the following series of transition metal ions, all metal ions have 3d2 electronic
configuration?
a) Ti+, V4+, Cr6+, Mn7+ b) Ti2+, V3+, Cr4+, Mn5+ c) Ti3+, V2+, Cr3+, Mn4+ d) Ti4+, V3+, Cr2+,
Mn3+

Solution (b): From Ti to Mn, configuration changes from s2d2 to s2d5. On oxidation they all lose
their electrons starting from the s orbital. So in option B all have d2 configuration.

5
31)A piece of wire 60 cm long is cut into two parts, one of them being 24 cm long. Each part is
then bent to form a square. The ratio of the area of the larger square to the smaller square
is

a) 11/3 b) 7/4 c) 3/2 d) 9/4

Solution (d)

32) In the cells of oil seeds which of the cell organelles have to be more active?

a) Mitochondria. b) Smooth Endoplasmic Reticulum.

c) Rough Endoplasmic Reticulum. d) Nucleoli.

Solution : (b)

33) Which of the following sugars tastes most sweet?

a) Fructose. b) Ribose. c) Sucrose. d) Lactose.

Solution : (a)

34)Scientists in an R & D company made three design improvements on a car: the first saves
50% of fuel, the second saves 30% of fuel and the third saves 20%. If the company
implements all three design changes at once, the new car will consume fuel that is ____ %
of the fuel consumption of normal car.

a) 50% b) 28% c) 100% d) 20%

Solution (b)

35) Aluminum is extracted from its oxide by


a) Calcination b) Reduction
c) Thermal decomposition d) Electrolysis

Solution (d)
36) The magnetic force on a moving charged particle can change the particle’s

a) speed only b) direction only


c) Both speed and direction d) neither of speed nor direction

Solution (b). The magnetic force is perpendicular to displacement and hence no work is done. The
kinetic energy does not change and hence speed remains the same. The change in acceleration is
due to change in direction of velocity only .

37) A ray of light is incident on system of mirror as shown in the adjacent


figure. What is the total deflection ( ) of the ray when it emerges out
after two reflections?
a) 2200 b)1800 c)1400 d) 1200

Solution (c):

6
38) The oxidation number of sulphur in sodium thiosulphate (Na2S2O3) is
a) +1 b) +2 c) +3 d) +4

Solution (b): 2+2x-6=0, therefore x= +2


39)The adjacent figure is a modification of the Switzerland flag to suit the
problem! Five identical small squares form the central cross. The length of
each side of the big square is 10 m. If the area of the white cross is 20% of
the area of the square flag, then the length of the side of the small square
is

a) 1.75 m b) 2.25 m c) 1.6 m d) 2 m

Solution (d)

40) The algae belonging to which group can sustain normal growth at the greater depth of
ocean?

a) Green algae. b) Blue-green algae. c) Brown algae. d) Red algae.

Solution : (d)

41) Snakes, the cold blooded animals, flick their bifid tounge often to:

a) sample air for chemoreceptors. b) sense vibrations in earth.


c) sense the nature of substratum. d) sense the temperature of air.
Solution : (a)

42) We all know that the sky appears to touch the ground at a distance. The distance at which
we perceive the sky to touch the ground is called horizon. The reason for the perception is
due to the fact that the Earth is a sphere (almost) and not a flat surface. Which of these
pictures below accurately depict the horizon for a person standing on a high rise building
like Burj Khalifa in Dubai? (Here, ‘h’ represents the height of the building while line ‘H’
represents the horizon)

Solution (a)

43) Sulphuric acid is manufactured by the contact process in which sulphur dioxide reacts with
oxygen in presence of a catalyst. If 5.6 moles of SO2 reacts with 4.8 moles of O2 and a large
excess of water, the maximum number of moles of H2SO4 that can be obtained is.
a) 5.6 b) 11.2 c) 4.8 d) 1.4

7
Solution (a): 2SO2 + O2 + 2H2O → 2 H2SO4. Limiting reagent is SO2. water and oxygen are in excess.
2 moles SO2 gives 2 moles sulphuric acid. therefore 5.6 moles of sulphuric acid is formed.
44) The element essential for determining the three dimentional structure of proteins is:

a) carbon. b) hydrogen. c) nitrogen. d) sulphur.

Solution : (d)

45) The general indigestion experienced by a patient suffering from obstructive jaundice is due
to:

a) the lack of emulsification of lipids.


b) the low pH in the intestine not supporting optimal activity of enzymes.
c) the acceleration of intestinal peristalsis reducing the retention time for food.
d) the diffusion of bile pigments in blood suppressing secretion of digestive juices.

Solution : (b)

46) A number is said to be a triangular number if it is the sum of consecutive numbers


beginning with 1. Which one of the following is not a triangular number

a) 1431 b) 190 c) 506 d) 28

Solution (c)

47) The equivalent weight of MnSO4 is half its molecular weight when it is converted to
a) Mn2O3 b) MnO2 c) MnO4- d)MnO42-

Solution (b): If equiv weight has to be half of molecular weight, then Mn2+ must be converted to
either Mn0 or Mn4+ so that change in oxidation number becomes equal to 2.
48) A light source of diameter 2 cm is placed 20 cm behind a circular opaque disc of diameter
4cm. Shadow is formed on a screen at a distance of 80cm. The ratio of the area of umbra
and penumbra shadow regions is equal to.
a) 0.22 b) 0.18
c) 0.58 d) 0.11

Solution (a): Consider triangle ABC and BDE and therefore Y1= 4 cm. Thus the
area of umbral region is .
Consider triangle HBG and BFE. and therefore Y2 =
12cm. The area of penumbral region is
. The ratio is 0.22.

49) Consider the following two statements.


Statement 1: The direction of acceleration of a particle must be always same as that of
velocity.
Statement 2: Acceleration is the rate of change of velocity.

8
Choose the correct option
a) Statement (1) is correct while statement (2) is wrong
b) Statement (1) and (2) are correct
c) Statement (1) is wrong while statement (2) is correct
d) Statement (1) and (2) are wrong.
Solution (C): Acceleration is in the direction of change in velocity and not velocity. Thus they can be
in different direction.

50) Rust is a mixture of


a)FeO + Fe(OH)2 b) FeO + Fe(OH)3
c) Fe2O3 + Fe(OH)3 d) Fe3O4 + Fe(OH)3
Solution (c)
51) If the distance between A and B is 230 km, B and C is 120 km, C and A is 350 km. Also, if the
distance between C and D is 200 km, distance between D and B is 330 km and distance from
A to E is 100 km and distance between D and E is 570 km. The diagram (not drawn to scale)
that represents this graphically is

Solution (b)

52) Which of the following contains the same number of atoms as 13.5 grams of aluminum?
a)10 g of sodium b) 10 g of magnesium
c) 20 g of potassium d) 20 g of calcium

Solution (d): 13.5g of Aluminum is equal to 0.5 moles = 0.5 x 6.022x1023 atoms. Calcium - 20 grams
is half mole.
53) Consider the following two statements. Statement 1 is an assertion of a concept while
Statement 2 is the reason.
Statement (1): When red light travels from air to water, for observer in water it appears to
be still red.
Statement (2): Colour of light is associated with frequency and frequency does not change,
when it travels in different medium.
Choose the correct option
a) Statement (1) is correct while statement (2) is wrong
b) Statement (1) and (2) are correct

9
c) Statement (1) is wrong while statement (2) is correct
d) Statement (1) and (2) are wrong.
Solution (b):

54) A spring of spring constant 7600 Nm-1 are attached to a block


of mass 0.25 kg as shown in figure. Frequency of oscillation on
frictionless surface is
(a) 27.76 Hz (b) 39.26 Hz (c) 9681.5 Hz (d) 98.39 Hz

Solution (a): Frequency of oscillation is

55) The following data was recorded for the reaction A + B →Product at 298K.
Experiment
No. [A] [B] Rate of reaction

1 1.00M 0.15M 4.20 x 10-3


2 2.00M 0.15M 8.40 x 10-3
3 1.00M 0.30M 8.40 x 10-3
From the above data one can conclude that
a) Rate ∝ [A]2[B] b) Rate ∝ [A][B]2 c) Rate ∝ [A][B] d) Rate ∝ [A]2[B]2
Solution (c): When amount of A is double the reaction rate doubles and as B doubles and A is kept
same, again rate doubles. Indicating that rate is proportional to both A and B.

56) The sum of 2 digits x and y is divisible by 7. What can one say about a 3 digit number
formed by these two digits.

a) xyx is divisible by 7 b) xxy is divisible by 7


c) xyx is divisible by 72 d) yyx is divisible by 7

Solution (a)

57) Most of the microbes employed in commercial fermentation for producing antibodies are:

a) thread bacteria. b) yeasts.


c) eubacteria. d) ascomycete fungi.

Solution : (a)

58) Most of the cellular RNA is synthesised and stored respectively in:

a) cytoplasm and ribosomes. b) ribosomes and cytoplasm.


c) nucleus and ribosomes. d) ribosomes and nucleus.
Solution : (c)

59)A number of bacteria are placed in a glass. 1 second later each bacterium divides in three,
the next second each of the resulting bacteria divides in three again, and so on. After one
minute the glass is full. When was 1/9th of the glass full?

a) 15 sec b) 58 sec c) 45 sec d) 38 sec

Solution (b)

10
60)A number x is a rational number if there exists integers p and q such that x = p/q. This is the
definition of rational numbers in which,

a) both p & q can be zero b) both p & q should not be zero


c) p can be zero but not q d) q can be zero but not p

Solution (c)

61) There is a solution of 1 Litre HCl of pH 5. When 9 L of water is added to this solution, the pH
turns out to be
a) pH 5 itself b) pH 10 c) pH 4 d) pH 6
Solution (d): Solution originally contains 10-5 moles of HCl in liter. As 9 liters of water is added,
solution still contains 10-5 moles of HCl but in 10 Litres of water. Therefore the solution was
diluted to a concentration of 10-6 Molar i.e. pH = 6

62) A wave is sent along a string by oscillating at one end. If the tension in the string is
increased then speed of the wave and wavelength of the wave

a) both increase b) speed increases, wavelength decreases


c) both decreases d) wavelength increases, speed decreases
Solution (a): Speed is proportional to and hence it increases. Wavelength is
proportional to speed and hence even that increases as well.

63) Clock A based on oscillations of spring and clock B is based on pendulum motion. Both the
clocks keep the same time on earth. If they are taken to a planet having half the density of
earth and twice the radius
a) then A runs faster than B. b) B runs faster than A.
c) both will run at same rate as earth d) both will run at equal faster rate than earth.
Solution (c): Time period of Spring clock A is . Thus it is independent of g and

hence on the planet. Time period of clock B is

. Thus time period will be same as that on earth.

64) Assuming ideal gas behavior, which among the following gases will have the least density
under room temperature and pressure.
a) Nitrogen b) Oxygen c) Ozone d) Fluorine

Solution (a): For all ideal gases the volume occupied by 1 mol of molecules is the same. Therefore
the lightest gas is the one that has least molecular weight.
65)The least positive integer, , such that 2 divides , 3 divides , 4 divides , 5 divides
and 6 divides is

a) 52 b) 120 c) 720 d) 62

Solution (d)

66) Which of the following places having same number of species is considered most
biodiverse?

11
a) species belonging to more taxa.
b) many of the species endemic.
c) many of the species economically important.
d) species adapted to greater number of habitats.
Solution : (b)

67) Axolotl, the Mexican Salamander, shows ‘neoteny’ or larva becoming sexually mature (adult).
Which of the following characters indicate larval features in it?

i. Naked skin
ii. External gills
iii. Lidless eyes
iv. Laterally compressed tail
v. Clawless digits

a) i, ii, iii, iv and v. b) only i, ii, iv and v


c) only ii, iii, iv and v. d) only ii and iv.

Solution : (d)

68) The solution set of the inequality is

a) Set of all positive real numbers b) set of all non-negative real numbers
c) set of all real numbers except -1 d) Set of all numbers satisfying
Solution (b)

69) Which among the following organic compounds is likely to have more than one possible
structure?
a) CH4 b) C3H8 c) C2H4 d) C3H6

Solution (d): C3H6 is the only one which can represent either as propene or cyclopropane. While
the rest represent only one single molecule.

70) In the circuit B1, B2, and B3 represent identical bulbs.


Consider the case
(i) With resistance R4 (ii) without the resistance R4
(R4 comparable with resistance of bulb)
a) B1,B2 and B3 glow with equal brightness in both cases.
b) B1 brightest in (i) and in (ii) B2 and B3 become brighter and B1 dimmer compared to case
(i).
c) B2B3 brightest in case (i) and B1 becomes brighter in (ii).
d) B1brightest in (i) and B2 becomes brighter in comparison to B3 in (ii).

Solution (b): With R4: the effective resistance of the circuit decreases. Hence current is
larger. This leads to increase in brightness of B1. The current is dived into three branches
and B2 and B3 will be dimmer.
Without R4: The effective resistance is higher and current reduces in the circuit. B1 is
dimmer than in case 1. The current in the branches of B2 and B3 is more and hence they are
brighter than in case 1.

12
71) Three identical resistors each of resistance R are
connected in the following four configurations.
Rank the arrangement in the order of their
equivalent resistors from highest to lowest.

a) i,ii,iii & iv b) iv,iii,ii & i

c) i,iii,iv & ii d) ii,iv,iii & i

Solution (c): Effective resistance in case of i)3R ii) R/3 iii) 3R/2
iv)2R/3

72) Given below are the structures of the famous molecules called Aspirin and Paracetamol.
Which among the listed functional groups do the two molecules put together NOT contain?

a) Ketone b) Ester c) Alcohol d) Carboxylic acid


Solution (a): The functional groups contained by both of them are – Alcohol (phenolic), amide,
ester and carboxylic acid.

73) Number plate of a vehicle consists of 4 digits. The first digit is the square of second. The
third digit is thrice the second and the fourth digit is twice the second. The sum of all 4
digits is thrice the first. The number is

a) 1132 b) 4264 c) 9396 d) 1642

Solution (c)

74) The pteridophytic character that is considered to have led to the evolution of gymnosperms
is:

a) homospory. b) heterospory.

c) furcate venation. d) sporophylls distinct from vegetative leaves.

Solution : (b)

75) Seeds trapped in crevices of rocks soak in water, swell and cause fragmentation of rock.
The process involved is termed:

a) imbibition. b) osmosis. c) Tyndall effect. d) water potential.

13
Solution : (a)

76) If the highest common factor of a, b and c is 1, where a, b and c belong to the set of natural
numbers, then the highest common factor of (a X b) and c is

a) c b) a X b c) 1 d) Insufficient data

Solution (c)

77) If a firecracker burns with emission of red colour light, which cation is it likely to contain?
a) Sodium b) Copper c) Iron d) Lithium
Solution (d):

78) A positively charged insulator is brought in contact with an uncharged conductor then
a) conductor acquires positive charge due to conduction
b) conductor acquires negative charge due to induction
c) conductor acquires positive charge due to induction
d) conductor cannot acquire any charge.
Solution (bonus) status : DELETED

79) Two infinite wires carrying identical current are placed at position A and C
normal to plane of the paper as shown in the adjacent figure. The resultant
magnetic field (B) at a point P on the perpendicular bisector is
a) Along perpendicular bisector pointing towards line AC
b) Along Line joining PC and pointing towards C
c) Along line joining PA and pointing towards A
d) Along line parallel to AC and pointing towards right

Solution(d) : The magnetic field due to current at position A and C are as shown in the
adjacent figure. The field directions are perpendicular to line joining the P and current
carrying wire. Thus resultant is parallel to AC.

80) When an incandescent bulb is switched on and the outer glass bulb also gets heated up.
This is due to
a) Conduction and convection of heat from filament to the bulb by the medium inside the
bulb at lower temperatures and by radiation of heat at higher temperature
b) Convection of heat from filament to the bulb by the medium inside the bulb at all
temperatures
c) Radiation of heat from filament to the bulb at all temperatures

14
d) Conduction of heat from filament to the bulb by the medium inside the bulb at higher
temperatures and by radiation of heat at lower temperature

Solution (a):

15
Indian Association of Physics Teachers
NATIONAL STANDARD EXAMINATION IN JUNIOR SCIENCE 2013-2014
Solution for question paper version 514
Total time: 120 minutes Marks: 240
Only one out of four options is correct

1) Solution: (a) The ray traces the path as indicated in the figure below. The angle of emergence is
also 450. The net deviation (angle d) is given by 900 (see figure)

450
d

2) Solution: d)

3) Answer (B)
4) Solution: b)
5) Solution: d)
6) Solution: b)
7) Solution: (d)
Using conservation of energy we have 100 mgh . Where h is vertical height from where the
h 10
object was released. Therefore h=10m. The angle of inclination is sin 0.5 . Thus
l 20
θ=300.

8) Solution: c)

9) Answer (C)

10) Solution (a) :


At any point inside the liquid, pressure is same in all direction given by ρgh. The force is therefore
ρgh(∆A) in a direction normal to inclined plane.

11) Solution: b)

12) Answer (D)


13) Solution (b): Light travels slower in water while sound travels faster in water. This makes light
bend towards normal and sound bend away from normal. Statement (i) is correct. Statement (ii)
is also correct while it does not explain the above phenomenon. Thus option (b) is most
appropriate.

14) Solution: b)
15) Answer (B)
16) Solution: a)
17) Solution: b)
18) Solution: a)
19) Solution: (C)
GM
The acceleration due to gravity is given by g
R2
GM GM
g g0 (2 R)2 R2
0.75
g0 GM
R2
20) Solution: a)

21) Answer (C)

22) Solution: (a) Initially particle is accelerating. The velocity of particle (which is slope of the curve)
should increase continuously. When particle decelerates, slope should decrease. This
represented only by option (a)

23) Solution: c)

24) Answer (D)

25) Sol: (c)


Solution: 100W bulb has lower resistance compared to 50W bulb since P = V2/R. The 100W bulb will
glow brighter when connected in parallel.
While connected in series, same current flows through both the bulbs. The power dissipated by each
bulb is given by I2 R and since 50 W bulb has a higher resistance it will glow brighter.

26) Solution: b)

27) Answer (C)

28) Solution: c)
29) Solution: (d)
Solution: Acceleration is the rate of change of velocity and if the velocity is constant acceleration by
definition is zero and cannot vary. All other situations are possible.

30) Solution: b)
31) Answer (c)
32) Solution: d)

33) Solution: (a) A B


The pressure along the horizontal solid line must be same. Thus
hp p g hq q g
hb
26.6X1.6=50.0Xhb. Thus hb = 0.85gcm-3. ha

34) Solution: d)

35) Answer (C)

36) Solution: d)

37) Solution: (b)


The acceleration is provided by gravity of earth and hence always vertically downward.

38) Solution: b)

39) Answer (c)


40) Solution: b)

41) Solution ( d )
The heat absorbed by Al; Q=Cm(∆t). That is 96=0.8XmX6. Thus m= 20g.

42) Solution: c)

43) Answer (B)

44) Solution: c)

45) Solution (b): The photoelectric equation is given by


EK hf . Thus EK is plotted against f. The slope will be equal to h.

46) Solution: b)

47) Answer (D)

48) Solution: a)
49) Solution: (c)

50) Solution: b)

51) Answer (C)


52) Solution: b)

53) Solution: (c)


Light travel at an enormously high speed and hence lighting is seen almost instantaneously while
sound takes 6 second. Thus h=350X6=2100m or 2.1 km.

54) Solution: c)

55) Answer (B)


56) Solution: a)
57) Solution (b):
When two bodies attract, one of the body may be charged or neutral. But when they repel then
surely both of them are having same charge. Thus repulsion is a sure test of charge and not attraction.

58) Solution: d)

59) Answer (C)

60) Solution: c)

61) Solution: a) Mean Stellar time period is 4 m lesser than mean Solar time period. As a result star
rises 4 m earlier than the previous day. In two months it will rise (4X30=120m) earlier. That is it
will rise at 7-2 = 5:00pm
62) Solution: c)

63) Answer (D)

64) Solution: c)
65) Solution: (a)
The work done by Normal force and gravitational force (weight) is zero. Work done by external force
is W=20X10=200J. The work done by friction is W/ =-6X10=-60J. The net workdone is W=200-
60=140J. This workdone results in change in Kinetic energy.
140J=Kf - Ki =Kf-0. Thus Kf = 140J

66) Solution: c)

67) Answer (A)

68) Solution: a)
69) Solution (c)
According to conservation of momentum; 2X4 + 3X(-1)=(3+2)v
Thus final velocity is 1ms-1. The initial kinetic energy is
1 1 1 1
Ki m1 v21 m2 v22 2X 42 3X 12 17.5 J
2 2 2 2
1 1
The final kinetic energy is K f Mv2 X 5X 12 2.5 J . The change in kinetic energy is
2 2
converted to other forms including sound. If we assume that only sound energy is given out then it is
given by 17.5-2.5=15J.

70) Solution d)

71) Answer (A)

72) Solution: a)

73) Solution (d) : The current is the rate of change of charge. Thus
2X 1016 X1.6X10 19
2 X1016 X( 1.6X10 19
)
I 3.2mA
2
74) Solution: b)

75) Answer (A)

76) Solution: c)

77) Solution: (b)


The magnetic field around the a current carrying conductor is given in the figure.

(1) In this case B1 is getting into plane of paper while B2 is coming out. Thus net magnetic field is
B=B1-B2=0 (R).

B1 B2

(2) In this case B1 and B2 is getting into plane of paper. Since P is closer to I2. Thus net magnetic
field is B2+B1 (P)

B1 +B2

(3) In this case B1 and B2 are equal and getting into plane of paper. Thus net field is B1+B2 (S)

B1 B2

(4) In this case B1 is getting into plane of paper while B2 is coming out. B2>B1. Thus net magnetic
field is B2-B1 and coming out (Q).
78) Solution d)

79) Solution: a)
80) Solution: d)
A
Indian Association of Physics Teachers
NATIONAL STANDARD EXAMINATION IN Junior Science 2012-2013
Solution (Version A)
Total time: 120 minutes Marks: 240
Only one out of four options is correct

1) Solution: c
2) Solution: (a); Consider an observer on the truck. According to him truck is stationary. He will
observe that ball goes up and lands in the boy’s hand.

3) Solution: b;

4) Solution: (d); Pressure depends on the height of the water. Thus all at bottom of all the vessels
are same.

1 1 1 1 1 1 1 1 1
5) Solution: b ; 9 and therefore 6& 8 is always true
x y z x y z x y z
6) Solution : c

7) Solution: b

8) Solution: c
9) Solution: a

10) Solution: c

11) Solution: a

12) Solution: (b); As ball goes up the velocity reduces due to acceleration due to gravity. Thus
v 20
acceleration is negative. It momentarily comes to rest at t 2s . After this ball
g 10
accelerates in the downward direction. The velocity becomes more and more negative. Thus
option b is correct option

13) Solution: a; Sixty Sixth Independence day 15/08/12 Wednesday. 2013 (67th) is Thursday, 2014
(68th) is Friday, 2015 (69th ) is Saturday, 2016 (70th ) is Monday, 2017(71th) is Tuesday, 2018 (72nd )
is Wednesday.
14) Solution: (b); Speed of sound in water is greater than the speed in air. Thus water is an
acoustically rarer medium compared to air. Therefore the refracted ray deviates away from
normal. That is r>i.
15) Solution: d;72012 = (74)503 = (2401)503= ---------------= 01; Remainder is 01.

16) Solution:c

17) Solution: a
18) Solution d
19) Solution: c

20) Solution: a
21) Solution: a

22) Solution: d
1
23) Solution: (a); Work done = Change in K.E; w m( v22 v12 ) . The velocity v2=v1+at ;
2
1 396
v2 = 2 + 2X9 =20ms-1. Thus w = X 0.5(202 22 ) 99 J .
2 4

24) Solution: d ; n3-n = (n-1)(n)(n+1). If n=4998 then (5000-2)3=125000000000-150000000+60000-


8=124850059992. Therefore 4997X4998X4999=124850054994
25) Solution: (b); The reading in the spring balance is the tension in the spring. If you pulled the
spring with 2N force on both the sides, then the tension is 2N. Even in the case of spring balance
suspended from a rigid support and pulled by 2N force, the rigid support exerts force of 2N on it
similar to above case.
26) Solution: c ; (ABXCD) +(ADXBC)=ACXBD ; (17X12)X(13X15)) + ((17X5)X(13X8))=ACX(13X17).
Therefore AC = ((12X15)+(5X8)) = 180+40 = 220.

27) Solution: c

28) Solution: a
29) Solution: b
30) Solution: a
31) Solution: (a); v-u = 40 cm and v/u =3 Solving the two equations we have v= 20 cm and u= 60cm.
1 1 1 -1 1 1
+ = ; and f 15cm
u v f 20 60 f
32) Solution: c ; cot2θ (1-3secθ+2sec2θ) =1 (multiplying by sin2θ).
3 2
cos2 (1 ) sin 2 ; cos2θ-3cosθ+2=1-cos2θ.
cos cos2
Therefore (cosθ-1)(cosθ-2)=(1-cosθ)(1+cosθ); 2cosθ=1; θ=600 or θ= -600.
Thus θ=360-60 = 3000
33) Solution: (d); The purpose of the spectacle is to make the person see objects very far away (at ∞)
though with his own ability can see up to a distance d. That is object at ∞ should produce the
1 1 1 1 1
image at distance d due to spectacles. ; . That is d=-0.8m or 80cm.
f d 1 d
1.25
34) Solution: a;
35) Solution: b

36) Solution: c
37) Solution: a
38) Solution: d
39) Solution: (c); Same mass of same wire implies same volume. A1l = A2 (2l) ; A1=2A2 ; The ratio of
l
R A1 1
resistance = S
Rl 2l 4
A2
40) Solution: c
x2+xy+xz=x(x+y+z)=135=27X5; Therefore x=5
y2+yz+yx=y(x+y+z)=351=27x13; Therefore y=13
z2+zx+zy=z(x+y+z)=243=27X9= Therefore z=9
x2+y2+z2=25+169+81=275

P
41) Solution: (a); P=I2R ; P` = (I+0.2I)2 (R+0.2R) = 1.22X1.2(I2R)=1.728 P; Thus 0.728
P
42) Solution: b; The base of the number system is 7; (363)7 = 3X49 + 6X7 +3 = (192)10
(1056)7=1X343 + 5X7+6 =(384)10 ; (1452)7 = 1X343 + 4X49 + 5X7 + 2 = (576)10
(654)7 = 333; (456)7 = 237; (165)7 = 96
43) Solution: a

44) Solution: d
45) Solution: c

46) Solution: d
47) Solution: (d); When two magnets are combined the pole strength increases and hence the
Magnetic field doubles. The number of field lines is proportional to Magnetic field thus it should
be double.

48) Solution: d; AB=30; BC=(BM+MC)=18+18=36, AC=30


AM = (BMXMC)/MD = AB2 BM 2
Therefore 900 324 576 24 AM . Thus MD=18X18/24 = 13.75.
Diameter of the circle = 37.5 and radius = 18.75
49) Solution:(b); Using thumb rule, the magnetic field due to segment AB, BC and CD is all in the
same direction out off the plane of the paper.
B
3
50) Solution: c; A(∆PQR) = A(∆BQR)+A(∆BPR)-A(∆BPQ)
4
A(∆BPQ) = ½ X4X3=6 ( angle B =900) P
A(∆BQR) = 4/15 X A(∆ABC) = 6.4
5
A(∆BPR) = 9/40 XA(∆ABC) = 5.4 Q
A(∆ABC) = ½ X6X8 = 24 ( angle ABC = 90 ) ; A(∆PQR) = 6.4 + 5.4 - 2
0
6
=5.8 A 4 R 6 C
1
51) Solution: d
52) Solution: c
53) Solution: a
54) Solution: (c); mC 2 E; mX(3X10 8 ) 2 200 X10 6 X1.6 X10 19
J . The mass getting
converted is given by, ∆m = 3.55X10-28Kg.

55) Solution: a ; a2 = (sin(x) + sin(y))2 ; b2 = (cos(x)-cos(y))2 and 2 = sin2x + sin2y + cos2x + cos2y
2-a2-b2 = 2(cos(x)cos(y)-sin(x)sin(y)) = 2 cos (x+y)

56) Solution: (c); The equation for displacement along the horizontal direction and vertical direction
g
are, x=(ucosθ)t and y =(usinθ)t – (1/2)gt2 and Thus y (tan )x 2
x 2 or y=ax-bx2.
2(u cos )
This implies the path is parabolic

57) Solution: d ; Height of the trapezium = diameter of the circle; AD=BC= 5+15=20; BP=15-5=10;
PC=10√3. The radius is 5√3 area of circle = πr2.

58) Solution: d
59) Solution: a
60) Solution: d
61) Solution: (a); Heat required to melt 60g of ice is Qice=60X80=4800 cal. The heat given out by 20 g
of water when cooled from 400C to 00C is Qwater=40X20X1=800cal. Since Qwater is less than Qice,
entire ice will not melt. Thus the final temperature will be 00C.
1
62) Solution: a ; y ; Let y=100 and x=4. Therefore K=xy=400. If x=5 (increased by 25%). Thus
x
K 400
y 80
x 5
63) Solution: (b); From the definitions of wavelength

2 1 1 2 1 1 2 1 1 2 1 1
64) Solution: c; ( ) ; ( ); ( ) ---------- ( )
15 3 5 35 5 7 63 7 9 9999 99 101
2 2 2 2 1 1 101 3 98
( )
15 35 63 9999 3 101 303 303

65) Solution: d

66) Solution: c

67) Solution: c
68) Solution: (d); For the power dissipation to be same, the effective resistance of parallel
1 1 1 100X 10
combination must be 10Ω. That is, ;R 11.11
10 R 100 100 10

69) Solution: a; If roots are 1 and m then the quadratic equation is (x-1)(x-m)=0. Thus x2-(m+1)x+m=0.
p = (m+1) and q=m. Therefore p2-2q = (m+1)2-2m = m2+1 and q=m.

m
70) Solution: (a); The resultant force on mass m is F F
600
2
Gm
FR F2 F2 2F 2 (cos 60); FR 3F 3
R2

m m

71) Solution: b ; (1-a)(1-b)(1-c) = 1 – (a+b+c) + (ab + bc + ca) – abc;


[(a+b+c)2 – (a2+b2+c2)]/2 = ab + bc +ca = (1-21)/2 =-10 ;
(1-a) (1-b) (1-c) = 1 – (1) + (-10) – (8) = -18

72) Solution: c
73) Solution: b

74) Solution: b
75) Solution: (c);
75 75
76) Solution: a ; 16 ; (time = distance/effective speed)
2x x 2x x
75 25
16 ; 16x=100 ; x=6.25 and 2x =12.5
x x

77) Solution (b)

78) Solution: d; Slope of the line parallel to 4x+3y=5 is m = -4/3 . The point through which the line is
4 4x 12
passing (-3,0). Equation (y-y1)=m(x-x1) ; y 0 ( x 3)
3 3
4x+3y+12=0

79) Solution: b

80) Solution: d

Answer Keys

Version A
Q no Answer Q no Answer Q no Answer Q no Answer
1 C 21 A 41 a 61 a
2 A 22 D 42 b 62 a
3 B 23 A 43 a 63 b
4 D 24 D 44 d 64 c
5 B 25 B 45 c 65 d
6 C 26 C 46 d 66 c
7 B 27 C 47 d 67 c
8 C 28 A 48 d 68 d
9 A 29 B 49 b 69 a
10 C 30 A 50 c 70 a
11 A 31 A 51 d 71 b
12 B 32 C 52 c 72 c
13 A 33 D 53 a 73 b
14 B 34 A 54 c 74 b
15 D 35 B 55 a 75 c
16 C 36 C 56 c 76 a
17 A 37 A 57 d 77 b
18 D 38 D 58 d 78 d
19 C 39 C 59 a 79 b
20 A 40 C 60 d 80 d
Junior Science 2011-12

23
24
25
26
27
28
29
JUNIOR SCIENCE 2010-11

1. b 21. b 41. d 61. c

2. b 22. c 42. d 62. d

3. c 23. d 43. b 63. c

4. d 24. c 44. c 64. c

5. d 25. b 45. a 65. a

6. a 26. b 46. d 66. c

7. c 27. c 47. b 67. a

8. d 28. b 48. c 68. c

9. c 29. c 49. c 69. b

10. a 30. b 50. a 70. c

11. d 31. d 51. b 71. a

12. a 32. b 52. c 72. b

13. c 33. b 53. b 73. b

14. d 34. c 54. d 74. b

15. c 35. b 55. a 75. c

16. b 36. b 56. a 76. c

17. a 37. a 57. a 77. d

18. d 38. a 58. c 78. b

19. d 39. b 59. b 79. a

20. d 40. c 60. b 80. a

--- 20 ---
Answers - NSEJS 2009 – 2010
1. B
2. B
3. D
4. A
5. D
6. C
7. B
8. B
9. D
10. B
11. D
12. C
13. D
14. D
15. D
16. C
17. D
18. B
19. A
20. C
21. A
22. B
23. C
24. B
25. B
26. B
27. A
28. B
29. D
30. B
31. a, b
32. a, d
33. c, d
34. a, c, d
35. a, b, c
36. a, b, c,d
37. a, c
38. b, d
39. a, b, c, d
40. a, d
NSEJS 2008-09 Answer Sheet
1. c
2. c
3. d
4. d
5. d
6. c
7. b
8. c
9. b
10. d
11. a
12. a
13. a
14. b
15. a
16. c
17. b
18. c
19. d
20. b
21. b
22. d
23. a
24. c
25. b
26. a
27. a
28. c
29. c
30. b
31. a, b, d
32. a, d
33. a, b, c, d
34. b, c
35. b, c
36. a, b, c
37. c, d
38. a, c
39. b, c
40. a, d

You might also like